0% found this document useful (1 vote)
244 views

FINALEvidence

This document provides an overview of key concepts in evidence law, including relevance, authentication of writings, the best evidence rule, witnesses, and impeachment. It discusses the basic principles of relevance and similar occurrences that may be relevant. It also outlines several policy-based exclusions of evidence, such as liability insurance, subsequent remedial measures, settlements, and offers to pay medical expenses. The document reviews methods of authenticating writings, the best evidence rule and its exceptions, competency and opinion testimony of witnesses, and various methods of impeachment.

Uploaded by

kashitampo
Copyright
© © All Rights Reserved
We take content rights seriously. If you suspect this is your content, claim it here.
Available Formats
Download as DOCX, PDF, TXT or read online on Scribd
0% found this document useful (1 vote)
244 views

FINALEvidence

This document provides an overview of key concepts in evidence law, including relevance, authentication of writings, the best evidence rule, witnesses, and impeachment. It discusses the basic principles of relevance and similar occurrences that may be relevant. It also outlines several policy-based exclusions of evidence, such as liability insurance, subsequent remedial measures, settlements, and offers to pay medical expenses. The document reviews methods of authenticating writings, the best evidence rule and its exceptions, competency and opinion testimony of witnesses, and various methods of impeachment.

Uploaded by

kashitampo
Copyright
© © All Rights Reserved
We take content rights seriously. If you suspect this is your content, claim it here.
Available Formats
Download as DOCX, PDF, TXT or read online on Scribd
You are on page 1/ 78

Evidence 2016 Summer BarBri NY Bar

EVIDENCE
Prof. Vincent C. Alexander
St. John’s University School of Law

I. RELEVANCE 関連性............................................................................................................5
A. BASIC PRINCIPLES......................................................................................................................5
1.
DEFINITION:.............................................................................................................................5
2.
THE RULE:.................................................................................................................................5
B. SIMILAR OCCURRENCES 類似事実の発生....................................................................................6
1. Overview.....................................................................................................................................6
2. Plaintiff’s accident history. 原告の事故歴..................................................................................6
3. Similar Accidents Caused by Same Event or Condition..............................................................7
4. Intent in Issue 故意......................................................................................................................7
5. Comparable Sales on Issue of Value 売買価格の比較(製品の価値の証拠)........................7
6. Habit 習慣 (FRE 406).................................................................................................................8
7. Industrial Custom as Standard of Care 注意義務の基準としての業界慣行.............................9
C. POLICY-BASED EXCLUSIONS[政策的証拠排除]............................................................................9
1. Liability Insurance (FRE 411) 第三者に対する賠償責任保険への加入の事実......................9
2. Subsequent Remedial Measures (SRMs) 事後の救済対応があった事実 (FRE 407)............10
3. Settlements 和解に関する事実の呈示 (FRE 408)..................................................................10
4. Pleas and Plea Discussions in Criminal Cases 答弁及び司法取引 (FRE 410)........................12
5. Offer to Pay Hospital or Medical Expenses 医療費支払の申し出 (FRE 409)........................12
6. REVIEW AND SUMMARY: Policy Based Exclusions............................................................13
D. CHARACTER EVIDENCE 性格証拠...............................................................................................13
1. Introduction...............................................................................................................................13
2. Criminal Cases (FRE 404(a)) 刑事事件....................................................................................14
3. Civil Cases 民事事件での性格証拠.........................................................................................18
E. DEFENDANT’S OTHER CRIMES OR MISCONDUCT FOR NON-CHARACTER PURPOSE.................19
1. The General Rule:......................................................................................................................19
2. The General Exception 一般的な例外......................................................................................19
3. THE MIMIC RULE...................................................................................................................19
4. Method of proof of MIMIC-purpose crimes:.............................................................................20
5. MIMIC Evidence in Civil Cases 民事事件での MIMIC:.........................................................20
6. Timing:......................................................................................................................................20
7. Burden of proof: 証明責任.......................................................................................................21
8. Other Requirements for MIMIC Evidence MIMIC による証拠の他の要件:..........................21
F. OTHER SEXUAL MISCONDUCT TO SHOW PROPENSITY IN SEX-CRIME PROSECUTION OR CIVIL
ACTION (FRE 413, 414, 415)...............................................................................................................21
II. AUTHENTICATION OF WRITINGS 書面の真正 (FRE 901)..................................22
A. METHODS OF AUTHENTICATING WRITINGS 書面の真正性の証明方法....................................22
B. SELF-AUTHENTICATING DOCUMENTS 真正性が推認される書面.............................................23
C. AUTHENTICATION OF PHOTOGRAPHS 写真の真正性.................................................................24
III. BEST EVIDENCE RULE 最良証拠ルール...................................................................25
A. OVERVIEW....................................................................................................................................25
1. Terminology..............................................................................................................................25
2. THE RULE:(FRE 1002)............................................................................................................25
3. Definition:..................................................................................................................................25
B. QUESTION #1: WHEN DOES THE BEST EVIDENCE RULE APPLY?.................................................25
1. THE RULE:...............................................................................................................................25

1
Evidence 2016 Summer BarBri NY Bar

2.
BEWARE:.................................................................................................................................26
C. QUESTION #2: WHAT QUALIFIES AS THE “ORIGINAL WRITING”? 原本とは何か?.................27
1. Definition of original:................................................................................................................27
2. Definition of duplicate:..............................................................................................................27
3. THE RULE for duplicates.........................................................................................................27
D. QUESTION #3: WHEN WILL NON-PRODUCTION OF THE ORIGINAL BE EXCUSED?......................27
1. THE RULE:...............................................................................................................................27
2. APPLICATION:........................................................................................................................27
E. “ESCAPES” FROM THE REQUIREMENTS OF THE BEST EVIDENCE RULE....................................28
IV. WITNESSES 証人............................................................................................................29
A. COMPETENCY OF WITNESS, IN GENERAL 証人の適格性...........................................................29
B. “DEAD MAN’S STATUTE” 生前の口頭約束に関する制定法(民事事件のみ)......................29
1. THE FEDERAL RULE:............................................................................................................29
2. THE RULE IN SOME STATES:..............................................................................................29
3. Definition:..................................................................................................................................30
4. Waiver:......................................................................................................................................30
C. FORM OF QUESTION (LEADING QUESTIONS) 誘導尋問 (FRE 611)............................................31
D. WRITINGS IN AID OF ORAL TESTIMONY 口頭証言の補助書面.................................................32
1. Present Recollection Refreshed (FRE 612)................................................................................32
2. Past Recollection Recorded (Hearsay Exception)......................................................................33
E. OPINION TESTIMONY 意見証拠...................................................................................................34
1. Lay Witness 一般人証人(FRE 701).........................................................................................34
2. Expert Witness 専門家証人(FRE 702).....................................................................................34
3. Ultimate Issues 主要事実に関する事項..................................................................................36
F. CROSS-EXAMINATION 反対尋問....................................................................................................37
G. CREDIBILITY AND IMPEACHMENT, IN GENERAL 信用性と弾劾.................................................37
1. Definitions:................................................................................................................................37
2. Bolstering Own Witness............................................................................................................37
3. Impeachment of Own Witness 自分の証人の弾劾(FRE 607)..................................................38
H. IMPEACHMENT METHODS 弾劾の方法.........................................................................................38
1. Prior Inconsistent Statements 以前の供述との不一致.............................................................38
2. Bias, Interest or Motive to Misrepresent 不実表示に関する偏見、利害関係又は動機........40
3. Sensory Deficiencies 知覚上の障害.........................................................................................40
4. Reputation or Opinion About Witness’s Bad Character for Truthfulness..................................40
5. Criminal Convictions 刑事有罪判決(FRE 609)........................................................................41
6. Prior Bad Acts (without conviction) that reflect adversely on witness’s character for
truthfulness (FRE 608) 犯罪ではないが悪行(真実性についての証人の性格に反するもの)42
7. Contradiction 矛盾....................................................................................................................44
I. REHABILITATION 信憑性回復.......................................................................................................46
V. PRIVILEGES 特権...........................................................................................................48
A. INTRODUCTION: (PURPOSES ARE TO FOSTER BENEFICIAL RELATIONSHIP.)..................................48
B. ATTORNEY-CLIENT PRIVILEGE...................................................................................................48
1. Definitions:................................................................................................................................48
2. Rationale:...................................................................................................................................48
3. Elements:...................................................................................................................................48
4. Waiver.......................................................................................................................................50
5. Exception...................................................................................................................................50
C. PHYSICIAN-PATIENT PRIVILEGE 医者と患者の特権.................................................................51
1. Rule:..........................................................................................................................................51
2. Elements:...................................................................................................................................51
4. Losing the privilege:..................................................................................................................51

2
Evidence 2016 Summer BarBri NY Bar

D. SPOUSAL PRIVILEGES 配偶者間の特権.......................................................................................52


1. Spousal Immunity (Spousal Testimony Privilege).....................................................................52
2. Spousal Communication Privilege 配偶者間通信の特権.........................................................52
3. Exceptions (applicable to both privileges).................................................................................53
4. Review and Summary: Spousal Privileges.................................................................................53
VI. HEARSAY 伝聞証拠........................................................................................................54
A. DEFINING HEARSAY (FRE 801)...................................................................................................54
B. NON-HEARSAY STATEMENTS 非伝聞証拠..................................................................................54
1. PURPOSE is key:......................................................................................................................54
2. Hearsay Tip:..............................................................................................................................54
C. THE 3 PRINCIPAL CATEGORIES OF NON-HEARSAY PURPOSES..................................................55
1. Verbal Acts (Legally Operative Words 法的効力発生文言)...................................................55
2. To Show Effect on Person Who Heard or Read the Statement..................................................55
3. Circumstantial Evidence of Speaker’s State of Mind 供述者の心理的状態の状況証拠.........55
D. PRIOR STATEMENTS OF TRIAL WITNESS 公判証人の以前の供述 (EXCLUSION FROM
HEARSAY).............................................................................................................................................56
1. The Rule:...................................................................................................................................56
2. The Three Exceptions (exclusions: from definition of hearsay):................................................56
E. PARTY ADMISSIONS (STATEMENT OF AN OPPOSING PARTY) 当事者の承認(FRE 801(D)).......57
1. THE RULE:...............................................................................................................................57
2. Terminology:.............................................................................................................................57
3. Rationale:...................................................................................................................................57
4. Adoptive Admission..................................................................................................................57
5. Vicarious Admission 代位承認.................................................................................................58
F. HEARSAY EXCEPTIONS TO KNOW FOR THE BAR EXAM【伝聞例外】.....................................59
F-#1 DECLARANT UNAVAILABLE 供述不能.....................................................................................60
1. Forfeiture by Wrongdoing 違法行為による喪失(FRE 804)....................................................60
2. Former Testimony in former proceedings 以前の証言(FRE 804)............................................60
3. Statement Against Declarant’s Interest 供述者の利益に反する供述(FRE 804).....................62
4. Dying Declaration 臨終供述(FRE 804)....................................................................................63
F-#2 SPONTANEOUS STATEMENTS 自然発生的供述.......................................................................64
5. Excited Utterance 興奮による発声(803(2)).............................................................................64
6. Present Sense Impression 当該時点での認識による印象(FRE 803(1)).................................64
7. Statement of Then-Existing Mental, Emotional, Intent, or Physical Condition..........................65
8. Statement for Purpose of Medical Treatment or Diagnosis........................................................66
9. Business and Public Records 事業及び公的記録 (FRE 803(6)(8)).........................................67
F-#3 OTHER EXEPTION.....................................................................................................................69
10. Other Exception.....................................................................................................................69
G. THREE FINAL MISCELLANEOUS HEARSAY ISSUES......................................................................71
1. Procedural Issue: Laying the Foundation for Hearsay Exceptions.............................................71
2. Note on the Confrontation Clause 対面条項(刑事事件のみ)..............................................71
3. Hearsay Declarants and Impeachment 伝聞証人及び弾劾(FRE 806)......................................73
VII. JUDICIAL NOTICE 裁判所による確知.......................................................................74
VIII. REAL EVIDENCE 物的証拠...................................................................................... 76
A. DEFINITION:..................................................................................................................................76
B. AUTHENTICATION RULE...............................................................................................................76
IX. PROCEDURAL CONSIDERATIONS: DIVIDING RESPONSIBILITY BETWEEN
THE JUDGE AND THE JURY..................................................................................................77
A. BURDENS OF PROOF.....................................................................................................................77
B. PRELIMINARY FACTS....................................................................................................................77
3
Evidence 2016 Summer BarBri NY Bar

4
Evidence 2016 Summer BarBri NY Bar

INTRODUCTION
3 major topics: relevance, witnesses, and hearsay.
3 minor topics: Authentication, best evidence, and privileges.
5 topics not covered in Lecture:

Big: read 3 sections not covered in the lecture. Judicial notice, real evidence and procedural considerations,
Federal rules of evidence in Multistate. Don’t need to worry about common law, majority/minority.

I. RELEVANCE 関連性
A. BASIC PRINCIPLES
1. DEFINITION:
Evidence is RELEVANT if it has any tendency to make a material fact more probable or
less probable than would be the case without the evidence. (FRE 401)
当該証拠がなかった場合と比べて、証拠が重要事実が存在する確率をより高めるか、低くするよう
な傾向にある場合、当該証拠は関連性があるといえる。
2. THE RULE:
All relevant evidence is admissible, unless:
a) some specific exclusionary rule is applicable (FRE 402), or
b) the court makes a discretionary determination that the probative value of the evidence
is substantially outweighed by pragmatic consideration (FRE 403):
全ての関連証拠は証拠能力があるのが原則だが、(i) 証拠排除法則が適用される場合、及び、
(ii) 比較考量(balancing test)により Pragmatic consideration が証拠価値よりも実質的に優越する
と裁判所が裁量で判断した場合は、証拠能力が認められない。
 (ii)の要件は全ての証拠で問題になり得る点に留意((ii)の要件を満たす限り裁判所はどの
ような証拠であっても証拠排除可能)。

Six types of pragmatic considerations: Remember three:


1) danger of unfair prejudice Unfair prejudice 不公正な予断
不公正な予断の危険
2) confusion of the issues 問題の混乱
Confusion 混乱
3) misleading the jury 陪審員の誤導
4) undue delay 不合理な遅滞
5) waste of time 時間の浪費 Waste of time 時間の浪費
6) unduly cumulative 不合理な重複
BRQ-24 争点と直接関係ない供述が行われた場合、供述に反対する者は Extrinsic Evidence 又は Prior Inconsistent
Statement で反論することは認められない。

Exam Tip: Judges have wide discretion to balance probative value with pragmatic
considerations and the determination will be fact specific. As a result, it is difficult to
formulate MBE questions on this issue.
これらの要素は balancing factor であり、ケースバイケースで決まるものである(裁判官が証拠価値
の決定に広範な裁量を有している)ため、試験では問題になりにくい。
One possible question: Which of the following is the least likely reason for the court to rule the
evidence inadmissible?
(A) Unfair prejudice
(B) Undue delay
(C) Unduly cumulative[不合理な重複]
(D) Unfair surprise.
(D) is not a reason to exclude evidence.
PR-55 裁判官は自分の担当する法廷で証人となることはできない。休廷中に無罪主張の被告人と弁
護人が実は有罪であると話していたのを聞いたとしても、それ自体を証言することはできない。
PR-83 被告が州の刑事免責(state grant of immunity)を付与された状態で連邦法上の訴追に関連す
る内容の供述をした場合、連邦側はこの供述に基づかずに、独立の正当な取得源から取得された証
拠による訴追であることを証明しなくてはならない。Murphy v. Waterfront Commission
PR-160 当事者が特定の証人を呼ばなかった場合(又は当事者自身が証人台に立たなかった場

5
Evidence 2016 Summer BarBri NY Bar

合)、相手方当事者がこの事実を有利な推定として主張することは認められない。
PR-161 最高裁判所によれば、検察官による被告の黙秘についてのコメント、または裁判所による
「沈黙は有罪の証拠である」という陪審に対する指示は、被告人の修正第 5 条に基づく Self-
Incrimination の保障(州に対しては修正 14 条を通じて適用)に対する違反に該当する。Griffin v.
California
BarBri Set 2-12, 5-5 ポリグラフ検査
Relevance は肯定し得るものの、ポリグラフ検査の probative value は unfair prejudice(jury を混乱ま
たはミスリードする)のリスクに比して低いと考えられるため、裁判所の裁量によって証拠排除さ
れることが通常。

Note on terminology:
(1) If an objection is “sustained” the evidence is inadmissible 異議が認められる⇒証拠能力なし
(2) If an objection is “overruled” the evidence is admissible 異議が認められない⇒証拠能力あり
B. SIMILAR OCCURRENCES 類似事実の発生
1. Overview
a) The General Rule of Relevance:
In general, if evidence concerns some time, event, or person other than that involved
in the case at hand, the evidence is inadmissible. Probative value is usually
outweighed by pragmatic considerations (e.g., weak relevance, danger of confusion,
misleading the jury, time-consuming).
関連性があるといえるためには、訴訟に関連する「時間」、「出来事」、「人」に関する証拠
でなければならない。そうでなければ、証拠能力がない。
b) Exception:
In some limited and specific circumstances, other similar occurrences may be
admissible, even if they relate to a time, event, or person other than that involved in
the present litigation.
訴訟に関連する事項以外の「時間」、「出来事」、「人」に関するものであっても、限定的に
その他の類似事実も証拠能力が認められる場合がある。
c) Six situations to know … 例外は 6 類型
2. Plaintiff’s accident history. 原告の事故歴
a) THE RULE: Generally, plaintiff’s accident history is inadmissible because it shows
nothing more than the fact that the plaintiff is accident-prone.
原則:原告の事故歴は、事故の傾向を示すものにすぎず、証拠能力なし
Being accident-prone is nothing more than character evidence which is not allowed in a civil action to
prove a person’s conduct on a particular occasion.

b) THE EXCEPTION: Plaintiff’s prior accidents may be admissible if the event caused
plaintiff’s injury is in issue. 例外:今回問題となっている事故の原因を作った過去の事故
HYPO 1. Phil drove into a lamp post and sues the municipality in negligence, alleging that the
placement of the post created a hazardous condition.
(a). Should the municipality be allowed to introduce evidence that Phil has filed numerous actions
against property owners based on his driving into other stationary objects (tree, bridge, brick wall) and
lost every time?
→No. Phil’s lawsuits have nothing to do with this lawsuit. Phil’s careless driver propensity is not a
proper purpose. 証拠能力なし。不注意な運転手であるとの性向は目的として不適切。
(c). What if the municipality claims that Phil’s injuries (e.g., his chronic back pain) resulted from the
prior accidents, not the current one?
→Admissible. Causation purpose. This is to prove that the prior accident was the cause of the Phil’s
injury. 証拠能力あり。因果関係の立証目的なので OK

2. Similar Accidents Caused by Same Event or Condition.


同じ事実・条件によって生じた類似の事故
a) THE RULE: Other similar accidents are generally not admissible, because they
suggest nothing more than general character for carelessness. But …
原則:他の類似の事故は証拠能力なし。

6
Evidence 2016 Summer BarBri NY Bar

b) THE EXCEPTION: other accidents involving the same instrumentality or condition,


and occurring may be admitted for 3 potential purposes, if the other accident occurred
substantially similar circumstances: 例外:以下の 3 つの目的の場合には、同じ手段又は条
件を含む他の事故又は実質的に同様の状態で生じた他の事故は、証拠能力がある。
(1) To show the existence of a dangerous condition 危険な条件の存在
(2) To show the causation of the accident 因果関係
(3) Prior notice to the defendant 被告に対する事前の認識
HYPO 2 Assume in Hypo 1 that several other vehicles had collided with the same lamp-post that Billy
ran into. Could Billy introduce those other accidents against the municipality?
→Yes. if those other accidents happened substantially similar under circumstances (e.g., weather,
lighting, traffic condition), and the purpose is one of the purposes above.

c) Related rule for Experiments and Tests: The standard for admitting experiments
and tests is the same. There must be substantial similarity between experiment and
the disputed fact. 実験及びテストに関する関連規則: 実験及びテストの証拠能力も同様。実質
的な事実の類似性が必要。
HYPO 3. Xerox のコピー機に問題があるという主張をする場合には、同じ機種を同じ条件で
事件・テストしなくてはならない。
PR-149 Absence of Similar Accident 同種の事故の不存在
同種の事故の不存在について、偏見が証拠価値を上回る場合は証拠能力が認められない。
たとえば、交通量の多い交差点で 14 年間事故がなく安全であったという事実は、危険な状態が存在
しなかったことについての証拠として証拠能力が認められる。(ただし一般に裁判所は同種事故不存
在の証拠採用に消極的)

3. Intent in Issue 故意
THE RULE: Prior similar conduct of a person may be admissible to raise an inference of
person’s intent on a later occasion (similar to MIMIC).
例外:以前の類似の事例は、以前の行動からの故意の推定について関連性を有する。
HYPO 3. Paris sues Brewski Co. for racial discrimination, alleging that she was qualified for the job
but was not hired because of her race. She seeks to show that Brewski hired no minority job applicants,
despite their qualifications, during the past six years. Admissible?
- Yes. This is to show discriminatory intent of the defendant.

4. Comparable Sales on Issue of Value 売買価格の比較(製品の価値の証拠)


THE RULE: The selling price of other property of a similar type, in the same general
location, and close in time to the period at issue, is admissible as evidence of value of the
property at issue.
同種、同じ地域、同時期の製品の販売価格は、製品の価値の証拠として証拠能力あり。

7
Evidence 2016 Summer BarBri NY Bar

5. Habit 習慣 (FRE 406)


a) Habit of a person (or routine of a business organization) is ADMISSIBLE as
circumstantial evidence of how the person (or business) acted on the occasion at issue
in the litigation. その状況下でどのように行動したかを推認するために 人の習慣を証拠とする
ことができる。

Sample Question 2 :列車との衝突事故で鉄道会社が警笛を鳴らしたかが論点になっている訴訟に


おいて、鉄道会社が線路近隣住民(当該事故の時にはいなかった人)に普段鉄道が通る時には警
笛の音がするという証言をさせることは許される。
b) DISTINGUISH: Character evidence refers to a person’s general disposition or
propensity[傾向・性向]. Character is usually not admissible to prove conduct on a
particular occasion. E.g., Fact that Carlos is a “careless” driver is inadmissible to
suggest that he ran a red light and caused the accident involving the plaintiff.
証拠能力がない Character Evidence と証拠能力がある habit の区別が重要。
c) DEFINITION: Habit is a repetitive response to a particular set of circumstances.
Thus, habit has two distinguishing characteristics:
習慣=特定の状況に対する反復的な反応をいう。下記 2 つの特徴を有する。
(1) Frequency 頻繁
(2) Particularity 特定性

* ①“Once a careless driver, always a careless driver” is not admissible as propensity


evidence. 「かつて不注意の運転手は、常に不注意である。」証拠能力なし
* ②But, “1,000 times seatbelt unwearer would be a seatbelt unwearer” is admissible as
habit evidence.「1000 回シートベルトをつけていなかった場合、いつもシートベルトをつけて
いない者である。」証拠能力あり
Hypo 4. In an auto accident case, the issue is whether Joe Isuzu stopped his car at the stop sign at the
intersection of Hickory and Main Streets.
Hypo 33(a). Plaintiff calls Wanda to testify that during the six months preceding the accident, she had
seen Joe run red lights, change lanes without using signals and run stop signs throughout town.
Admissible as habit evidence to prove that Joe ran the stop sign at Hickory and Main?
- No. Those prior accidents merely tend to prove a careless driver. No particularity here.
特定性がないので×
Hypo 33(b). Wanda will testify that she has seen Joe run the stop sign at Hickory and Main on at least 8
occasions with in 2-week period. Admissible as habit?
- Yes. Here, we have frequent and particular responses enough to show habit. 8 occasions may be
enough. No magic number for frequency. It depends on the fact.
何回だと Frequency といえるかどうかは不明確だが、このケースでは十分

d) Exam Tip: On a multiple-choice question, look for key words indicating frequency:
“always”, “invariably[不変]”, “automatically”, “instinctively”, “habitually.”
これらのキーワードが入っていると、Frequency を認めやすい。⇒証拠能力あり
e.g. 彼は車の手入れに熱心⇒character / 彼は毎週日曜朝に車のブレーキ点検をする⇒habit

e) Business Routine: The regular practice of an organization is admissible to prove


conduct on a particular occasion.
会社の通常の慣行は特定の状況下での行為の証明として証拠能力あり。
Example: To prove that a particular letter was mailed by CEO, evidence that CEO put
letter in her out-box on Tuesday, and messenger “routinely” picks up mail in CEO’s
out-box at 3:00 P.M. each business day for delivery to mail room.

8
Evidence 2016 Summer BarBri NY Bar

6. Industrial Custom as Standard of Care 注意義務の基準としての業界慣行


THE RULE: Evidence as to how others in the same trade or industry have acted in the
recent past may be admitted as some evidence as to how a party in the instant litigation
should have acted, i.e., as evidence of the appropriate standard of care.
業界慣行も、注意義務の基準としての証拠能力あり。
Example: Plaintiff is injured when a blade spins off a lawn-mower. In an action against the
manufacturer, she may show that 80% of all other lawn-mower manufacturers, during the
relevant time period, had installed devices to prevent blade spin-off.

C. POLICY-BASED EXCLUSIONS[政策的証拠排除]
General Rationales:
a) Balancing of relevance and pragmatic considerations, 関連性と実際的検討の バランス
and
b) Encouraging beneficial out-of-court conduct.裁判所外の有益な活動の促進
1. Liability Insurance (FRE 411) 第三者に対する賠償責任保険への加入の事実
a) RULE: Evidence that a person has (or does not have) liability insurance is
inadmissible for the purpose of proving negligence or ability to pay.
責任保険を有している(又は有していない)という証拠は 過失又は支払能力を証明する目的で
は証拠として認められない。
* 関連性があるかといえば関連性はあり得る(保険に入っていた=損害賠償が必要となる過失
事故を予見していた、と主張され得る。) c.f., Party Admission (VI.E.)

b) EXCEPTION: But such evidence may be admissible for some other relevant
purpose, such as: 以下のような関連する目的で行われる場合には証拠能力あり。
(1) proof of ownership or control, if that issue is disputed (by the other party), or
所有権又は支配権が論点となっている場合、それらの権利の証明に用いるとき
(2) impeachment of a witness. 証人の弾劾
(Impeachment is a process of trying to show that witness should not be believed.)
KEY CONCEPT: Bias means there is some relationship between the witness
and a party that could cause the witness to lie. Bias が重要。証人と当事者の関係によ
り証人が嘘をつきやすくなる。
HYPO 5. Rosie fell down a well[井戸] on Trump’s property, Rosie sues Trump, contending that the
well was impossible to see because of overgrown foliage. Trump denied that he was negligent and also
defends, in the alternative, on the ground that he did not own the land in question.
(a) should Rosie be allowed to introduce evidence that Trump carried a homeowner’s liability insurance
policy on the property?
Mixed answer: No, if the purpose is to show negligence. But Yes, if the purpose is to show the
ownership of the accident.
(b) Same case as Hypo 1. Apprentice, a witness called by Trump, testifies that she had been on
Trump’s property just prior to the accident and there was no foliage covering the well. May Rosie
show, during cross-examination of Apprentice, that Apprentice is a claims adjuster employed by
the company that issued the homeowner’s policy to Trump?
- Yes. Purpose is to impeach the witness by showing a bias, it’s another proper purpose.
Apprentice might be biased because she is an employee of the liability insurance company and may
lie for the insurance company.
Bias の証明により証人の信用性を弾劾するのが目的なので、証拠能力あり。
BarBri Set 3-13 賠償責任保険と損害保険
上記はあくまで第三者に対する損害賠償責任に関する保険( liability insurance)について
の話であり、自己の財産に関する損害保険(casualty insurance。火災保険や地震保険)に
ついては証拠排除されない。

9
Evidence 2016 Summer BarBri NY Bar

2. Subsequent Remedial Measures (SRMs) 事後の救済対応があった事実 (FRE 407)


a) Definition: Repairs 修繕 , design changes デザイン変更 , or policy changes taken after
an accident that could have prevented the accident. 事故が発生した後の方針転換
c.f., Party Admission (VI.E.)
b) THE FEDERAL RULE: SRMs are inadmissible for the purpose of proving:
以下の事項を証明する目的では SRM は証拠能力なし。
(1) Negligence 過失
(2) Culpable conduct 犯罪行為
(3) Product defect 製品の欠陥
(4) Need for warning 警告の必要性
Rationale: to encourage post-accident measure 事故の後の救済を奨励するため
c) THE EXCEPTION: But such evidence may be admissible for some other relevant
purpose, such as proof of: しかし以下の場合は証拠能力あり。
(1) Ownership 所有権 or
(2) Control 支配 or
(3) Feasibility of a safer condition 安全な状態の実現可能性,
if that issue is disputed (by the defendant). 争点となっている場合
Product Liability: (i) manufacturing defect, (ii) design defect and (iii) failure to warn.
HYPO 6. Penelope bought a cup of coffee at Dante’s Coffee Inferno and scalded her tongue because
the coffee was too hot. She sues Dante’s in negligence, and Dante’s denies any negligence.
(a) At trial, Penelope seeks to introduce evidence that after the accident, Dante’s installed new
thermostats on its coffee-brewing equipment. Penelope contends that this conduct is an admission
by Dante’s that better safety controls were feasible. Admissible?
- No. Basic rule exclusion applies. It’s a subsequent remedial measure. Not admissible even for
feasibility of a safer condition because Dante is not disputing the feasibility. Dante is disputing its
negligence (i.e., Dante only states that the conduct was Reasonable).
(b) Same cases, except now assume that Penelope contends that Dante’s negligence consisted of the
failure to place warnings on its coffee cups indicating that its coffee was too hot for immediate
human consumption. Dante defends, in part, on the ground that it was impossible to affix labels to
its coffee cups. Penelope seeks to introduce evidence that after the accident, Dante’s began to use
cups that were pre-printed with warnings. Admissible?
- Yes. Now Dante is contesting the feasibility of the safety design. Feasibility becomes an
acceptable purpose once the opposite party contends it.

Note: In a products liability action based on strict liability, the manufacturer’s


subsequent remedial measures are inadmissible to show the existence of a defect in
the product at the time of the accident. The federal (multistate) rule differs from the
rule in some states.
3. Settlements 和解に関する事実の呈示 (FRE 408)
a) THE RULE:
(1) If there is a disputed civil claim,
(2) the following are INADMISSIBLE
(a) settlements 和解 or
(b) offer to settle 和解の申込み or
(c) statements made during settlement discussions 和解交渉中の言動
(3) for the purpose of to show a liability or to impeach a witness as a prior
inconsistent statement.

* Rationale: To encourage settlements and frank settlement discussions.


和解および和解交渉での率直な発言を奨励するため

10
Evidence 2016 Summer BarBri NY Bar

b) The requirement of a disputed claim: The ban on settlement evidence only applies
if, at the time of the discussion, there is a claim (a demand of some sort) and that
claim is disputed by the other side (either as to validity or amount). 和解に関する証拠能
力の制限は、議論の時点で請求が存在し、紛争となっている場合である。
Note #1: If the other party makes an offer to settle immediately after the accident,
such offer is likely to be admissible to prove validity or amount of a claim.
Note #2: If the other side admits the liability and the amount against a claim, but
offers to pay less for settlement, such offer is admissible to prove validity or amount
of the claim.(because of no dispute)
Note #3: However, if the other side admits liability against a claim and offers to pay
less, but does NOT make any admission as to the amount of liability, the offer is
inadmissible to prove liability.
c) THE EXCEPTION:
(1) Settlement evidence may be admissible if offered to impeach a witness on the
ground of bias. Bias の恐れのある証人を弾劾するために申請する場合は証拠能力あり。

(2) Statements of fact made during settlement discussion in civil litigation with a
government regulatory agency those are admissible in a later criminal case (e.g.,
corporate fraud case in which corporate officers make admissions of fact during
civil settlement talks with the SEC and are later prosecuted for crimes based on
the same facts).
Rationale: public policy favors prosecutor’s use of highly probative factual evidence.
(エンロン事件を踏まえて追加されたルール)

Note: The exception in criminal cases only applies to statement of fact and does NOT
apply to settlements and offers to settle. [事実に関する供述のみ]
HYPO 7. Huck and Tom were simultaneously struck by a truck being driven by Joe. Huck and Tom
both filed suit against Joe, each seeking $100,000, and Joe denied the allegations.
(a). Before trial, Huck and Joe settled Huck’ claim for $50,000. When Tom’s case went to trial, Tom
sought to introduce the Huck-Joe settlement as evidence that Joe, in effect, had acknowledged his
fault. Admissible?
- No. Tom’s offer of evidence as an evidence to prove Joe’s fault.
(b) Before Tom’s case went to trial, Tom and Joe met to discuss possible settlement. During the
discussion, Tom said, “Look, I’ll accept $50,000 in settlement. The fact that I was jay-walking [信
号無視] may confuse the jury.” Joe declined. At trial, should Joe be allowed to introduce:
(1) Tom’s offer to settle?
- No. Same rules apply to offer of settlement.
(2) Tom’s admission that he was jay-walking?
- No because it occurred in the course of the settlement and the rules against settlement also
applies.
(c) At the trial of Tom’s case, Joe called Huck as a witness and Huck testified that Joe did not drive
negligently. On cross-examination of Huck, should Tom be allowed to prove the Huck-Joe
settlement?
- Yes. Difference is the purpose for which the evidence is offered. This is to prove Huck’s bias and
not Joe’s negligence. Huck may be bought out by Joe.
HYPO 8.
(a) A’s and B’s cars collided. B immediately ran up to A and said, “Look, I’ll settle with you for
$100,000 if you don’t sue.” Should A be allowed to introduce B’s statement against him at a
subsequent trial?
- Yes. The rule for exclusion of settlement is not applicable if there is no dispute over claims.
Claims need not to be lawsuits. Here, A and B are not disputing over the claim of A.
(b) After A’s and B’s cars collided, A sent a letter to B saying, “The accident was all your fault. I
demand that you pay my damages in the amount of $100,000.” B called A on the phone and said,
“You’re right about the accident. It was all my fault and I owe you the full $100,000 you’re asking
for. But you know how fickle juries can be. If you don’t accept $50,000 now, you’ll have to sue me
to get anything.” Should A be allowed to introduce B’s statements against B at a subsequent trial?
- Yes. B is not disputing over the claim of A.

11
Evidence 2016 Summer BarBri NY Bar

Variation on 8(b): What if B had said, “It was all my fault, but you didn’t suffer $100,000 in damages”?
-No. Parties have a dispute over the amount of damages.
HYPO 9 . Arnold sold toxic “Terminator” action figures to the public. The Environmental Protection
Agency (EPA) sued Arnold for civil penalties. Arnold denied liability. In negotiations with the EPA,
Arnold offered to settle for half the amount sought, admitting during the discussions that the toys were
toxic. Thereafter, the Government prosecuted Arnold for violating criminal laws against the distribution
of toxic toys. In the criminal case, as evidence of Arnold’s guilt, should the Government be allowed to
introduce:
(A) Arnold’s offer to settle with the EPA?
- No. Enron exception does not apply to settlement itself.
(B) Arnold’s admission to the EPA that the toys were toxic?
- Yes. Enron exception does apply to statement of fact.

4. Pleas and Plea Discussions in Criminal Cases 答弁及び司法取引 (FRE 410)


a) THE RULE: The following are inadmissible
係属中の刑事事件及び後の民事事件での被告に対して、以下の事項は証拠能力がない。
(1) an offer to plead guilty 有罪答弁の申入れ
(against the defendant in a pending criminal case or in a subsequent civil case)
(2) a withdrawn guilty plea 撤回された有罪答弁
(against the defendant in a pending criminal case or in a subsequent civil case)
(3) a plea of nolo contender (no contest) 不抗争の抗弁
(against the defendant in a subsequent civil case)
(4) statements of facts in connection with any of the above (1)-(3)に関連する事実の供述
b) Beware: A plea of guilty (not withdrawn) is admissible against the defendant in
subsequent litigation based on the same facts. 撤回されていない有罪答弁は、後の同一事
実に基づく訴訟における被告について証拠能力を有する。(Hearsay の問題にも注意。)

PR-79 司法取引の段階でなされた有罪答弁だけで、判決も出されていないような時点で、
他の事件の証拠として使用することはできない。
BarBri Set 4-3 有罪答弁の結果として有罪判決になった場合、当該判決の証拠能力は認めら
れる。

5. Offer to Pay Hospital or Medical Expenses 医療費支払の申し出 (FRE 409)


a) THE RULE: Evidence that a party has paid or offered to pay an accident victim’s
hospital or medical expenses is always INADMISSIBLE to prove liability.
医療費支払いの申し出は責任の証明についての証拠能力はない。
b) Rationale: To encourage charity. 慈善活動を奨励するため
Note: This rule does not exclude other statements made in connection with an offer to
pay hospital or medical expense.
HYPO 10. Donna’s car hit pedestrian Pabro. Donna immediately ran to Pabro and said, (a) “Don’t
worry about a thing. I’ll pay for your hospital bills. (b) I’m sorry I ran the red light.” Pabro
subsequently sues Donna.
(a). Is statement (a) admissible against Donna?
- No. Offer to pay hospital expenses and always inadmissible.
(b). Is statement (b) admissible against Donna?
- Yes. It’s admissible. The rule against offer to pay medical expenses only applies to the offer and not
to statements concerning facts made in conjunction with the offer.
医療費支払の申し出とともに事実についての供述がなされている場合、申し出部分について
証拠能力はないが、事実部分については証拠能力あり。この時点では当事者間に紛争はない
ので、「和解交渉中の発言」にも当たらない。⇒紛争がある場合には、「和解交渉中の発
言」となり、事実に関する過失についての発言も証拠能力がないことになる。
6. REVIEW AND SUMMARY: Policy Based Exclusions
Evidence Inadmissible Purpose Admissible Purpose

12
Evidence 2016 Summer BarBri NY Bar

1 Liability Insurance Negligence Ownership or control if disputed


Ability to pay Impeachment
2 Subsequent Remedial Negligence, Ownership, control, feasibility
Measures culpable conduct, or destruction of evidence, if
product defect, or controverted
need for warning Impeachment
3 Settlement Discussions Validity or amount of For all other purposes (including
disputed claim Bias)
4 Offers to Pay Medical Liability For all other purposes
Expenses
5 Withdrawn Pleas Guilt, (FRE) civil liability Not admissible

D. CHARACTER EVIDENCE 性格証拠


1. Introduction
a) Definition: Character evidence refers to a person’s general propensity or disposition,
e.g., the character traits of honesty (or dishonesty), peacefulness (or violence),
carefulness (or carelessness)
(1) Consider (i) purpose, (ii) criminal or civil, and (iii) form.
(i)目的、(ii)刑事か民事か、(iii)使用形態に着目

b) Potential purposes for the admissibility of character evidence:


(1) Person’s character is an essential element in the case.
(this is rare in civil, and never in criminal case)
(2) Character evidence as circumstantial evidence of the person’s conduct on a
particular occasion.
e.g., robbery case and peacefulness of defendant
(3) Witness’s bad character for truthfulness to impeach credibility.
(c.f., IV.H)
c) The General Rule: (FRE 404, 405)
(1) Character evidence is not admissible to prove propensity (“conduct in
conformity”) (purpose #1). 性向に関する性格証拠は証拠能力なし。
(2) Character evidence is admissible for other purposes (purposes #2, #3, and #4)
その他の場合には性格証拠も証拠能力あり。
d) Purpose Matters. In approaching a character evidence problem, you must first
identify the purpose for which the evidence is offered. 最初に目的を特定
(1) Propensity 性格・性向
If evidence of a person’s character trait is offered to prove that the person has a
“propensity” to act in a certain way – in other words, as evidence that the person
acted in conformity with the character trait on the particular occasion in question.
Or “conduct in conformity.”
性格証拠の目的が当該人物のある種の性向を立証することである場合⇒証拠能力なし
(2) Veracity of a witness 証人の真実性(弾劾目的):
If evidence of a witness’s character for truthfulness is offered to impeach the
witness. ⇒弾劾目的=証拠能力あり

13
Evidence 2016 Summer BarBri NY Bar

(3) Non-propensity purpose 性向目的でない場合:


If evidence of person’s prior bad act is offered for some purpose other than
proving propensity (“conduct in conformity”).
性向以外の目的で以前の悪行が証拠として提出される場合⇒証拠能力あり
(4) Trait as element 要素としての特徴:
If evidence of a person’s character trait is offered because the trait is an essential
element of the claim or defense.
性格的特徴が請求又は抗弁の本質的要素である場合⇒証拠能力あり

2. Criminal Cases (FRE 404(a)) 刑事事件


a) Defendant’s Character 被告人の性格証拠
(5) The General Rule: Evidence of the defendant’s character to prove defendant’s
conduct on a particular occasion is not admissible during the prosecution’s case-
in-chief.
原則:被告人の性向立証のための性格証拠は証拠能力なし。
Rationale: Fear that the jury will misuse the evidence and convict the defendant
not because he’s guilty, but because he’s a bad person.
陪審が、犯罪を実行したから有罪となるのではなくて、被告人が悪人なので有罪であるな
どと判断することにより、証拠を誤った方向に使用するおそれがある。
(6) The Exceptions:
(a) The defendant may introduce evidence of his own good character for a
relevant trait (by reputation or opinion testimony of a character witness) to
prove his conduct. 被告人による関連する性向についての被告人の良性格立証は
OK
Emanuel -58 認められるのは問題となっている犯罪に関連する特性についてであ
り、被告人の truthfulness は被告人側からの立証もできない。

(b) However, if the defendant does so, the prosecution may rebut with evidence
of the defendant’s bad character for the same trait. (defendant opens the
door to rebuttal) 被告人が良性格の立証をする場合には、検察側も同じ特徴につ
いての悪性格証拠を用いて反論可能。
Rationale: The defendant can evaluate the risk of unfair prejudice, and if
the defendant is willing to open the door to character evidence, we will give
him that option.
(7) The form of character evidence 性 格 証 拠 の 形 式 : When character evidence is
admissible to prove propensity, the only proper methods are:
(a) Reputation or opinion. 評判/意見は OK←Extrinsic Evidence で立証
(b) No specific act is admissible to prove the character’s propensity. It’s hard to
prove such a specific act. 特定の行為による性格立証は証拠能力なし。
(why? Proof of specific act is time consuming.)
* 関連性(relevance)がない内容についても証拠能力なし。Extrinsic Evidence でもよい。

BSM-140 被告人側が良性格の立証のために、被告人のよい評判を知る証人に証言をさ
せたとしても、その良性格が犯罪に関連する性格との関連性を欠く場合には、良性格立
証であっても、証拠能力は認められない。たとえば、 Assault で起訴されている被告人
が、暴力的でないことについての評判を知る証人に証言してもらうのは証拠能力が認め
られるが、Truthfulness や Honesty について証言させる場合には、証拠能力は認められな
い(Relevancy 関連性の問題)。
Emanuel-4 Fraud で 起訴 されている被告人が、良性格証拠として 会社 の上司に ”on
several occasions the corporation had entrusted him with large sums of money”を証言させるこ
とはできない。Specific instances だから。
HYPO 12. Rambo is charged with murder.
(a) During its direct case, should the prosecution be allowed to introduce evidence that Rambo has

14
Evidence 2016 Summer BarBri NY Bar

been convicted three times for assault, has a bad reputation for violence, and he recently stampeded
a herd of cattle through the middle of town?
- No. It might be relevant but the prosecution is not allowed to introduce character evidence in its
case-in-chief.
(b) Should the prosecution’s proposed evidence be admitted on the ground that defendant’s violent
character is an essential element of the crime with which Rambo is charged?
- No. It’s not an element of murder. No crime in bar exam has a trait of character as its element.
被告人の性格・性向が犯罪の構成要件になっている場合は、検察側は立証を要するが、Bar
に出てくる問題でそのような犯罪は出てこない。⇒性格立証は証拠能力なし
HYPO 13.Same murder prosecution. During the defense case, Rambo calls Trautman to the stand. May
Trautman testify to the following:
(a). “I’m familiar with Rambo’s reputation for peacefulness, and it is excellent.”
- Yes. Defendant can offer evidence for its good character. Reputation evidence can be used both
under the federal rules and in NY.
(b) “I personally know Rambo, and in my opinion he is a peaceful person.”
- O.k. Federal rules allow opinion evidence.
(c) “I’ve seen Rambo turn the other cheek when assaulted by bullies; he’s the president of the local
Pacifist Club.”
- No. It’s relevant but in a wrong form. This is a specific act and cannot be used as character
evidence.
(d) “Rambo’s reputation for bravery and honesty is excellent.”
- No. Right form but a wrong character form. This is not relevant trait. Violence or peacefulness
may be relevant but not bravery or honesty.関連性なし。

e) Prosecution’s Rebuttal 検察官提出の被告人の性格証拠


If defendant has “opened the door” by calling character witnesses, the prosecution
may rebut in two ways: 被告人が性格証拠を提出したら、検察官は 2 つの方法で反論可能。
(1) (Intrinsic Evidence) By cross-examining defendant’s character witnesses by
questioning their knowledge of specific acts by the defendant that are relevant to
the character trait at issue. 被告人が提示する性格証人に対する被告人の特定の行為に
ついての認識を反対尋問で質問。議論となっている性格的要素との関連性必要。反対尋問
であれば、評判・意見のみならず特定の行為についても質問可能。
(a) Form:
(i) For opinion witnesses: “Did you know ~~~?”
意見「知っているか?」
(ii) For reputation witnesses: “Have you heard ~~~?”
評判「聞いたことあるか?」
(b) Purpose: To test the witness’s knowledge of defendant’s character, Not to
prove the specific act. 特定の行為を立証するためではなく、被告人の性格につい
て証人にどれだけの認識があるかをテスト
(c) Good faith requirement: Even though the prosecution is not proving the
specific act, it must have a good faith basis to believe that the specific act
took place. 誠実性の原則
(2) (Extrinsic Evidence) By calling its own reputation or opinion witnesses to
contradict defendant’s witness. 特定の行為は×

Note: 検察官による rebuttal が認められるのはあくまで被告人側証人によって言及された点の


みであり、この範囲は限定的に解釈される。例えば被告人側証人が被告人は遵法的である(law-
abiding)と述べたとしても、問題となっている種類の事件に関し遵法的かどうかのみが争点であ
り、関連性のない種類の事件(例えば本訴が殺人の場合における、脱税)についての遵法性を
反証することは認められない
HYPO 13. In the Rambo murder trial, Rambo called Trautman to testify to Rambo’s good character for
peacefulness
(a) Could the prosecutor ask Trautman, on cross-examination, “Have you heard that Rambo was
arrested last year for assaulting Rocky?”
- Yes. The prosecution is allowed to test the witness’s knowledge. If the witness does not know
about it, it may suggest that the witness does not know Rambo well either. This is not to prove that
Rambo was arrested or that he assaulted Rocky. Rambo が逮捕されたことや Rocky に暴行を加

15
Evidence 2016 Summer BarBri NY Bar

えたという事実でなく、証人の知識(証人が Rambo のことをよく知らないこと)の立証。


(b) Could the prosecutor ask Trautman, on cross-examination: “Did you know that Rambo shot Judge
Dredd three years ago?”
- Yes. Just like 13(a). The purpose is to test the witness’s knowledge.
(c) If Trautman denies having heard or knowing of the bad acts mentioned by the prosecutor, may the
prosecutor prove that the acts actually occurred?
- No. The only purpose is to test the witness. The prosecutor cannot go ahead to prove the fact.
(you can ask (make query) but cannot prove!!)
(d) Could the prosecutor properly ask Trautman, “Have you heard that Rambo cheated on his income
taxes last year?”
- No. Wrong character trait (irrelevant). This question is related to Rambo’s honesty but the
specific trait asserted by the defendant is his peacefulness.
HYPO 14. In the Rambo murder trial, assume Rambo introduced Trautman’s testimony that Rambo
has a good reputation for peacefulness. After the defense rests, the prosecution calls Murdock to testify
that he has known Rambo for twenty years, is familiar with Rambo’s reputation for peacefulness in the
community, and that such reputation is bad. Rambo’s attorney objects on the ground that this is
impermissible character evidence. Objection is sustained?
- No (overruled). Because the defendant opened the door.
HYPO 15. In the Rambo murder trial, assume that the only witness who testified during the defense
was Rambo himself, and his only testimony was that he did not commit the murder. After the defense
rests, the prosecution call Murdock to testify that Rambo has a reputation for violence. Rambo’s
attorney objects on the ground that this is impermissible character evidence. Objection is sustained?
- Yes (sustained). P cannot introduce an evidence for character because D did not open the door merely
by testimony of innocence. 被告人自身が無罪の証言をしている場合には(良性格立証をして
いるわけではない)、検察官は性格証拠を提出することはできない。

Note: Bar exam では defendant が door を open するのは witness を呼ぶ場合だけ(自分で良性格立


証を行うという方法は質問されない)。
f) Victim’s Character in a Self-Defense Case 正当防衛の際の被害者の性格
(1) General Rule: A criminal defendant may offer evidence of the victim’s violent
character to prove that the Victim was the first aggressor.
刑事事件において、被告人は、被害者が最初の攻撃者であることの立証のために被害者の
暴力的な性格についての証拠を提出できる。
(a) Form: reputation or opinion only. 評判/意見のみ。特定の行為の立証は×
(b) Prosecution rebuttal 検 察 官 の 反 論 : The prosecution may rebut in two
different ways, by evidence of: 以下の方法で検察官は反論可能。
(i) the victim’s good character of that trait (i.e., V is a peaceful person) or
被害者の当該要素についての良性格
(ii) the defendant’s bad character of that trait. (i.e., D is a violent person)
被告人の当該要素についての悪性格
HYPO 18. Defendant Bobby has been charged with assault for throwing a chair at Tonya. Bobby
claims that Tonya started the fight and lunged at him with a knife. To prove that Tonya was the
first aggressor, may Bobby call Nancy to testify:
(a) That she knows Tonya and that in her opinion, Tonya is a very violent woman.
Admissible. D is allowed to offer evidence concerning the victim’s bad character to prove that
Tonya was the first aggressor.
(b) That she (Nancy) had been the victim of a knife attack by Tonya a few years ago.
Inadmissible. This is a specific act and not admissible form of evidence.特定の行為なのでだめ
(c) What if Bobby offers evidence that, at the time of the altercation with Tonya, he was aware
of her prior knife attack on Nancy?
Admissible. Bobby can offer this evidence to prove his own state of mind, i.e. his reasonable
belief that Tonya was attacking him. The purpose is not to prove the victim’s bad character. 被害
者の悪性格を立証するのでなく、被告人自身の精神状態(被害者の暴力的な性格又は以前
の特定の暴力的行為について知っていたため、被害者が攻撃していると合理的に信じたこ
と)の立証であればよい。
Separate rule of relevance: If the defendant, at the time of the alleged self-defense, was aware of
the victim’s violent reputation or prior specific acts of violence, such awareness may be proven to
show the defendant’s state of mind— fear—to help prove that he acted reasonably in responding as
he did to the victim’s aggression.

16
Evidence 2016 Summer BarBri NY Bar

(2) Special Rule for Defendant’s Knowledge of Victim’s Character for Violence.
被告人の被害者の暴力に関する性格の知識についての特則 The defendant may offer
evidence of his own knowledge of the victim’s bad character for violence for the
purpose of showing that he reasonably believed the need of self-defense. 正当防衛
の必要性を合理的に信じたことを立証目的として被害者の悪性格を知っていることを証拠
として提出できる。
(a) Rationale: This is not propensity evidence because the defendant is not
using the evidence as proof that the victim started the fight. Rather, the
evidence is relevant simply to show that the defendant reasonably believed
that the victim would start a fight.
(b) Form: Because this is not propensity evidence, any form is allowed
(reputation, opinion or specific acts). 性向に関する証拠でないので、いかなる方
法でもよい。評判、意見、特定の行為、どれでもよい。
g) Victim’s Character in a Sexual Misconduct Case (FRE 412) 性犯罪の被害者の性格
(1) The (“Rape Shield”) Rule:
In a case involving alleged sexual misconduct (civil or criminal), the defendant
ordinarily may not introduce evidence of
民事でも刑事でも性犯罪の場合には、被告人は以下の証拠を提出できない。
(a) The victim’s reputation for promiscuity 被害者の評判 or
(b) The victim’s specific sexual behavior. 被害者の性行為
(2) Two rationales:
(a) “Promiscuity evidence” has low probative value, 乱交証拠は証拠価値が低い。
(b) And may discourage rape victims from coming forward.
被害者による犯罪告発を萎縮
(3) THE EXCEPTIONS: Notwithstanding the general rule, a defendant may
introduce:被告人は以下の証拠を提出可能
(a) Evidence of the victim’s sexual activity with others, but only to prove that
someone other than the defendant was the source of physical evidence.
被害者の被告人以外との性行為。被告人以外の誰かが物的証拠の出所であることを
証明する場合のみ。
(b) victim’s other sexual activity with the defendant, but only if the defense is
consent. 被害者と被告人とのその他の性行為、ただし被告人による「同意」の抗弁
の証拠として。
(c) where exclusion would violate defendant’s right of due process.
被告人のデュープロセスの権利によって提出が必要となる場合。
Example: “Love Triangle Defense.” Defendant should be allowed to show
that the victim had a sexual relationship with X at the time of defendant’s
alleged rape if X was aware of victim’s sexual contact with defendant.
Purpose: to suggest the victim had a motive to falsely claim that the sexual
contact with defendant was nonconsensual. (三角関係。最高裁事例)

17
Evidence 2016 Summer BarBri NY Bar

3. Civil Cases 民事事件での性格証拠


a) THE RULE:
Character evidence is generally inadmissible to prove propensity in civil cases.
原則として性向立証のための性格証拠は証拠能力なし。
HYPO 17. A sues B for automobile negligence.
(a) During the plaintiff’s case in chief, A seeks to offer evidence of B’s reputation for careless driving.
Admissible?
- No. It’s a character evidence to prove the defendant’s propensity.
(b) During the defense, B calls Witness to testify that in her opinion, B is a prudent and careful driver.
Admissible?
- No. Defendant cannot introduce evidence to prove its good character.被告側も良性格立証はで
きない。証人を呼べない。
HYPO 20. Nicole’s estate sues OJ for wrongful death damages, alleging that OJ intentionally killed
Nicole.
(a) During the defense, may OJ properly introduce evidence of his peaceful character?
- No. This is a civil case even though it’s based on a criminal conduct.
刑事事件に関係しているとしても民事事件なので良性格立証もできない。
(b) In support of his claim of self-defense, may OJ properly introduce evidence of Nicole’s violent
character to prove that she was the first aggressor?
- No. Purpose is to prove that Nicole was the first aggressor, which is a propensity purpose, and
it’s inadmissible in a civil case.

h) THE EXCEPTION:
Evidence of person’s character is admissible in civil action where such character is an
essential element of a claim or defense. (FRE 405)
性格が請求又は抗弁の本質的要素である場合には、民事事件での性格証拠も証拠能力を有する
(弾劾目的だけでなく、実質的な証拠となりうる。)。
Only 3 situations ただし、以下の 3 つの場合のみ(Bar でよく聞かれるのは(1)と(2)のみ)
(1) Negligent hiring (negligent entrustment) 過失による雇用(過失による委託)
BIG-8 Negligent Hiring の事例
Employer が不注意なまたは暴力的な性格を有する従業員を過失によって雇ったことにつ
いて訴えられている場合、従業員の性格が in issue 請求の基礎となる。
PR-175 これに対して、Employer の性格(不注意又は暴力的な性格)は In issue 請求の基
礎ではないため、これを立証するための性格証拠は認められない。
BIG-8 従業員であるトラックの運転手が酒酔い運転で少女をひいてしまった場合に、少
女の両親が、従業員と雇用主を訴えている場合であっても、請求の理由が Negligent
Hiring でなく単なる従業員の過失である場合には、Negligent Hiring に該当しない。した
がって、従業員の性格は in issue ではないため、運転手は運転が下手であるとの評判は性
格証拠として証拠能力を有しない。

(2) Defamation 名誉毀損


Defamation には真実性の抗弁があるため、事実として原告が dishonest であるということ
は立証できる。
(3) Child Custody Dispute
PR-20 性格証拠の提出方法
性格が本質的証拠である場合に、性格証拠を提出する場合の形式として
は、Reputation、Opinion、Specific Act のいずれでもよい。
Example (1): P was struck in 2012 by a truck being driven by Charlie, who was acting within the scope
of his employment for Acme Trucking.P sues Acme, alleging that Acme was negligent in hiring Charlie
in 2011 and thereafter allowing him to drive on Acme’s behalf. (Tort theory: Acme knew or should have
known that Charlie was an accident risk.) P may introduce the testimony of character witnesses that
Charlie had a reputation for being a careless driver, and they have a low opinion of Charlie’s care in
driving. P may also prove that Charlie had been involved in three prior accidents.
Example (2): P sues Newspaper for libel based on a story in which P was accused of being dishonest.
To support its defense of “truth,” Newspaper may introduce reputation, opinion and specific-act
evidence about P’s dishonesty; and P may use the same type of evidence to show P’s honesty.

18
Evidence 2016 Summer BarBri NY Bar

Example (3): In a matrimonial[結婚の] dispute, H and W contest the custody of child X. On the fitness
of each parent to have custody, evidence is allowed of each parent’s relevant acts and reputation as well
as opinion evidence bearing on their fitness.

E. DEFENDANT’S OTHER CRIMES OR MISCONDUCT FOR NON-CHARACTER PURPOSE


非性格目的での被告人によるその他の犯罪又は非行(FRE 404(b))⇒有罪判決の証拠能力
刑事有罪判決の利用方法
目的 利用方法
1 Non Character Purpose MIMIC
2 Character Purpose Sexual Crime のみ OK
3 Impeachment Purpose (1) Veracity(2) Knowledge
1. The General Rule:
A defendant’s other crimes or specific bad acts are not admissible during the prosecution’s
case-in-chief if the only purpose is to prove propensity, i.e., because of defendant’s bad
character he is more likely to have committed the crime currently charged.
唯一の目的が性向の立証である場合には、被告人のその他の犯罪又は特定の悪行は、検察の立証段
階では証拠能力なし。
Example: D is charged with robbing bank A. The fact that D robbed bank B six months later
would be inadmissible character evidence.
2. The General Exception 一般的な例外
The defendant’s other crimes or bad acts may be admissible if offered to show something
specific about the charged crime (something separate and apart from a mere propensity to
commit the crime).
起訴されている犯罪に関する特定の事項(犯罪を犯す性向とは別のもの)を証明しようとする場合、
被告人による他の犯罪や非行の事実について、証拠能力が認められる。
3. THE MIMIC RULE
The most common non-character purposes 一般的に性格目的でないもの:
M Motive 動機 (うらみがある、など)
I Intent 意図 (目的)
M Mistake or accident, the absence thereof 誤り・偶発でないこと
I Identity 同一人性 (i.e., connection between the crime and the defendant)
*The modus operandi must be specific and unusual to qualify as evidence for
proving identity (The modus operandi must be a "signature" of the defendant).
C Common scheme / plan 共通する計画
HYPO 19. Defendant is charged with the murder of Officer Johnson. The prosecution seeks to prove
that Defendant was convicted and imprisoned five years ago for narcotics sales in the aftermath of an
investigation and arrest made by Officer Johnson. Defendant objects on the ground of impermissible
character evidence. What ruling?
- The objection should be overruled. This is admissible. This is an evidence to prove specific the
Defendant’s motive to revenge to the offer. 【M】
HYPO 20. Defendant is charged with possession of narcotics with the intent to sell. He defends on the
ground that he was merely a possessor and user—not a seller—of the drugs. The prosecution seeks to
prove that Defendant sold drugs a year ago in the vicinity of the arrest in the current case. Admissible?
- Yes. Defendant’s statement of mind is an issue, i.e. his intention is to sell the drugs or just to possess
them. This is not to prove that he’s a drug dealer, but to prove his intention by showing that he sold
drugs before. 被告人の意図に関するものなので証拠能力あり【I】
HYPO 21. Lizzie Borden is accused of intentionally killing her mother with an ax. Defense: accident.
Prosecution seeks to show that Lizzie threw a knife at her mother during a family quarrel one week
before the mother’s demise. The evidence:
(A) Is admissible because it shows Lizzie’s propensity for violence. OR
(B) Is admissible because it shows the ax incident was not an accident. ?
- (B) is correct. Inadmissible to prove her propensity for violence. If this is to prove absence of mistake,
it’s admissible. 暴力的性向の立証目的では×だが、誤りがなかったこと(偶発性)についてであ
れば OK 【M】

19
Evidence 2016 Summer BarBri NY Bar

HYPO 22. D is charged with the armed robbery of a Wal-Mart in Indianapolis early in the afternoon of
July 1. Defense: mistaken identity. Prosecution seeks to introduce evidence that around noon on July 1,
D robbed a Penney’s and a Sears in Indianapolis, in the same vicinity as the Wal-Mart.
- Yes. This is admissible for the identity purpose to rebut the alibi.
アリバイを崩すための同一人性目的の立証なので OK.【I】
HYPO 23. Defendant is prosecuted for robbing the First National Bank. Defense: alibi. Prosecution
introduces evidence that the robber wore a red ski mask, carried a .38 caliber gun and used a uniquely
worded stick-up note. Prosecution then seeks to prove that Defendant used the same modus operandi
when robbing the Second National Bank a year ago.
- Admissible. This is an identity example. 同一人性の事例として OK。Specific and unusual modus
operandi tends to prove the identity of the defendant. Crucial question is how specific and unique it is.
General propensity purpose is inadmissible. E.g. just gun and mask cannot be used as modus operandi.
特定のかつ普通でないやり口は被告人との同一人性の立証に傾きやすい。どれだけ特定的で
独自性があるかどうかが問題。単に銃とマスクだけでは×【I】
HYPO 24. Defendant is charged with robbing the First National Bank. The prosecution seeks to prove
that two days before the robbery, Defendant stole a white Acura from a neighbor in the same town. The
robber of the First National Bank used a white Acura for the “getaway.” 銀行強盗の犯行に使われた
白の Acura を同じ街で被告が盗んだ。
- Admissible. Common scheme or plan, not to prove the propensity (once a robber, always a robber).
Prove the identity of the defendant through the common scheme or plan.
共通する計画を通じて被告人との同一性を証明。【C】

4. Method of proof of MIMIC-purpose crimes:


MIMIC として利用が認められる他の犯罪についてどのように立証するか。有罪判決があればそれに
よることができるが、有罪判決がなくとも立証は可能。
a) By conviction, or
i) By evidence that proves the crime occurred.
Conditional relevancy standard—prosecution need only produce sufficient evidence
from which a reasonable juror could conclude that defendant committed the other
crime.
BarBri Set 5-2: MIMIC については specific act の立証が許されている。(↔bad character の立証
のときには許されない)

5. MIMIC Evidence in Civil Cases 民事事件での MIMIC:


Although MIMIC evidence most often appears in criminal cases, it may also be used, if
relevant, in civil cases, such as tort actions for fraud or assault. 刑事事件で多く使われるが、
関連性があれば民事でも利用可能。詐欺又は暴力に対する不法行為請求では利用可能。
(e.g., discriminatory tort case)
6. Timing:
If a MIMIC category is satisfied, the prosecution may use other-crimes evidence as part of
its case-in-chief; MIMIC evidence is not dependent on defendant’s introduction of
favorable character evidence. MIMIC を充足すると検察側はその他犯罪の証拠を立証の際に利用
できる。MIMIC の証拠は被告が良性格証拠を提出したかどうかにかかわらず、提出できる。

20
Evidence 2016 Summer BarBri NY Bar

7. Burden of proof: 証明責任


The “sufficiency” standard 証拠の十分性の原則 : The prosecution must produce sufficient
evidence for a reasonable jury to conclude that the D committed the prior act by a
preponderance of the evidence. (FRE 104(b)) 証拠の優越性により、合理的な陪審員をして被告
が以前に犯罪を行ったと結論付けるのに十分な証拠を検察側が提出しなくてはならない。
Note: not beyond reasonable doubts. 「合理的な疑いを超える」ではない
* The ultimate decision on whether the defendant committed a prior bad act is up
to the jury.
The judge merely makes sure that there is sufficient evidence for a reasonable
jury to conclude that the defendant committed the prior bad act.
8. Other Requirements for MIMIC Evidence MIMIC による証拠の他の要件:
a) Pragmatic considerations 実 用 主 義 的 検 討 : Court must weigh probative value vs.
prejudice.証明力と予断のバランス
BSM-83 Robbery で起訴されている被告人に対して、検察側が起訴事実以外に前年に行われた
Armed Robbery2 件、通常の Robbery3 件を証拠として提出。この場合、他の事件は被告人の動
機との関連で提出されており、証拠能力が認められるのが原則となる。しかし、類似の他の
事件が存在することにより、不公正な偏見のおそれが、動機を立証目的とした証拠としての
証拠能力を実質的に上回る場合には、裁判所は例外的に証拠能力を否定することになる。

j) Limiting instructions 限 定 指 示 : Court must instruct jury about limited purpose of


MIMIC evidence
裁判所は MIMIC 証拠の目的が限定されていることについて陪審に指示しなくてはならない。

k) Pretrial notice 審理前通知 : Upon defendant’s request, prosecution must give pretrial
notice of intent to introduce MIMIC evidence.
被告の請求により、検察官は MIMIC を採用する意図があることを通知しなくてはならない。

F. OTHER SEXUAL MISCONDUCT TO SHOW PROPENSITY IN SEX-CRIME PROSECUTION


OR CIVIL ACTION (FRE 413, 414, 415)
THE RULE:
In any case alleging sexual assault, the defendant’s prior specific acts of sexual assaults (such
as rape) are admissible as part of the case-in-chief of the prosecution (in a criminal case) or of
the plaintiff (in a civil action) for the purpose of showing the defendant’s propensity to sexual
assault.
性的暴行事件では、検察側は被告の性的犯罪についての性向を証明する目的で被告の以前の性的暴行を証
拠として利用できる。(児童淫行(child molestation)でも同様)
a) Propensity Reasoning: “Once a rapist, always a rapist” is allowed in federal court. 「かつ
てのレイプ犯は常にレイプ犯である」の原則が認められている。
b) Note: This rule allows prior acts only; NOT reputation or opinion.

21
Evidence 2016 Summer BarBri NY Bar

II. AUTHENTICATION OF WRITINGS 書面の真正 (FRE 901)


1. EXAM TIP: Whenever a writing appears on the exam, be on the lookout for three
potential issues (aside from relevance): 関連性以外に書面証拠は以下の 3 つを確認すること。
a) Authentication 真正性,
b) Best Evidence Rule 最良の証拠, and
c) Hearsay 伝聞証拠.
2. THE AUTHENTICATION RULE FOR WRITING:
If the relevance of a writing depends upon its source or authorship, a showing must be
made that the writing is authentic (genuine), i.e., that it is what it purports to be. 書面の関連
性がソース又は作成者によって異なる場合には、書面は真正であることの証明が必要となる。
*Terminology: The process of authentication – of proving that a document is genuine – is
called “laying a foundation.” 書面の真正性の証明は前提事実の証明と呼ばれる。
A. METHODS OF AUTHENTICATING WRITINGS 書面の真正性の証明方法
Imagine that a party is seeking to prove that DOCUMENT was written by X.
There are four ways to do it (illustration).
1. Testimony by a witness with personal knowledge. 個人的な知識を持つ証人による証言
Example: Testimony from Witness that Witness saw X sign the document
X が書いているところを見た証人。

2. Proof of the author’s handwriting 作成者の筆跡


a) Lay person’s opinion 通常人の意見
(The witness must have familiarity with X’s handwriting as a result of experience in
the normal course of affairs, before the lawsuit – not as a result of preparation for the
litigation).証人となる通常人は X の筆跡について、訴訟の準備のためではなく通常の業務にお
ける経験の結果としてよく知っている者でなければならない。
BSM-141 筆跡の Authentication と異なり、テープに録音された声の同一性についての証言を
Lay Person が行う場合、Lay Person は、訴訟提起後に、かつ証言自体を目的として当事者の声
についてよく知った者であっても、証言することができる。→訴訟準備目的でもよい。

b) Expert comparison opinion 専門家の比較による意見.


(The expert must be qualified and must compare document to a genuine sample or
exemplar X’s handwriting).
c) Jury comparison 陪審員による比較
(The trier of fact compares document to a genuine sample or exemplar X’s
handwriting).
3. Proof by Circumstantial Evidence
A party may rely on circumstantial evidence (anything relevant that connects the alleged
author to the document), such as appearance, contents, substance, internal patterns or
other distinctive characteristics (e.g., if the document refers to information that only X
would know, this may be used as an inference that X is the author).
4. Ancient Document Rule 古文書ルール
authenticity may be inferred if document is:
以下の全てを満たせば真正を推認
a) at least 20 years old
b) facially free of suspicion, and (e.g., 修正されていない)
c) found in a place of natural custody
5. Solicited Reply Doctrine 勧誘に対する返答の原則

22
Evidence 2016 Summer BarBri NY Bar

a) Document can be authenticated by evidence that it was received in response to a prior


communication to the alleged author. 書面の作成者との以前の通信に対する反応として受
領されたものであるとの証拠があれば、書面の真正が証明される。
b) Example: P mails contract offer to X, properly addressed and posted, and later
receives an acceptance purportedly signed by X.
Authentication: Procedural Considerations
Burden of Proof: The “sufficiency” standard 十分性の基準
a) Authentication is a matter of “conditional relevance,” which means that the party
offering the evidence must produce sufficient evidence for a reasonable juror to
conclude that the document is genuine (e.g., that X is the author).
合理的な陪審員が書面の真正性を認める程度に十分な証拠で証明することが必要。
b) Similar to the standard for proof of prior bad acts (MIMIC).
HYPO 25. During plaintiff’s case-in-chief, Witness testifies that, in her opinion, document
was written by X because she is familiar with X’s handwriting. X advises the judge that he
intends to testify during the defense that the document is a forgery and argues that the judge
cannot admit the document into evidence until the judge is personally convinced that the
document was written by X. Good argument?
No. Document is admissible under the sufficient standard. Witness’s testimony is enough to
authenticate. The judge decides the admissibility of the evidence and the authenticity of the
document.
B. SELF-AUTHENTICATING DOCUMENTS 真正性が推認される書面
Some documents are presumed authentic. No foundation testimony is needed for these “self-
authenticating” documents:
1. Official publications 公的な出版物
e.g.: government pamphlets, tax return form
⇒Private Publisher による出版物は 3 や 4 に当たらない限り真正性は推認されない。

2. Certified copies of public or private documents on file in public office


公的機関の証明書付書面の写し(私的・公的を問わず)自動車登録証の証明書付写し
e.g.: deeds, mortgages
3. Newspapers or periodicals 新聞・定期刊行物
e.g.: Wall street journal (ex. offered to prove stock price on a particular day)
4. Trade inscriptions and labels ラベル
e.g.: label on a candy bar (to prove made by Hersey’s)
5. Acknowledged (notarized and signed) document 公証済みの書面
e.g.: notarized document
6. Commercial paper コマーシャル・ペーパー
e.g.: checks, promissory note
7. Certified Business Records 証 明 書 付 業 務 記 録 , offered into evidence under the business
records hearsay exception – must be certified
a) by someone within the business 業務に従事する者によってなされること
b) who knows how the records are regularly made 通常の記録方法を知る者によること
c) and that they were made in a regular way 通常の方法で記録されていること
d) at or about the time of the event recorded 記録された出来事の時点であること
Note: The certification is usually done by attaching an affidavit to the business record.

A. AUTHENTICATION OF PHOTOGRAPHS 写真の真正性


1. Photograph as “demonstrative” evidence 展示証拠としての写真:
23
Evidence 2016 Summer BarBri NY Bar

If the purpose of the photograph is to “illustrate” a witness’s testimony, it can be


authenticated by the witness testifying, based on personal knowledge that the photograph is
a fair & accurate representation of the people or objects portrayed.
目的が証言を説明することであれば、写真が公平かつ正確に対象物を撮影しているという知識に基
づく証人の供述によって写真の真正を証明できる。
*写真の真正性について、通常撮影者の供述は必要でない。
HYPO 26. Alice testifies that she observed the auto accident that occurred at the intersection of
Hickory and Elm Streets on July 1. She is shown a photograph and asked whether it is a fair and
accurate portrayal of the Hickory and Elm intersection as she remembers it on July 1. “Objection: No
foundation that Alice was the photographer.” What ruling?
- Overruled. Evidence is admissible because it is being offered as demonstrative evidence. Alice just
uses the photo to illustrate her testimony. Who the photographer is does not matter. Witness may
authenticate the photo just by admitting that’s a fair and accurate representation.
展示証拠なので証拠能力あり。写真を誰が撮ったかは関係ない。
2. Photograph as a “Silent Witness”無言の証人:
Sometimes, a photograph is not illustrating a witness’s testimony, but rather is itself the
evidence (e.g., photos from surveillance cameras, automatic teller machines, etc.). A party
offering such a photograph must show:
a) That the camera was properly installed & working. カメラが適切に設置、作動していた
b) That the film was properly removed & developed. フィルムが適切に抜かれ、現像された
c) And that the film has not been tampered with.フィルムが不正変更されていない
(1) The most effective way to show an absence of tampering is by establishing a:
chain of custody 物証保管の継続性. By showing everybody holding the film in the
course.
Emanuel-51 設置型の自動撮影装置によって撮影された写真は、装置の信用性さえ立証できれば、いつ、
どのように撮影された写真かを別途立証する必要はない。

BIM-14 写真に証拠能力が認められるには、(i)写真が事案に関係すること、(ii)それらの事実の適切な代理
人としての証人によって証明されることが必要。写真を撮った者を証人として呼び出す必要はなく、写
真で撮られている場所(たとえば事故現場)に詳しい者がその場所であることを証言すれば十分である

24
Evidence 2016 Summer BarBri NY Bar

III. BEST EVIDENCE RULE 最良証拠ルール


A. OVERVIEW
1. Terminology
The Best Evidence Rule is better understood as the “original writings” rule.
2. THE RULE:(FRE 1002)
a) A party who seeks to prove the contents of a writing must either
(1) produce the original writing (or acceptable duplicate), or
(2) provide an acceptable excuse for its absence.
当事者が書面の内容を証明する場合、(i)原本(若しくは許容されるコピー)の提出又は(ii)その
不存在について合理的な理由を示すことが必要。
b) If the court finds the excuse acceptable, the party may then use secondary evidence,
such as oral testimony or a copy, to prove the contents. 裁判所が書面不存在の理由を合理
的だと判断した場合、当事者は、書面内容について口頭での証言など 2 次的証拠を使うことが
できる。
PR-52 事件や事実についての個人的な知識に関する証言に最良証拠ルールは適用されない。
書面化されていない取引の内容に関する証言は最良証拠ルールの範囲外。
PR-72 原本が故意に破棄され、又は提出が拒否された場合には、Secondary Evidence 二次的証
拠の提出は認められないと裁判所は判断している。
Barbri Set 6-2 best evidence rule は書証についてのみ。キャラクターに関する著作権侵害訴訟な
どでの証拠物となりうる Clay doll については適用がない。原告が clay doll を破壊してしまっ
た後に、その形状について証言するという場合、その破壊が good faith であったか否かは関係
ない。

3. Definition:
A “writing” includes documents, recordings, films and X-rays.
書面、記録、フィルム及び X 線も含む。
HYPO 27 Bubba ordered 100 pounds of shrimp from Gulf Shrimp Co. pursuant to a written purchase
order. In his suit for breach of contract, Bubba takes the stand and testifies, “I didn’t get what I ordered.
The purchase order called for 3” jumbo shrimp and they delivered 1” mini- shrimp.” Which of the
following would be a valid objection to Bubba’s testimony?
(A) The actual shrimp are the best evidence of what was delivered.
(B) The purchase order is the best evidence of what the contract required.
- Only (B) is the good objection because the best evidence rule does not apply to shrimp.エビ自体は書
面ではない。
Questions to ask yourself:
a) When does the best evidence rule apply? いつ適用されるか?
b) What is an original? 何が原本か?
c) What is a good excuse? 合理的な理由はあるか?
B. QUESTION #1: WHEN DOES THE BEST EVIDENCE RULE APPLY?
4. THE RULE:
The rule applies only when the party seeks to prove the contents of a writing, which arises
in two principal situations: 書面の内容を立証しようと当事者が考えた場合で、以下の 2 つの状況
でのみ適用される。(書面による証拠の場合にのみ適用されるルール)
a) The writing is a legally operative document (i.e., the writing itself creates rights and
obligations). OR 文書が法的に有効な書面(権利義務を生じさせる書面)、又は
e.g., patents, deeds, mortgages, divorce decrees, written contracts.
b) The witness is testifying to facts that she learned solely from reading about them in a
writing. 証人が書面を読むことによってのみ理解した事実を証言している場合。⇒ルールが適
用されて書面が最良証拠として提出されるべき。そうでなければ直接証言させるべき。

5. BEWARE:
The best evidence rule does NOT apply when a witness with personal knowledge
25
Evidence 2016 Summer BarBri NY Bar

testifies to a fact that exists independently of a (non-legally operative) writing which


records the fact.
事実を記載・記録した法的に有効でない書類が独自に存在したことについて、個人的知識がある証
人の証言には最良証拠ルールは適用されない。⇒書面でなく、証言者の証言を優先して提出。
PR-136 取り調べの際に警官が被告人の自白を聞き、テープに録音した場合であっても、直接自白
を聞いた警官による証言は、Best Evidence Rule によっても制限されない。テープではなく、警官
による Firsthand Knowledge が優先する。
Barbri Set 1-7 テレビのインタビューで人を侮辱するような発言をした場合、defamation はインタ
ビューの放映により実行される。従って、そのテレビを見ていた人が発言内容を証言する場合、
証言は独自の個人的知識に基づくものであり、最良証拠ルールによりテレビの録画に劣る、とい
うことはない。
HYPO 28. Tommy the Terrorist is charged with detonating[爆発させる] a bomb. No one witnessed
the detonation, but it was captured on film by an unmanned surveillance camera. Counterterrorism
Agent Jack Bower testifies that he watched the film and it clearly shows Tommy was the bomber.
Objectionable?
- Yes. No personal knowledge of the burglar. He knows the facts from the film, thus the tape must be
produced as the best evidence.
証人はビデオから事実を知ったので、そのビデオ自体を最良証拠として提出すべき。 この
ケースでは、当該爆発は報道によって引き起こされたのではない。
If a witness saw the bomber directly and the tape also recorded the bomber, the witness can testify.
直接目撃者とビデオがある場合には直接目撃者を採用できる。
HYPO 29. Agent Jack Bower is prosecuted for giving perjured testimony at a congressional hearing
into the use of torture during the interrogation of terrorist suspects. At trial, a congressional aide offers
to testify to what Jack said during the hearing. True or False: The aide’s testimony is improper because
the transcript is the best evidence of what Jack said. Agent は議会での証言中の偽証で訴えられてお
り、議員の補佐官が Agent の聴聞手続の発言を証言。
- False (aide’s testimony is proper). Best evidence rule does not apply here. Testimony is not a legally
operative document. 証 言 は 法 的 に 有 効 な 書 面 で は な い 。 Testimony and perjury still exist
regardless of the transcript. 証言の反訳記録にもかかわらず、証言及び偽証はなお存在する 。
e.g.,反訳記録者が居眠りしていた。
HYPO 30. Agent Jack Bower, claiming he worked a 24-hour shift, sues Boss for nonpayment of wages
and failure to reimburse for expenses.
(a) Without producing any documents, Jack testifies, “I worked 24 hours and my expenses were $15
million.” Boss objects—“Best evidence rule. Produce the time sheets and expense receipts.”
- Overruled. Best evidence rule does not apply. Time sheets are not legally operative documents.
Worker worked regardless of what the time sheets recorded. Expenses occur regardless what the
receipts and cancels record.
ルールの適用なし。タイムシートや領収書は権利義務書類ではない。従業員の勤務時間
はタイムシートによって定まらない
(b) Without producing any documents, Boss testifies: “Jack’s time sheets show he worked only 20
hours, and the receipts show only $10 million in expenses.”
- Sustained. Best evidence rules apply. Boss has no personal knowledge but he knew only by
reading the time sheets and receipts, thus those documents must be produced as original documents.

26
Evidence 2016 Summer BarBri NY Bar

B. QUESTION #2: WHAT QUALIFIES AS THE “ORIGINAL WRITING”? 原本とは何か?


1. Definition of original:
Whatever the parties intended as the original; any counterpart[相当するもの] intended to
have the same effect; any negative( ネ ガ ) of film or print from the negative; computer
print-out.
2. Definition of duplicate:
duplicate—any counterpart produced by any mechanical means that accurately
reproduced the original (e.g., photocopy, carbon copy).
3. THE RULE for duplicates
Rule on duplicates: duplicate is admissible to same extent as original
UNLESS:
(a) it would be unfair (e.g., photocopy of fuzzy fax) to admit the duplicate, or
(b) genuine question is raised as to authenticity of original.
原本の真正性に対する疑問がある (一方当事者が偽造文書であるなど、真正性に対する主張を行っ
ている場合)
So what’s not considered an original? Hand written copy. 手書きの写しは×
PR-33 If data are stored in a computer or similar devise, any printout or other output readable by sight, is an
“original”. データが PC その他の機器に保存された場合、その印刷物又はその他の解読可能なもの
は原本である。

C. QUESTION #3: WHEN WILL NON-PRODUCTION OF THE ORIGINAL BE EXCUSED?


原本の不提出に合理的な理由があるか? (FRE1004)

1. THE RULE:
A party need NOT produce the original (or an acceptable duplicate) if the original
a) is lost or cannot be found with due diligence, 喪失した場合、また正当な調査をしても見
つからない場合 or
BIM-49 実際に“lost”なのかどうかについては、Preliminary Question であるので、裁判所が決
定する。陪審に決定させる必要はない。

b) has been destroyed without bad faith, 悪意なく破棄された場合 or


c) cannot be obtained with legal process (beyond subpoena power or outside of
jurisdiction) 法的な手段によって取得されない場合(管轄権外にある場合).
PR-27 相手方が保有しており、原本を Trial に提出することを拒んでいる場合、原本を提出し
なくてよい。

2. APPLICATION:
If the court is persuaded by preponderance of evidence that the excuse has been
established, then secondary evidence is admissible (e.g., oral testimony or a handwritten
copy). 証拠の優越によって裁判所が理由ありと認める場合、第 2 次的証拠(口頭の証言、手書きの
写し)に証拠能力が認められる。

27
Evidence 2016 Summer BarBri NY Bar

D. “ESCAPES” FROM THE REQUIREMENTS OF THE BEST EVIDENCE RULE


最良証拠ルールの回避:原本が存在するが原本提出を求めないパターン:
1. Voluminous records can be presented through a summary or chart, provided the original
records would be admissible and they are available for inspection. (FRE1006)
証拠が多量でかさばる場合。※ただし、原本自体の証拠能力があり、検査可能な場合。
“large volume”がキーワード
PR-45, BarBri-5-13 証拠の原本となる書類が多量でかさばる場合、原本を提出せずに、チャー
ト、要約又は計算などで提出すればよい。数年間毎日湖の水位を記録していた場合、原本の証拠
能力があり、提出可能である限りにおいては、証拠の要約やチャートなどの提出でよい。

2. Certified copies of public records. 公的記録の証明付写しである場合 (FRE1005)


3. Collateral documents (i.e., not important document in the case).
If the court, in its discretion determines that the document is unimportant to the issues in
the case.(裁判所がその裁量において重要でないと判断した場合)間接的な書類である場合
BRQ-12 Written Evidence の証拠としての重要性が低い場合には適用されず、 Original を提出しな
くてもよい。E.g.) 新聞に掲載された記事

28
Evidence 2016 Summer BarBri NY Bar

IV. WITNESSES 証人
BarBri Set 1-16 A unresponsive answer by a witness is subject to a motion to strike by examining counsel, but not by
opposing counsel. 証人が質問されていないことを勝手に話し始めた場合、質問している代理人が motion を出せ
る。

A. COMPETENCY OF WITNESS, IN GENERAL 証人の適格性


1. Basics: There are two requirements for a witness to be considered “competent” to testify:
適格性(証人が証言をするのに適切かどうか)の 2 要件

a) The witness must have personal knowledge, and


*Personal knowledge = 自分の目で見た・耳で聞いたもの
b) The witness must take an oath or affirmation, which simply means:
1) Demonstrate an understanding of the obligation to tell the truth, and
真実を証言する義務の認識
2) promise to tell the truth 真実を証言することを約束.
BIM-20 裁判官は、職権で証人の呼び出しを行うことができ、証言する証人を取り調べることがで
きる。
PR-68 精神状態に何らかの問題があったとしても、証人としての適格性は否定されない。単に証人
の信用性の弾劾に利用されるのみである。年齢、信条、精神障害、有罪判決、婚姻状態、当事者と
しての訴訟の意図も同様に、適格性の問題ではなく、信用性の問題となる。
Emanuel-39 4 歳の子供であっても( (i)personal knowledge と(ii)oath の要件を満たす限り)証言可
能。
BarBri WS-12 juror は自分が jury として担当する事件と、別の事件であっても jury deliberation の間
の出来事については証言能力がない(incompetence)。他方、自分が jury を務めた事件における証
人の発言を別の事件において証言することはできる。
BarBri Set 3-5 Translator も oath が必要。他方、利害関係については開示不要。

B. “DEAD MAN’S STATUTE” 生前の口頭約束に関する制定法(民事事件のみ)


2. THE FEDERAL RULE:
Under the Federal Rules, there is no Dead Man’s Statute – a witness is not incompetent
simply because she may have an interest in the outcome of the litigation. 連邦法上はかかる
法律はない。証人が訴訟結果に利害関係があるというだけでは証人は不適格とならない。⇒証人は
証言可能。
(問題意識)「死人に口なし」だが死者についての証言を許すべきか?たとえば、契約の不履行を
争っている当事者の一方が死亡した後に生存当事者が訴訟提起を行い、生存当事者その他の利害関
係人が、死亡当事者が生前に口頭で言っていたことについて証言を行う場合に証言が認められる
か、という問題。

3. THE RULE IN SOME STATES:


Some states have Dead Man’s Statutes (or Dead Man’s Acts), which generally provide:
a) In a civil action, 民事事件
b) an interested party 利害関係人
c) may not testify 証言できない
d) against the estate of a decedent (dead party, or his representative, e.g. estate, parents
of dead children.) 死亡している当事者に対して
e) concerning communications or transactions between the interested witness and the
decedent.死亡者との通信・取引についての証言
死人に口無しだからと言って、利害関係人が自分に都合のいいように証言してはダメ。死んだ
のが事件の当事者ではなく第三者(予定されている証人であった)場合にはこの法理の適用は
ない。

4. Definition:
A person is “interested” only if the outcome of the case will have a legally binding effect
on the person’s rights or obligations. 証人が「利害関係」を有するのは、訴訟の結果が権利義務
29
Evidence 2016 Summer BarBri NY Bar

に法的拘束力のある影響を与える場合のみ。
5. Waiver:
The dead person’s rights may be waived if: 死亡者の権利は以下の場合に放棄される
a) The decedent’s representative does not object. 死者の代理人が異議を述べない
b) The decedent’s representative testified about the transaction. 死者の代理人が証言
c) Or, the decedent’s testimony is introduced. (typically, introduction of deposition.)
死者の証言が提出される場合(典型的には証言録取書)
Exam tip:
Although the Federal Rules have no Dead Man’s Statute, the MBE may still test on it by
telling you in the question that the case arises in a state with a Dead Man’s Statute. 連邦法
にはないが MBE では聞いてくることが多い。Erie doctrine によって連邦であっても適用をうける。
問題文に Dead Man Statute についての記載がない場合には、適用されないものとして考え
ること。
HYPO 31. Shania sued Elvis for breach of an oral contract. Elvis denied that any contract
was made. Elvis died before trial. (a) May Shania testify to what Elvis said and did in
negotiating the contract? (b) May Shania’s friend Faith, who witnessed the making of the
contract, testify to what Elvis said and did?
- Under federal law: yes for both (a) and (b).
- Under dead man statute: (a) No (b/c interested witness). (b) yes.

30
Evidence 2016 Summer BarBri NY Bar

B. FORM OF QUESTION (LEADING QUESTIONS) 誘導尋問 (FRE 611)


1. Definition:
A question is “leading” when the form of the question suggest the answer (e.g., “Isn’t it a
fact that . . . .” or unevenly balanced alternatives. such as “Were you driving fast and
furious, or in some other way?”)
2. THE RULE:
a) Leading questions are generally NOT allowed on direct examination of witness.
主尋問では誘導尋問は×
b) Leading questions generally are allowed on cross-examination of witness.
反対尋問では OK

3. THE EXCEPTIONS: 主尋問で誘導尋問できる例外


Leading questions may be allowed on direct examination in four situations
1) Preliminary introductory matters (not controverted issue, e.g. name, address,
occupation etc.) 導入部分に関する事項(争いのない事項、氏名、住所、職業など)
2) Youthful / forgetful witness 若い・忘れやすい証人 (physical/mental weakness 含む)
BarBri Set 3-7 4 歳の子供に証言をさせる場合、主尋問での誘導尋問も認められる。

3) Hostile witness 敵対的証人


PS-120 相手方、相手方の配偶者などがこれに該当する。Improperly uncooperative も含む。

4) Adverse party or someone under the control of the adverse party


e.g., 医療訴訟で基礎的な事実関係を確認するために相手方の医者を証人尋問する場合
PS-120 Emanuel-30
原告が被告を敵性証人として呼んだ場合に、被告の弁護士が被告に対して行う反対尋問において、誘導
尋問をすることはできない。弁護人が自分の依頼者に対して尋問をする場合には、証人尋問者に対し
て、証人が有利な発言をするように仕向けられるため、認められない(味方同士の尋問では誘導尋問は
認められない)。
*主尋問・反対尋問の区別は証人申請をした者による尋問(主尋問・direct examination)か他方当事者に
よる尋問(反対尋問・cross-examination)かという区別である。そのため、原告が自らの証人申請をせ
ず、被告が原告本人を証人申請した場合、被告側の尋問が主尋問、これに対する原告側の尋問が反対尋
問ということになる。

31
Evidence 2016 Summer BarBri NY Bar

C. WRITINGS IN AID OF ORAL TESTIMONY 口頭証言の補助書面


[Important, topic is related to hearsay]
1. Present Recollection Refreshed (FRE 612)
a) Basic rule: Witness may not read from a prepared memorandum (because it’s
hearsay); must testify on basis of current recollection. 準備してきた書面を読んで証言し
てはならない。現在の記憶をベースに証言しなくてはならない。
b) Refreshing Recollection 記憶喚起:
But, if a witness forgets something he once knew, he may be shown a writing (or
anything else) to jog his memory. 知っていたことを忘れてしまった場合には、記憶喚起の
ためにどのような書面その他の資料でも見てもよい。
c) Acceptable Refreshers 記 憶 喚 起 の た め に 利 用 可 能 な も の : anything. But refreshers
cannot be evidence. Only witness’s present recollection can be evidence.
記憶喚起のためにはなんでも利用できるが、記憶喚起のために使われた資料自体は証拠とはな
らない。したがって、伝聞証拠の例外ではない。証人の現在の記憶のみが証拠となる。
d) Safeguards against abuse 濫用防止:
If an item is used to refresh a witness’s memory, the opposing party has a right to:
1) inspect the memory refresher. 調査
2) use it on cross-examination. 反対尋問で使用
3) introduce it into evidence. 証拠として提出

Emanuel-12 Present Recollection Refreshed は(i)証人尋問の間に見て記憶を喚起するものと、(ii)


証人尋問の前に見て記憶を喚起するものがあり、後者の場合についても他方当事者は上記と
同じ濫用防止のための権利を有するが、この場合、裁判所が”justice requires the party to have
those options”と判断した場合に限られる。
HYPO 32. Homer Simpson’s house was burglarized two years ago, and several valuable items were
stolen. Homer sued his insurer for failing to pay the loss covered by his homeowner’s policy. While
on the stand at trial, Homer has trouble remembering all of the stolen items. To refresh Homer’s
recollection, his attorney shows him a copy of a list of the missing items that Homer prepared for the
police the day after the burglary. Insurer objects on the ground of lack of authentication, best evidence
rule and hearsay. HS の家が 2 年前に不法侵入され、価値のある動産が盗まれた。家の所有者の保険に基
づいて HS が保険会社を訴えたところ、HS は盗まれた物が何であったか忘れてしまった。記憶を喚起する
ために、弁護士は HS が警察に届け出るために事件の数日後に作成した逸失物のリストを HS に提示した。
保険会社は、文書の真正性、最良証拠ルール、伝聞証拠を理由に異議。
(a) May Homer’s attorney use the list to refresh Homer’s recollection?
- Yes. You can show the witness anything to refresh the recollection. You may use beer to refresh
the recollection. Do not need to worry about hearsay because it does not become evidence.
記憶喚起のためであれば、証人に何を見せてもよい。 ビールで記憶喚起できるのであれ
ば飲ませてもよい。 もちろんビール自体は証拠ではない。 それ自体が証拠にならないの
であれば、Hearsay について考える必要なし。
(b) If Homer’s recollection is refreshed, may he then read the list into evidence?
HS の記憶が喚起された場合、逸失物のリストを証拠として読み上げることができるか?
- No. The list is not evidence. It cannot be shown to the jury, cannot be read into evidence.
リスト自体は証拠ではない。陪審に対して見せてはならない。

32
Evidence 2016 Summer BarBri NY Bar

2. Past Recollection Recorded (Hearsay Exception)


記録された過去の記憶(伝聞証拠禁止法則の例外) (FRE 803(5))

a) THE RULE: A writing may be read to the jury as a “past recollection recorded” if:
以下の 5 つを満たすと、伝聞例外に該当し、書面自体が記録された過去の記憶として、実質的
証拠としての証拠能力が認められる。
1) Showing writing to witness fails to jog memory 記憶喚起失敗,
2) The witness had personal knowledge at former time, 過去に個人的な知識があった。
3) The writing was either made by the witness or adopted by the witness,
証人によって作成され、又は証人によって採用された書面
4) The writing was made while the event was fresh in the witness’s memory.
証人の記憶が新鮮なときに書面が作成された(2-3 週間以内)。

5) And, the witness can attest that, when made, the writing was accurate.
作成時において正確であったと証人が証言できる。
BIM-7 証人による Unavailability(証言の不能)の要件は必要とされず、証拠能力が認められ
る。
BIM-35 記録された過去の記憶が記載された書面を証拠として提出しようとする場合には、上
記の 5 要件を満たすことが必要となる。これに対して、公判証人が証言を行う前に自己の記
憶喚起のために記録された過去の記録を読んで証言に望む場合には、書面自体に証拠能力を
認めるわけではないので、5 要件を満たす必要はない。証拠能力のない書面も参考にしてよ
い。この場合は、1 の Present Recollection Refreshed となる。

b) METHOD 利用方法:
If the foundation for a recorded recollection is satisfied (points (1)- (5) above), then:
1) The witness may read the document to the jury,
証人は陪審員に対して読むことができる。→Present Recollection Refreshed との違い。
(テープの場合は再生して聞かせることが可能。Emanuel-42)

2) But the witness may not show the document to the jury. 陪審員に見せるのは×
3) But the opposing party may show the document to the jury (by introducing it as
an exhibit). 相手方当事者は陪審員に見せてもよい(証拠物として提出)。
HYPO 33. As in Hypo 32, Homer looks at the list of stolen items he prepared for the police the day
after the burglary. But this time, it fails to jog his memory, and he is still unable to testify on the basis of
current recollection. At this point, Homer’s attorney seeks to read the list into evidence. Objection:
hearsay. How should the court rule? HS の家が 2 年前に不法侵入され、価値のある動産が盗まれた。家
の所有者の保険に基づいて HS が保険会社を訴えたところ、HS は盗まれたものが何であったか忘れてし
まった。HS は警察に届け出るために事件の数日後に HS が作成した逸失物のリストを見た。しかし今回は、
記憶を喚起することができなかった。そして現在の記憶に基づいては証言できない状況。 HS の弁護士が
リスト自体を証拠として提出した。これに対して伝聞証拠の異議。
- Overruled. It’s hearsay because it is to prove the content of an out-of court statement. But, past
recollection recorded is a hearsay exception. Once Homer knew, he forgets and is unable to jog his
memory. He made the memo just after the event and Homer may be able to attest the accuracy. HS はか
つて知っていたが、忘れてしまい、かつ記憶喚起が不可能である。HS がメモを事件直後に作成し、正確
性について証言することができる場合は、伝聞証拠の例外として証拠能力あり。

HYPO 34. After laying foundation, Homer’s attorney seeks to introduce Homer’s memorandum into
evidence as an exhibit.
(a) Proper? No. Too much influence on jury. Reading is allowed but exhibit is not allowed.
(b) May the insurer have the memorandum introduced as an exhibit?
Yes. because opposing party has a right to introduce it as a evidence.

33
Evidence 2016 Summer BarBri NY Bar

C. OPINION TESTIMONY 意見証拠


3. Lay Witness 一般人証人(FRE 701)
a) THE RULE: Lay opinion testimony is admissible if it is:
一般人の意見でも以下の場合には、証拠能力あり。
1) Rationally based on the witness’s perception (personal knowledge),
個人的な認識に基づいて合理的に and かつ
2) helpful to the jury. 陪審員に有用
3) not based on scientific, technical, or other specialized knowledge.
*3)の specialized knowledge に該当する場合、専門家証人であり、qualification が必要。
PS-3 一般証人が意見又は結論の形式で証人自身の推論を証言した場合、証言は証拠能力な
し。たとえば、証人 A が、銃声を聞いた後に B が走り去っていくのを見たという場合、「銃
声を聞いたこと」と「B が走り去っていくのを見たこと」は証言できるが、その事実からの
推論である「被害者である C を撃ったのは B である。」との推論を証言することはできな
い。陪審員もその推論は可能であり、Helpful であるとはいえない。
PR-147 一般証人が証言をする際には Personal Knowledge に基づいて行う。一般的に不動産の
所有者は不動産の価値についての Lay Witness となるが、Neighbor は Lay Witness とはならな
い 。 Neighbor が 市 場 価 値 に つ い て の 鑑 定 評 価 に 基 づ い て 証 言 す る 場 合 に も 、 Personal
Knowledge に基づいていると言えないため、Lay Witness として証言することはできない。

b) Examples: A lay witness may testify about such things as


1) Sobriety (or drunkenness) 飲酒
2) Speed of vehicle スピード
3) Sanity (or insanity)
4) Emotions 感情
5) Smells/odors におい
6) Handwriting 手書き

4. Expert Witness 専門家証人(FRE 702)


a) THE GENERAL RULE: Witness may testify to an opinion as an expert only if:
1) The witness is qualified (by education and/or experience) 専門家としての適格性
2) The testimony is about a subject matter where scientific, technical, or specialized
knowledge will be helpful to the jury.
科学的、専門的、技術的な知識が陪審員にとって有用な場合(obvious なものについては
専門家意見が不要であるため認められない)
BIM-13 専門家としての資格があるかどうかを決定するのは裁判所の権限。陪審員による決定
は不要。専門家としての資格が認められた後でも、専門家としての能力が劣ることを主張し
て証明力を争うことは許される。
PR-21 被 害 者 の 死 因 に つ い て 意 見 を 述 べ る 医 者 が 、 死 因 を 特 定 す る た め の 方 法 が
Experimental(実験的)であり、結果が不完全で公表できるものではないという場合には、当
該医者による死因の分析については証拠能力がない。

b) Proper Basis of Opinion 意見の適切な根拠


1) The opinion must be based upon a “reasonable degree of probability or
reasonable certainty,” and 合理的な蓋然性の程度又は合理的な特定性
2) The opinion must be based on one of the following three data sources:
3 種類のデータソース
(a) The expert’s personal knowledge 個 人 的 な 知 識 (e.g., a treating physician,
diagnosis)
PR-8 専門家証人が基礎とすべき事実については、First Hand Observation 直接的な観
察に基づくものである必要はない。

34
Evidence 2016 Summer BarBri NY Bar

(b) Evidence that is already in the trial record 審理にすでに提出されている証拠


(made known to the expert through a hypothetical question, which is already
recorded as evidence すでに証拠として記録されている証拠をベースとした仮定的
な質問)

(c) Facts outside the record, but only if those facts are of a type reasonably
relied on by experts in the particular field. 記録外の事実だが、専門家の領域で
は合理的に信頼できる内容の事実である場合のみ OK
Note: If an expert relies on facts outside the record, the expert may
generally discuss the bases of the opinion, but may not disclose the
inadmissible facts to the jury because it’s hearsay, unless their probative
value in helping the jury evaluate the opinion substantially outweighs their
prejudicial effect. The opponent, however, may disclose the underlying
bases on cross-examination. 専門家証人が記録外の事実に依拠する場合には(証
拠能力のない資料による場合)、意見の根拠(資料)について議論できるが、伝聞
証拠であるため、事実を陪審員に伝えられるか(資料の読み上げができるか)は、
証拠価値と悪影響との衡量による(FRE703)。ただし、相手方当事者は反対尋問で基
礎となる根拠を伝えることができる。
BIM-40 専門家証人の意見の根拠は、専門家証人の証言が証拠能力を持つために、根拠自体が
証拠能力を有することは必要ではない。ただし、専門家の領域で合理的に信頼できる内容の
事実であることが必要。

HYPO 35. Dr. Seuss, a board-certified child psychiatrist, testifies, “In my opinion, within a reasonable
degree of medical probability, Bartholomew Cubbins’ preoccupation with hats is a disabling psychosis.
My opinion is based on (1) my own clinical interviews and tests of Bartholomew; (2) exhibits A and B
in evidence (MRI test results, medical office records of Dr. Grinch); (3) interviews of Bartholomew’s
friends Wump, Gump and Thump; and (4) a written report prepared by Dr. Sigmund Fraud.
(a) Bartholomew moves to strike Dr. Seuss’s opinion because it is based, in part, on inadmissible
hearsay. How should the court rule?
- Overruled. Dr. Suess’s opinion is allowed as expert opinion. (1) personal knowledge, (2)
recorded evidence, and if (3) and (4) are within those materials relied on experts in the field, then
this is an admissible expert opinion.
(b) Should Dr. Seuss be permitted to testify further, “Let me tell you what Wump said during our
interview, and let me read you what was in Dr. Sam I. Am’s report”?
- Generally it goes too far. An expert may give an opinion and generally describe its bases but
cannot introduce the contents of the materials.

c) Relevance and Reliability 関連性及び信頼性


1) THE RULE: To be admissible, expert opinion must be relevant to the issue at
hand and sufficiently reliable. That means:
(a) The expert has used reliable methods and 信頼できる方法
(b) The expert has reliably applied those methods to the particular facts of the
case. 事案の特定の事実について信頼できる方法を適切に適用
2) The reliability standard for scientific evidence 科学的証拠の信頼性の基準
Court serves as “gatekeeper,” and will use four principal factors to determine
reliability of principles and methodology used by expert (all types) to reach
opinion (Daubert)—“TRAP”
(i) Has the methodology been tested? 方法論はテストされているか?
(ii) Are there known rates of error? 誤りの確率はあるか?
(iii) Has the methodology been accepted?
当該方法論が一般的に受容されているものか?
Note: General acceptance is NOT required (b/c this is one of factors)
(iv) Has the methodology been subject to peer review in publication?
方法論は同領域の専門家による意見を得ているか?
d) Learned Treatise in Aid of Expert Testimony (Hearsay Exception)
専門家証人の補助目的の専門的学術書・論文(伝聞証拠の例外) (FRE 803(18))
1) On direct examination of party’s own expert:
Relevant portions of treatise, periodical, or pamphlet may be read into evidence
35
Evidence 2016 Summer BarBri NY Bar

as substantive evidence (to prove truth of matter asserted) if established as


reliable authority.
あくまで専門家証人を呼んでいることが前提
2) On cross-examination of opponent’s expert:
Read into evidence to impeach and contradict opponent’s expert.
Comes in as substantive evidence.
3) BUT learned treatise may not be introduced as exhibit to jury. 見せるのは不可

5. Ultimate Issues 主要事実に関する事項


a) THE RULE:
Opinion testimony (lay or expert) is not objectionable just because it embraces an
“ultimate issue” in the case (e.g., in DWI (driving while intoxicated) case, layperson
testifies “X seemed drunk” could be admitted).
主要事実についても意見証人が認められうる。
BUT, all other requirements for opinion testimony must be satisfied, including the
requirement that the opinion is HELPFUL (e.g., witness would not be allowed to give
opinion that “defendant is guilty (or innocent)”).
b) THE EXCEPTION:
In a criminal case, “Ultimate issue” is still proper objection if expert seeks to give
direct opinion that defendant did or did not have relevant mental state. 刑事事件では、
専門家証人は主観的要件について証言できない。
Thus, the following is not allowed: “D’s insanity prevented him from understanding
that he was shooting at the victim in this case.” The expert can only testify in general
terms about the effects of a defendant’s mental condition without linking it to the
particular case, such as: “D has schizophrenia. A person with such disease cannot
distinguish fact from fantasy.”
HYPO 36. In a personal injury case, Defendant is alleged to have been driving recklessly at the time of
a car accident. Witness who observed the event testifies that Defendant looked angry, smelled of
alcohol and drove through the intersection at 80 m.p.h. Witness then states, “It looked to me as though
Defendant was engaged in conduct constituting a reckless disregard for the safety of others.”
Objectionable?
(A) Yes, because Witness is testifying to the ultimate issue.
(B) Yes, because Witness’s opinion is not helpful.
B is correct! Because the witness statement is mere legal jargon

36
Evidence 2016 Summer BarBri NY Bar

D. CROSS-EXAMINATION 反対尋問
1. Cross-examination is a right. 反対尋問は権利
If a witness testifies but then cannot be cross-examined, the witness’s direct testimony will
be struck.
反対尋問ができないのであれば、証人の主尋問はなかったことになる。
BIM-4 反対尋問の範囲・程度は最終的には裁判官の裁量。裁判官は効果的な真実の確認を促進し、
時間の無駄を避け、不合理な嫌がらせから証人を保護するために合理的なコントロールを有する。
特に意味のある反対尋問の十分な機会が与えられたような場合には、反対尋問を打ち切ることがで
きる。

2. Proper subject matter of cross-examination:


a) Matters within the scope of direct examination, or 主尋問の範囲内の事項
b) Matters that test the witness’s credibility. 証人の信用性に影響する事項
PR-17 反対尋問は(a)主尋問の範囲内の事項、(b)証人の信用性に影響する事項に限定されるのが通常
であるが、裁判所の裁量によって、主尋問の範囲を超えている内容であっても、主尋問をする場合
と同様の内容の追加事項についての質問をすることも可能である。
BarBri Set 3-16 Cross-examination は good faith で行われる必要がある。例えば、bad acts が実際にあるか
どうかについて聞いてみないと分からない場合、good faith base での質問が許される。

E. CREDIBILITY AND IMPEACHMENT, IN GENERAL 信用性と弾劾


1. Definitions:
a) Credibility 信用性: whether a witness is believable. It rests on three things:
1) Perception 知覚
2) Memory 記憶
3) Honesty 誠実性
b) Impeachment 弾劾: the process of trying to demonstrate that a witness is not credible.
c) Rehabilitation 信憑性回復 : the process of trying to repair a witness’s credibility after
the witness has been impeached. 反対尋問によって損なわれた証人の信用性を回復するため
の、反対尋問で取り上げられた事項に関する再主尋問
2. Bolstering Own Witness
a) RULE: not allowed until after the witness’s credibility has been attacked.
(b/c no need to do it!)
HYPO 37. Plaintiff calls Witness 1 to the stand. Witness 1 testifies that she saw Defendant’s car run the
red light. Defense counsel states that she has no questions for the witness. After Witness 1 steps down,
Plaintiff calls Witness 2 who testifies, “Witness 1 has a good reputation for truthfulness.”
Objectionable?
→Yes. because, Improper bolstering.
Variation: Witness 1, after testifying that she saw Defendant’s car run the red light, then testified, “I
told everyone at work the next day that I had seen Defendant run the red light.” This is an inadmissible
prior consistent statement: (b/c (i) improper bolstering no attack, (ii) minimal probative value (just
repeating), (iii) hearsay)

b) Exception: The witness’s prior identification of the person (e.g., Witness testifies
that she recognizes D, sitting in court, as the perpetrator[犯人]. In addition, “I picked
D out of a line-up two weeks after the robbery”). Might seem like hearsay (out-of-
court statement offered to prove truth of statement) but prior identification by trial
witness is not barred by hearsay rule. It is labeled as “exclusion” from hearsay, and
comes in as substantive evidence (not bolstering credibility). bolstering ではない。
NOTE: Witness who made prior identification must testify at trial and must be subject
to current cross-examination.
3. Impeachment of Own Witness 自分の証人の弾劾(FRE 607)

37
Evidence 2016 Summer BarBri NY Bar

RULE: It is permitted without limitation.


Note: Even though the Federal Rules speak only of impeaching a witness on cross-
examination, a party can impeach her own witness during direct examination.

F. IMPEACHMENT METHODS 弾劾の方法


a) Overview: Methods of impeachment—
1) Prior Inconsistent Statements 以前の不一致の供述
2) Bias, Interest or Motive to Misrepresent 偏見、利害関係、不実表示の動機
3) Sensory Deficiencies 知覚上の障害
4) Reputation or Opinion 評判又は意見
Bad Character for
5) Criminal Convictions 刑事事件の有罪
Truthfulness (4,5,6)
6) Bad Acts (without conviction) 悪行
7) Contradiction 矛盾
b) Procedural Overview:
1) Intrinsic impeachment: Cross-examination of the witness you are trying to
impeach (confronting the witness). 弾劾しようとする証人に対する反対尋問
2) Extrinsic impeachment: Impeaching a witness by introducing extrinsic evidence
(e.g., documentary evidence or by calling other witness). 書証の提出又はその他の証
人申請による弾劾
Note: Extrinsic evidence cannot be used for (6) bad acts and (7) contradiction.

1. Prior Inconsistent Statements 以前の供述との不一致


a) Definition: A prior inconsistent statement is simply a prior statement (orally or in
writing) that is materially inconsistent with the witness’s trial testimony.
以前の供述と不一致の供述とは、証人の公判での証言と重要な点で一致しない、以前になされ
た供述である(以前の供述は書面でも口頭でもよい)
BarBri Set 1-2 "記憶がない (does not remember)"という供述は以前の供述との不一致供述にな
らない。そのため、記憶がないという証人の供述を弾劾する目的で過去の供述を証拠提出す
ることはできない。

b) THE RULE: A prior inconsistent statement may be used only to impeach a witness.
以前の不一致の供述は、証人の弾劾目的でいつでも利用できる。
(1) Rationale: A witness who says one thing on one occasion and another thing on
another occasion may not be credible. 信用性に欠ける。
(2) Purpose: Ordinarily, a prior inconsistent statement is admissible only to
impeach (i.e., not as substantive evidence that the prior statement is true).
弾劾を目的するのであれば常に証拠能力あり。
(3) Exception: A prior inconsistent statement may be admitted both to impeach and
as substantive evidence (i.e., to prove the truth of the prior statement, this is a
hearsay exception), if the statement was made:
(a) orally under oath, and 誓約の下に口頭で行われた場合
(b) as a part of a formal hearing proceeding of trial or deposition .
審判手続き又は証言録取の公式手続きの一部として行われた場合
弾劾証拠かつ実質的な証拠として、証拠能力あり(伝聞例外になる VI.D.2.参照)。
BIM-1 Grand Jury 起訴陪審での供述も含む。

c) Procedural considerations:
(1) THE RULE:

38
Evidence 2016 Summer BarBri NY Bar

A witness who is being impeached with a prior inconsistent statement must be


given an opportunity to explain or deny prior inconsistent statement . 以前の供述で
弾劾されている証人は、以前の供述に関する説明又は否定をする機会を与えられなければ
ならない。
Timing: Confrontation timing is flexible. The inconsistent statement may be
proven by extrinsic evidence, so long as the Witness is later given an opportunity
to return to the stand and explain.
(2) EXCEPTION:
If the Witness is the opposing party, there is NO need to give the witness/party an
opportunity to explain the prior inconsistent statement. 相手方当事者本人が証人の
場合には、説明の機会を与える必要はない。
Related Hearsay Issue for Party Statements: A prior statement of the
opposing party will also be admissible for substantive evidence under the
separate hearsay exception for party admissions. (FRE 801(d)) (VI.E 参照)
当事者による承認に基づく伝聞証拠の例外として実質的証拠としての証拠能力が認められ
る。
HYPO 38. Defendant is sued for negligence in a multi-vehicle accident in which he was driving his
Suburban. Witness testifies for plaintiff that she saw the Suburban run the stop sign.
(a). On cross-examination, may Defendant’s counsel seek to establish that a few days after the accident,
Witness told the police that the Jeep Cherokee, not the Suburban, ran the stop sign?
- Yes. Prior inconsistent statement is admissible in order to impeach a witness’s testimony.
(b). If Witness admits she made the prior inconsistent statement, may Defendant use the statement as
substantive evidence that the Jeep Cherokee, rather than the Suburban, ran the stop sign?
- No. Prior inconsistent statement is inadmissible to prove the truth unless the hearsay exception
applies, i.e. either oral statement under oath or other formal hearing proceeding. Statement to
police does not fall within the exception.
(c). What if Witness made her prior inconsistent statement about the Jeep Cherokee during a pretrial
deposition in which she gave sworn testimony?
- Yes because a prior inconsistent statement in deposition statement is a hearsay exception.
HYPO 39. In an auto accident case, Plaintiff testifies that she was wearing her seat belt. Defendant
does not cross-examine her. During the defense, Defendant calls Joe the Bartender, who testifies that
Plaintiff told him, at Joe’s bar a week after the accident, that she had not been wearing her seat belt.
(a). Should Plaintiff’s motion to strike be granted on the ground that Plaintiff was not given an
immediate opportunity to explain or deny the inconsistency?
- No. Because the witness is the opposing party, no need to give the witness an opportunity to
explain.
(b). Is Plaintiff’s statement admissible to both impeach Plaintiff and as substantive evidence that she
was not wearing her seat belt at the time of the accident?
- Yes. Though this is not within the prior inconsistent statement allowed to prove the truth, this is
within another hearsay exception for party admissions.

39
Evidence 2016 Summer BarBri NY Bar

2. Bias, Interest or Motive to Misrepresent 不実表示に関する偏見、利害関係又は動機


a) Definition:
Bias may be based on any fact that would give a witness a reason to testify favorably
or negatively about a party’s case.
b) Examples: The witness is
(1) a party 当事者
(2) a friend, relative, or employee of a party 友人、親族、当事者の従業員
(3) someone paid by a party 当事者が支払を行っている者
(4) someone with a grudge against a party 当事者に恨みを持っている者
(5) anyone who has something to gain by the case coming out one way or the other.
訴訟の結果により何らかの利益を得る者。
(6) prosecution witness who is testifying pursuant to plea bargain.
Trial を控えた者は検察に有利な証言をする可能性があるので bias あり。

c) Form: any form is OK


☆ 証人の Bias の立証はどの方法でも OK (specific act の証拠も可)

d) Procedural issues:
(1) Witness must be confronted with alleged bias while on the stand.
(2) If confrontation prerequisite is met, bias may be proven by extrinsic evidence.
証人本人に対する直接弾劾の前置が求められるが、直接弾劾をした後であれば間接証拠に
よる弾劾も認められる。
3. Sensory Deficiencies 知覚上の障害
a) Definition: Anything that could affect the witness’s perception or memory.
証人の知覚又は記憶に影響を与えるもの
b) Examples: bad eyesight, bad hearing, mental retardation, forgetfulness, intoxication
at time of event or while on the witness stand. 目が悪い、難聴、知能発達傷害、中毒
c) Procedural issues:
(1) Intrinsic confrontation is not required.
対面は不要(反対尋問でなくてよい。)
(2) Extrinsic evidence is allowed. 外部証拠も許容される(書面、他の証人 OK)。
4. Reputation or Opinion About Witness’s Bad Character for Truthfulness
真実性に関する証人の悪性格についての評判・意見 (FRE 608) (⇒被告人の性格についての証拠との
混同に注意)

a) Veracity : The character trait of being truthful.真実性に関するもののみ


A witness’s bad character for veracity is a frequent subject of impeachment and
governed by very specific rules (Impeachment methods 4, 5, and 6). If a party
testifies, the party is subject to impeachment for bad character for truthfulness
regardless the exclusion rule for character evidence for propensity.
b) THE RULE: A party may impeach a witness (the “target witness”) by calling
another witness (the “character witness”) to testify to the target witness’s bad
character for veracity. 他の証人(性格証人)を証人の悪性格を証言させるために連れてきて、
証人を弾劾することが出来る。Extrinsic Evidence で立証可能。
(1) Rationale: If the witness is a “lying kind of person” (i.e., has a propensity to
lie), then the witness might be lying on the stand. (“once a liar, always liar” is
allowed.) うそつきであれば、また嘘をついている。
(2) Form of the testimony (same as the rule for character evidence):
(a) reputation or opinion. 評判・意見
(b) specific acts are NOT allowed. 特定の行為については×
(3) Procedural issues: Any witness who has testified may be impeached by this

40
Evidence 2016 Summer BarBri NY Bar

method and extrinsic evidence is allowed.


証人は弾劾されうるものであり、外在的証拠(書面・他の証人)も許容される。
HYPO 40. Larry testifies for the prosecution that he saw Defendant commit the crime. During the
defense: Defendant calls Rev. Al to testify that Larry has a lousy reputation for truthfulness among
members of Rev. Al’s congregation, and in Rev. Al’s opinion, Larry is not a truthful person.
(a). Admissible to suggest Larry’s testimony is false?
Yes. Both reputation and opinion are allowed forms.
(b). May Rev. Al follow up his opinion as follows: “Let me tell you how I reached my opinion of
Larry. During the past year, he lied to me on six separate occasions.”
- No. This is inadmissible because it is a specific act testimony.

5. Criminal Convictions 刑事有罪判決(FRE 609)


a) Rationale: A person who has been convicted of a crime is more likely to lie under
oath than is a person with an unblemished record. 犯罪人は嘘をつきやすい。
b) THE RULES:
(1) Two types of admissible convictions:
(a) Crimes of false statement
Conviction of any crime (felony or misdemeanor) as to which the
prosecution was required to prove false statement as an element of a crime.
 e.g., Perjury, false statement, fraud, embezzlement
 Not: Crimes of violence, drug crimes, theft
PS-19 Crimes of false statement の場合は、常に証拠能力が認められる。裁判所が裁
量によって証拠から除外することはできない。

(b) Felony
Admissible if the probative value of the conviction (on the issue of
veracity) outweighs the risk of unfair prejudice to a party.
重罪は不公正な偏見のリスクを有罪判決の証拠力が上回れば証拠能力あり。
Note: Misdemeanors is always inadmissible (except for crime of false
statement).
BIM-3 Felony Conviction の記録(刑事有罪判決)の写しは、伝聞証拠であるが、伝
聞証拠の例外に該当するものである。したがって、弾劾目的の利用のみならず、
刑事・民事を問わず判決の本質的内容について証拠能力が認められる。
BarBri Set 4-3 felony の判決は、その判決が guilty plea によって得られたものか trial
を経た物かを問わず証拠能力が認められる。

(2) Time limit:


To be admissible, a conviction or the release from prison, whichever is later must
be within 10 years since (i) date of conviction OR (ii) date of release from
confinement imposed for the conviction, whichever later, unless the proponent
shows probative value of credibility is substantial.
(i)判決日又は(ii)懲役終了日のいずれか遅い日から 10 年以内のものでないと証拠能力なし。
但し、裁判所が Probative Value を認めた場合は 10 年以上のものでも OK.

(3) Procedural issues


(a) Proof: Conviction may be proven
 Intrinsically (by asking the witness about it on cross-examination)
内在的(反対尋問で証人に直接質問)or
 Extrinsically (by introducing a record of the conviction).
外在的(有罪歴の記録を提出)
(b) Timing: No need to give the witness an opportunity to explain.
証人に説明の機会を与える必要はない。
HYPO 41. Defendant is prosecuted for arson. At trial, Defendant testifies in his own behalf, urging
that the fire at his warehouse was an accident. On cross-examination, may the prosecutor properly ask
Defendant:

41
Evidence 2016 Summer BarBri NY Bar

(a). Whether he was convicted eight years ago of misdemeanor income tax fraud?
- Yes. Conviction of any sort of claim of false statement can be used to impeach the defendant. 9
年前の所得税脱税の軽罪でも、虚偽の言動に関するいかなる種類の有罪も弾劾証拠として
証拠能力を有する。
(b). Whether he was released from prison nine years ago for his misdemeanor conviction for possession
of marijuana?
- No. Not admissible. Possession of marijuana is a misdemeanor and is not admissible to impeach.
マリファナ所持は、Misdemeanor であり、虚偽の言動に関する犯罪ではない。
(c). Whether he was convicted two years ago of misdemeanor shoplifting?
- No. It’s a misdemeanor. Shoplifting or theft is not a crime of false statement.
(d). Whether he was convicted five years ago for a murder?
- Maybe. This is a felony and then the question is whether the probative value outweighs the risk
of unfair prejudice.

c) REVIEW AND SUMMARY: Convictions 刑事有罪判決


Purpose Admissible? Rationale
被 Propensity to Commit Crimes No. Unfairly prejudicial
告 性格目的(原則)
Propensity to Commit Sexual Rape is treated
Yes. differently
Assault 性的犯罪
Directly relevant to
MIMIC the current charges;
Yes.
(Non-propensity) NOT offered for
propensity
証 Common: Must be within Convictions for
人 Impeaching a Witness’s 10 years crimes of dishonesty
Character for Veracity Crimes of deceit: Yes or for serious crimes
証人の信用性について弾劾する Misdemeanor: No may show that a
目的 Felony: maybe witness is willing to
lie under oath
Cross-examining a Character Yes but question only Convictions are the
Witness Purpose: to impeach kind of things that
性格証人に対する反対尋問 character evidence. affect people’s
(被告人が性格証拠を提出した場 opinions and
合の反論) reputations
6. Prior Bad Acts (without conviction) that reflect adversely on witness’s character for
truthfulness (FRE 608) 犯罪ではないが悪行(真実性についての証人の性格に反するもの)
a) THE RULE:
A witness may be asked about prior bad acts if those acts relate to truthfulness (e.g.
lying in resume) in cross-examination. 真実性に関するような事項について、証人に対し
て反対尋問で過去の悪事について質問できる。
BarBri WS-11:裁判所は、その裁量において、検察官が被告人質問において、被告人の真実性
についての性格に反する事項を反対尋問で質問すること(被告人が履歴書に虚偽記載をした、
など)を認めることができる。

b) Limitations:
(1) Basic:
 Cross-examiner must have good-faith basis for the inquiry, and permission to
make the inquiry is subject to the court’s discretion.
 The inquiry is limited to the act of untruthfulness itself, not its consequences,
such as job termination, civil judgment, or arrest.
(2) Method: The bad act may be proven by intrinsic evidence only. Extrinsic
Evidence is NOT allowed.
反対尋問(内在的証拠)のみで立証(反対尋問者は証人の答えによって反論される)。

42
Evidence 2016 Summer BarBri NY Bar

PR-41 Extrinsic Evidence 外在的証拠(他の証人など)による Prior Bat Act の立証は認め


られない。反対尋問においては、特定の行為について質問することも許される。

(3) Exam tip: Proof by extrinsic evidence may still be allowed if the bad act is
relevant for some other purpose (such as proof of bias).
悪事に関する質問がその他の目的(偏見の立証)のためであれば、外在的証拠による証明
もなお許される可能性がある。→偏見の立証はいつでもどのような方法でもよい。
☆ 証人の Dishonesty の Propensity の立証目的ではなく、証人の Bias の立証はいつでもど
のような方法でもよい。
*被告人による被告人の良性格立証に対する検察側の立証との比較
検察側は悪性格立証を行う場合、特定の行為は反対尋問のみで立証可能。
許される立証方法は以下のとおりで、特定の行為については証人の弾劾の場合と同じ。
(1) Extrinsic Evidence(独自の証人)→評判・意見のみ
(2) Intrinsic Evidence(反対尋問)→特定の行為
HYPO 42. Witness gives favorable testimony for Defendant. On cross-examination, Plaintiff asks
Witness whether she assaulted her mail carrier two years ago (no charges were brought).
- Not allowed. It does not relate to the witness’s truthfulness. Assault is a crime but the witness is not
convicted. 真実性に関連しないので×
HYPO 43. After Witness testifies for Defendant, Plaintiff asks Witness whether she made false
statements in an application for food stamps in July 2001 (no charges were ever brought).
- Allowed. Making false statements is a bad act related to truthfulness.
HYPO 44. Same cross-examination. Witness vehemently denies making false statements in the
application for food stamps. May Plaintiff thereafter call a welfare agent to prove that Witness made the
false statements?
- No. You may ask about bad acts but cannot introduce extrinsic evidence to prove bad acts.
HYPO 45. Prosecution of Michael for embezzlement of the office petty-cash fund. Dwight testifies for
Michael. On cross-examination, Dwight is asked whether he was arrested three years ago for passing
counterfeit money. Objectionable?
- Yes. Arrest is not a bad act. Arrest is a mere accusation and is not an act of the witness. You should
question about the witness’s act itself. 逮捕は Bad Act ではない。逮捕は単なる Accusation であっ
て、証人自体の行為ではない。証人自体の行為について質問すべき。有罪となればよいが、
逮捕だけではその行為を行ったのかどうかは不明確。 (+jury に悪印象を与えるため、証拠能
力自体を否定する必要がある。)
HYPO 46. Prosecution of Donald. Winston testifies for the prosecution. On cross-examination, Winston
is asked whether he was arrested a month ago for selling marijuana and is awaiting trial on those
charges.
- Allowed. Marijuana is not related to truthfulness and arrest is not a bad act. However , evidence of a
pending trial is admissible for the purpose of proving bias. (The pending trial indicates that he might
be testifying to curry favor with the prosecution.) Biased, because he may witness in favor of (or
against) prosecutors.

43
Evidence 2016 Summer BarBri NY Bar

c) REVIEW AND SUMMARY: Arrests 逮捕歴


Purpose Admissible? Rationale
Impeaching a Inadmissible An arrest is not a conviction, and the
Witness’s character for arrest itself is not a prior bad act.
veracity (Note: Witness’s bad act related to
truthfulness may be admissible )
Impeaching a Character Yes. Arrests (while not proof of the bad act)
Witness’s Knowledge Question only are the kind of things that affect people’s
opinions and reputations
Impeaching a Witness Yes. An arrest may make a witness biased
by Showing Bias against law enforcement; or, pending
charges may give a witness an incentive
to “curry favor” with the prosecution.
7. Contradiction 矛盾
a) THE RULE:
A witness may be impeached by showing that she made a mistake or lied about any
fact she testified to during direct examination.
主尋問中に証言した事実について嘘や誤りがあった場合、弾劾される。
b) Procedural issues:
(1) If the contradiction goes to an issue that is significant to the case, then it may be
proven by extrinsic evidence.
事案にとって重要な問題についての矛盾は外在的証拠を使って立証してもよい。
(2) If the contradiction goes to a matter that is collateral (insignificant to the issues
in the case or to the witness’s credibility), then proof is limited to intrinsic
evidence (and the cross-examiner is stuck with the witness’s answer).
本題に関係しない事項の矛盾(事件中の争点にとって重要でない、又は証人の信用性)の
弾劾は、内在的証拠(反対尋問)に限定される。
訴訟全体にとって重要な事項はいろいろな方法をとることができるが、無関係な事項
についてまで、書証や他の証人を呼ぶのは不要。無駄は省く。
BIM-13 Collateral かどうかの判断は、証人による反対の主張がなかった場合に、証拠
能力があるかどうかで判断する。もし証拠能力があるのであれば、証人の供述は無関
係な事項についての弾劾により排除される。
HYPO 47. In an auto accident case, Witness testifies for Plaintiff that, while leaning against a maple
tree near the intersection of Yale and Harvard Avenues, he saw that the traffic light was red for
Defendant as his car entered the intersection and hit Plaintiff. On cross-examination, Witness is asked
(a) “Isn’t it a fact that the tree near the intersection of Yale and Harvard is an oak tree?” and (b) “Isn’t it
a fact that the traffic light was not functioning at all on March 1?” Witness insists that his direct
testimony was accurate.
(a). During the defense, may Defendant properly call a municipal employee to testify that the tree at the
corner of Yale and Harvard is an oak tree?
- No. Questioning is fine. This is collateral, insignificant to the case. It’s a waste of time.
(b). During the defense, may Defendant properly call a police officer to testify that the traffic light at the
intersection was not functioning at all on March 1?
- Yes. This is a significant issue and contradiction with extrinsic evidence is admissible way.

44
Evidence 2016 Summer BarBri NY Bar

Impeachmen Means of Proof Foundation Note


t Method
Prior •Cross-examination Witness must be given Prior Inconsistent
Inconsistent •Extrinsic evidence (if not a opportunity to explain or Statements may be
Statements collateral matter) deny the inconsistentadmitted both to
statement. (exception for impeach and as
hearsay declarants) substantive evidence, if
the statement was made:
(a)orally under oath, and
(b)as a part of a formal
hearing proceeding of
trial or deposition.
Bias •Cross-examination Witness must be asked on Specific act の 証 明 も
•Extrinsic evidence cross-examination about facts OK
showing bias or interest
before extrinsic evidence is
allowed. If these facts are
admitted on cross-
examination, admissibility of
extrinsic evidence is within
court’s discretion.
Sensory •Cross-examination None required
Deficiencies •Extrinsic evidence
Reputation or • Extrinsic evidence (e.g. None required Specific act is NOT
Opinion about calling other witnesses) allowed
Witness’s Bad
Character for
Truthfulness
Criminal •Cross-examination None required • Felony: if the
Convictions •Extrinsic evidence probative value of the
conviction outweighs
the risk of unfair
prejudice.
• Crime of false
statement
• within 10 years
Prior Bad Act •Cross-examination only Not applicable Arrest is not a bad act.

Contradiction •Cross-examination None required


•Extrinsic evidence (if not a
collateral matter)

45
Evidence 2016 Summer BarBri NY Bar

G. REHABILITATION 信憑性回復
1. Timing
RULE: Generally, a witness may be rehabilitated only after the witness’s credibility has
been attacked through impeachment.
信憑性回復は、証人の信用性が弾劾により攻撃されたあとでないとできない。
Note: Introducing evidence to support a witness’s credibility before the witness’s
credibility has been attacked is called “bolstering” and is not allowed.
信用性が弾劾される前に証人の信用性を補強する証拠の提出は「補強」と呼ばれるが、 「補強」は
認められていない。
Note: A witness’s prior statement of identification is admissible, even if the Witness’s
credibility has not yet been attacked.
同 一人性に関する以前の供述については、信用性が弾劾されてなくても証拠能力を有する。
(VI.D.2.a 参照)

2. Methods of Rehabilitation 信憑性回復の方法


a) Good character for truthfulness 真実性についての良性格(FRE 608)
(1) THE RULE:
It is admissible only when impeachment suggests that the witness is lying (as
compared to merely being mistaken).
i.e., If a witness’s character for truthfulness has been attacked (through
impeachment methods (4) bad character, (5) criminal conviction, or (6) prior bad
acts), then the opposing party may introduce corresponding evidence of the
witness’s good character for truthfulness. 真実性についての性格が弾劾された場合に
は、反対当事者は良性格を立証する証拠を提出できる。
(2) Form of the rehabilitation (same as the rule for impeachment):
(a) reputation or opinion. 評判・意見
(b) specific acts are NOT allowed.特定の行為は×
b) Prior consistent statement 以前と一致する供述 (FRE 801(d))
(1) THE RULE: A prior statement may be used in any of the following:
(a) To rebut a charge of recent fabrication:
When? If the witness’s trial testimony is charged as a recent fabrication
(e.g., product of recent improper influence or bias), a prior statement by the
witness that is consistent with her testimony will be admissible to rebut the
charge IF the statement was made before the motive to fabricate arose.
賄賂を受け取ったという弾劾に対し、賄賂の受領前から同じ証言をしていると反駁
(b) To rebut a contention of inconsistency: or
Example: A police officer who testifies that she singled out the defendant
because of a prominent scar on his face is impeached with a showing that
the officer omitted to mention the scar in her informal handwritten report.
For rebuttal, it could be shown that the officer mentioned the scar in her
formal typewritten report.
法廷証言に対し異なる発言をしていたという弾劾に対し、証言と整合する別の供述
を証拠として提出する場合
(c) To rebut a contention of sensory deficiency:
Example: A witness is impeached with a contention of faulty memory based
on the fact that a year after the accident about which she testified, she
underwent treatment for a mental disorder. For rebuttal, it could be shown
that she gave the same account of the accident to the police shortly after the
accident happened.
事故で記憶障害を起こしているという弾劾に対し、その前に同じ証言をしていると

46
Evidence 2016 Summer BarBri NY Bar

反駁
(2) Purpose: A prior consistent statement that fits within the rule is admissible to
rehabilitate and as substantive evidence that the prior statement was true.
(Hearsay exception).
信憑性の回復及び実質的な証拠として証拠能力を有する(伝聞例外。VI.D.2.c 参照)。
HYPO 48. Brad v. Jennifer. On July 1, pedestrian Brad was struck by a car driven by Jennifer.
Angelina, a stranger to Brad and Jennifer at the time, witnessed the accident and told the police on July
1 that Brad looked sober as he crossed the street. At trial, six months later, Angelina testifies for Brad,
“He looked sober as he crossed the street.”
(a) On cross-examination, the only question Angelina is asked is whether she was convicted eight
years ago of income tax evasion, to which she answers “Yes.” On re-direct, may Angelina properly
testify that she told the police on July 1 that Brad had looked sober?
- No. Angelina is impeached by showing the bad character for veracity by income tax. But the
cross-examination does not suggest Angelina’s motive to lie. 嘘をつく動機が示唆されていないので
そもそも consistent prior statement を出すことはできない
(b) Assume that on the cross-examination of Angelina, she is asked, “Isn’t it a fact that after this
accident, you and Brad became close friends and are now living together as lovers?” to which she
answers, “Yes.” On re-direct, may Angelina properly testify that she told the police on July 1 that
Brad had looked sober? If so, for what purpose?
Yes. Angelina is impeached by showing bias. Prior consistent statement is admissible because it
was made before the motive to lie (bias) arose. Admissible both for rehabilitation and substantive
purposes. 事故の後で証人が被告人と一緒に住んでいるということであれば、嘘をつく動機があると
考えられる。その場合、証人に対する偏見が生じる。しかし、それよりも前の供述でその内容が現在
の供述と一致しているのであれば、以前の供述には信憑性があるといえる。

Emanuel-30 当事者が申請した場合、裁判所は他の証人の証言を聞くことを防ぐため、証人に対し退廷を命じ
ることができる。既に証言済みの証人に関しても、事後他の証人に対する反対証人として証言が行われる可
能性があるため、かかる退廷命令は適用される。
BarBri Set 1-16 証人尋問中に証人が質問されていない事項に回答した場合(unresponsive answer)
examining counsel は motion to strike ができるが、opposing counsel は motion to strike ができない。

47
Evidence 2016 Summer BarBri NY Bar

V. PRIVILEGES 特権
A. INTRODUCTION: (PURPOSES ARE TO FOSTER BENEFICIAL RELATIONSHIP.)
1. In general, on MULTISTATE exam, apply basic rules on privileges as covered in lecture.
2. Federal procedure issue on MULTISTATE exam: If bar examiners specifically indicate the
action is pending in federal court, apply the following procedural rules:
a) In a federal-court action arising under federal substantive law (all civil cases arising
under Constitution or federal statutes, and all criminal cases): “privileges are
governed by the principles of the common law as they may be interpreted by the
federal courts in the light of reason and experience.” For the most part, these are the
basic rules on privileges as covered in lecture.
b) In a federal-court action based on diversity jurisdiction, where state substantive law
applies to parties’ claims and defenses (Erie situation), the federal court must apply
privilege law of the state whose substantive law is applicable.

Note: In diversity actions, federal courts also apply state law on competency (e.g.,
Dead Man’s Statutes) and state law on burdens of proof and presumptions. Aside
from these three exceptions (privileges, competency, and burdens of proof /
presumptions), FRE apply in all federal-court actions, including diversity cases.
連邦裁判所で行う州籍相違事件(州法が適用)でも連邦法上の証拠原則が適用。但し、証明責
任・推定、特権、証人の適格性については州法が適用される。
(連邦裁判所の事件の中の、州籍相違事件の中の、証明責任/推定・特権・証人の適格性という
限定された分野においてのみ州法の証拠法が適用されるが、それ以外はすべて連邦法の証拠方 、
ということ。)
BarBri Set 2-17 第三者に聞かれていることを会話の当事者がわかっている場合には privilege が
失われるが、盗み聞きされていたにすぎない場合には privilege は失われない。

3. Substance: Recognized Privileges


Exam で問われるのは以下の 3 つのみ
1) Attorney - Client 弁護士と依頼者
2) Physician - Patient 医者と患者
3) Husband - Wife (Spousal privilege) 配偶者間
BarBri Set 3-1 Clergy-penitent privilege(聖職者の特権)というものも存在する基本的には弁護
士と依頼者の特権と同様。

B. ATTORNEY-CLIENT PRIVILEGE
4. Definitions:
a) Confidential communication
b) between an attorney and client (or their representatives)
c) made during professional legal consultation will be privileged
d) unless the privilege is waived by the client,
e) or unless an exception applies.
Client が放棄するか例外に該当する場合でなければ、弁護士と Client の間の Communication は保護される。

5. Rationale:
To encourage client to speak openly with the attorney
6. Elements:
a) Confidential:
Client must have reasonable expectation of confidentiality (e.g., (i) no privilege if
client knows that third party is listening in; or (ii) if client asks attorney to disclose the
communication to a third party). 依頼者が秘密保持を合理的に期待していること。(i)第三者
が聞いていることを依頼者が知っている場合、又は(ii) 依頼者が弁護士に対し第三者に対して
情報を開示することを要求している場合は含まれない。

48
Evidence 2016 Summer BarBri NY Bar

Joint client rule: If two or more clients with common interest consult the same
attorney, their communications with counsel concerning the common interest are
privileged as to third parties. But if the joint clients later have dispute with each other
concerning the common interest, privilege does not apply as between them.
共同で相談しにいった場合、第三者に対しては特権があるが、共同依頼者間の紛争が生じたと
きは特権を主張できない。
PS-84 Client が逮捕後に警察へのパトカーの中で弁護士に電話で連絡して相談している際に、
同乗していた警官がその内容を聞いた場合、弁護士・依頼者特権で保護される内容でなくな
り、警官の証言は証拠能力が認められる。

b) Communications: Privilege applies only to communications themselves, not to


underlying information, pre-existing documents, or physical evidence.
背景事情、既存の書面、物理的証拠は含まれない。
BIM-62 弁護士が依頼者と会ったときに依頼者を観察して知見された弁護士の感想は、保護
されるべき通信には当たらない。

c) Attorney—member of the bar or person that client reasonably believes is a member


of the bar, and it includes:
Representative of the attorney 弁護士の代理人・被用者
Any agent reasonably necessary to facilitate the provision of legal services (e.g.,
accountant working with attorney to “translate” client’s financial matters, secretary)
法的サービスの提供に合理的に必要となる代理人・被用者(秘書、弁護士と協働する会計士)
BIM-23 医者が代理人である弁護士の指示により依頼者を診療した場合、弁護士・依頼者間
特権は医者に適用される。この場合、医者は弁護士を代理していると判断されるからであ
る。この場合、医者と依頼者(患者)間の特権は問題とならない。
PR-168 弁護士が弁護活動のために医者を雇って依頼者が医者の診断を受けた場合には、依
頼者と医者の間の通信は Attorney-Client Privilege として保護される。
Emanuel-27 弁護士が雇った探偵についても同様。

d) Client—includes person seeking to become client (e.g., privilege attaches at outset of


formal consultation with attorney even if client does not retain attorney), and includes:
依頼者には依頼者になろうとして相談する者も含む。最終的に弁護士を雇う必要はない。
 Representative of client—any agent reasonably necessary to facilitate the
provision of legal services (e.g., for corporate client, any employee who
communicates with corporation’s attorney to enable attorney to provide legal
services to the corporation) 法的サービスの提供に合理的に必要となる代理人(顧問弁
護士と連絡を取る従業員など)
e) Professional legal consultation: The primary purpose of the communication must be
to obtain or render legal advice, not business or social advice 経営上のアドバイスや社会
生活上のアドバイスでは×。法的アドバイスを求めるものでなくてはならない。

49
Evidence 2016 Summer BarBri NY Bar

7. Waiver
a) Voluntary Waiver
Only the client has the power to waive the privilege. After the client’s death, the
privilege continues and only the client’s estate can waive it.
依頼者だけが特権を放棄できる。弁護士と依頼者の関係が終了しても存続する。 依頼者が死亡
した場合、依頼者の遺産に帰属する。
b) Subject Matter Waiver
A voluntary waiver of the privilege as to some communications will also waive the
privilege as to other communications if:
(a) The partial disclosure is intentional,
(b) The disclosed and undisclosed communications concern the same subject matter,
and
(c) fairness requires that the disclosed and undisclosed communications be considered
together.
同一の契約に関する書類が 3 通あって、そのうち 1 通のみを開示するのでは誤った印象を与え
るような場合→他の証書類についても waiver したことになる。

c) Inadvertent Waiver 意図せざる開示→waive したことにはならない


An inadvertent disclosure of a privileged communication will not waive the privilege
so long as the privilege-holder:
(a) took reasonable steps to prevent the disclosure, and
(b) takes reasonable steps to correct the error.

8. Exception
a) Future crime or fraud [if purpose of communication is to facilitate them]
将来の犯罪又は詐欺(目的が犯罪又は詐欺をするためである場合)
E.g., client tells attorney, “Help me disguise the bribes I made so that they look like legitimate business
expenses.”

b) Client puts the legal advice in issue (typically white collar crimes)
依頼者が法的アドバイスを争点にした場合
E.g., in tax fraud prosecution, defendant defends on ground that she relied on advice of her attorney in
reporting income.

c) Attorney – client dispute 弁護士と依頼者間の紛争


E.g., client sues attorney for malpractice; attorney sues client for unpaid fees.
HYPO 49. Delbert is sued for his alleged negligence in an auto accident. He tells his attorney what
happened and gives her the cell phone with which he was making a call at the time of the accident.
Before trial, Delbert is deposed by plaintiff’s counsel:
(a). Must Delbert respond if asked, “What did you tell your attorney about the accident?”
- No. That’s privileged. It’s a communication between attorney and client, is confidential and
made to obtain legal advice.
(b). Must Delbert respond if asked, “Describe what you were doing at the time of the accident?”
- Yes. That fact is not protected by the attorney-client privilege because the fact itself is not a
communication.
(c). If served with a subpoena, must Delbert’s attorney produce Delbert’s cell phone?
- Yes. Cell phone is not a communication and the attorney must produce it.

50
Evidence 2016 Summer BarBri NY Bar

C. PHYSICIAN-PATIENT PRIVILEGE 医者と患者の特権


9. Rule:
Confidential communication or information acquired by physician from patient for the
purpose of diagnosis or treatment of a medical condition is privileged.
診療又は治療のために医師が患者から取得した情報又は秘密の通信は特権の対象となる。
Also applicable to psychotherapists (M.D. or other professional certified to diagnose or
treat mental / emotional illness). e.g., social worker, psychologist 心理学者.
Note: Federal law distinction: privilege exists only for psychotherapists.
10. Elements:
a) Applies to relationship between doctor and patient (and typically includes therapists,
nurses, and physician’s assistants; 医者(療法士、医者のアシスタントも含む)と患者
BIM-10 Licensed Doctor であることが必要。診療前に医師資格を剥奪されていることを患者が
知っている場合には、特権は認められない。

b) Covers communications and information acquired by the doctor,


患者との通信及び医者が得た情報が対象。
c) So long as it is confidential and 秘密情報であること
PR-28 患者と医者以外の第三者が ease dropping 立ち聞きによってやり取りを聞いていた場合
には、秘密として保護されない。

d) For the purpose of medical diagnosis / treatment


医療的診断・治療を目的とした情報であること。
医者・患者の特権が適用される内容であり、患者により特権が放棄されたとしても、患者の医
者に対する発言は原則としては Hearsay であるため、証拠能力を認めるためには、Hearsay の
例外 9 の Statement for purpose of medical treatment or diagnosis 医学的治療、診療目的の供述に該
当することが必要。
BIM-10 医者と患者が結婚していてもこの特権との関係では無関係。配偶者間の特権の問題と
はなりうる。
BIM-23 患者が医者に対して、傷害事件のときに過失があったことを話したとしても、患者と
医者間での特権は医療的・診断治療を目的とした内容しか保護されないので、特権の主張は
できない。
BIM-29 保険加入目的で医者に見てもらうのは、diagnosis 又は treatment 目的であるとはいえな
い。

11. Rationale: To encourage candor and to protect privacy.


12. Losing the privilege:
The privilege will be waived if the patient expressly or impliedly puts physical or mental
condition in issue (e.g., plaintiff in a personal injury case, or defendant asserting an
insanity defense). 特権を有する当事者が明示的に又は黙示的に物理的又は精神的状況を問題とし
た場合には放棄される。たとえば傷害事件の原告や、心神喪失の抗弁を主張する被告など。
HYPO 50. Patient’s lung is being examined by physician in hospital room while visitor is present. (1)
Patient tells doctor, “Do you suppose my wheezing[ぜーぜー言うこと] is due to the three packs of
cigarettes I smoke per day?” (2) After visitor leaves, patient tells doctor, “Know any good lawyers? I
haven’t paid my income taxes in three years.”
(a). In state court action in which condition of patient’s lungs is an issue, could doctor be compelled to
disclose statement (1)?
- Yes. Statement (1) is not privileged because it’s not confidential. Third party in the room makes
the communication not confidential. Also attorney-client privilege will be destroyed by existence
of a third party but not by existence of secretary.
(b). In prosecution for income tax evasion, could doctor be compelled to disclose statement (2)?
- Yes. This is not for the purpose of medical diagnosis/treatment.

D. SPOUSAL PRIVILEGES 配偶者間の特権


13. Spousal Immunity (Spousal Testimony Privilege)

51
Evidence 2016 Summer BarBri NY Bar

a) The Rule: In a criminal case, the prosecution cannot compel the defendant’s spouse
to testify about anything against the defendant.
刑事事件で被告人の配偶者に証言を強制できない。
b) Rationale: To protect marital harmony. This is not to protect communication.
通信内容保護目的でなく、婚姻自体を保護する目的
c) Holder of the Privilege: Witness spouse (not defendant spouse)
d) ELEMENTS:
1) Applies only to criminal cases 刑事事件のみ
2) Covers testimony against a spouse 配偶者に対する証言
3) So long as witness and defendant are currently married. 証言時に婚姻中
4) May be waived by the witness-spouse. (Defendant-spouse does not have a power
to keep the witness-spouse from testifying.)
被告人の権利ではない!→証人となっている配偶者のみが放棄できる(被告人となってい
る配偶者は証人となる配偶者が証言することを禁止する権限はない)
BIM-31 刑事事件の被告人の配偶者は婚姻中に証言を拒絶でき、すべての情報について拒絶で
きる。すなわち、婚姻前の二人の間での会話も拒絶の対象となる。
BIM-72 証言を強制させることはできないが、証人としての呼出自体は可能。

14. Spousal Communication Privilege 配偶者間通信の特権


a) THE RULE: Confidential communications between spouses will be privileged
b) Rationale: To encourage candor between spouses.
c) Holders: Both spouses (i.e., either spouses can invoke the privilege)
d) ELEMENTS:
1) Applies to relationship between married spouses 夫婦が婚姻中に行った通信
2) Covers communications, 通信のみ
PR-119 Information secured by one spouse through observation during the marriage as to the
health, or intoxication, habitual or at a particular time, or the mental condition of the other spouse
would be protected by the privilege.
健康状態、酩酊、習慣、精神状態も Communication に含まれる。

3) So long as they are confidential. 秘密性


Emanuel-5 第三者(例えばナース)が存在する場合は秘密の通信に該当しない。

4) May be waived only by both spouses. 放棄には夫婦の双方が同意する必要がある。


BIM-12 後に離婚したとしても、婚姻中に行った通信に対する特権は消滅しない。いずれの
当事者も(i)夫婦間通信の内容を証言するように強制されず、(ii)他の当事者による開示を防
ぐことができる。
BIM-31 この特権は夫婦のいずれかが事件の被告となっている必要はなく、第三者の事件に
夫婦の一方が証人として呼び出された場合にも適用される。
PR-75 夫婦間の通信が、全部又は一部について、犯罪又は詐欺を実行する計画又は犯罪を
実行しようとする者(誰でもよい)を幇助し、容易にする場合には、特権は適用されな
い。
Common Law Spouses(内縁)の間での通信はこの特権による保護を受けない。
BarBri Set 5-18 夫婦の一方が死亡し、他方は放棄した(e.g.友人に話した)場合にはもはや
privilege ではない。

15. Exceptions (applicable to both privileges).


Communication or acts:
a) In furtherance of future crime or fraud (e.g., joint criminal activity)
将来の犯罪(共同犯罪行為)
b) destructive of family unit (e.g., spousal or child abuse)
家庭生活の崩壊(DV 又は児童虐待)

52
Evidence 2016 Summer BarBri NY Bar

c) in litigation between the spouses themselves (e.g., divorce, breach of contract)


夫婦間の訴訟
HYPO 51. Niles is prosecuted for the murder of his brother Frazier. Niles and Daphne are a married
couple. Niles comes home on the night of Frazier’s demise wearing a blood-stained Armani topcoat,
which Daphne observed.
(a). At trial, the prosecutor calls Daphne to the stand to testify to her observations about Niles’ topcoat,
but she refuses to testify. The prosecutor seeks to compel her testimony.
- No. She can refuse to testify about communication or non-communication fact under the spousal
immunity.
(b). Assume Daphne is willing to testify against Niles and seeks to testify to the following: “Niles told
me when he got home that he stabbed Frazier for insulting the music of Willie Nelson.” Does Niles
have a valid objection?
- Yes. Spousal immunity does not apply because Daphne is willing to testify. However, this is a
confidential communication between spouses and the spousal communication privilege applies. In
order to waive the privilege, both spouses must waive simultaneously.
HYPO 52. Assume that Daphne divorces Niles before his case goes to trial. The prosecutor calls her to
the stand.
(a) Can Daphne be compelled to testify to her observations about Niles’s topcoat?
- Yes. No privilege here. Topcoat is a physical evidence and thus, spousal communication privilege
does not apply. Spousal immunity does not apply because they are divorced when Daphne is to
testify.
(b) Can Niles prevent Daphne from disclosing his admission to her about stabbing Frazier?
- Yes, he can. That admission is still protected by the spousal communication privilege because it
was made when they are married. Whether they later got divorced does not matter. To protect
communication between spouses

16. Review and Summary: Spousal Privileges


Type of case it Married at the May be
Privilege Protects
applies in time of the … waived by
Spousal Only by both
Any case Confidential
Communication Communication spouses
(Civil/Criminal) communications
together
Witness
Spousal Testimony Only criminal Testimony about
Testimony spouse on
(Immunity) case anything
her own

BarBri Set 6-9 その他の privilege:self-incrimination


証言によって証人が刑事責任を問われる reasonable possibility (×convinced)がある場合には証言拒絶できる。Jury ではな
く、judge が決める。詳細は Criminal Procedure のノートを参照。

53
Evidence 2016 Summer BarBri NY Bar

VI. HEARSAY 伝聞証拠


A. DEFINING HEARSAY (FRE 801)
1. THE DEFINITION: Hearsay is:
a) an out-of-court statement of a person (called a declarant) (oral or written)
法廷外の供述(口頭でも書面でもよい、non-verbal conduct も含む)not animal, not machine
BarBri Essay-5 Statement = verbal or non-verbal conduct intended as assertion
そのため、Thumb up も statement になり得る。
他方、単なる質問は hearsay が問題になる statement ではない(assertion がないから)。
似顔絵も statement になる。
BarBri Set 3-3 Tally Record(計算表)も statement に該当する。

b) offered to prove the truth of the matter asserted in the statement.


供述中で主張される事項の真実性を証明するために提出される
BIM-74 専門家の作成した報告書も、意見のみを含むもの(事件の対象物の価格の鑑定な
ど)であっても伝聞証拠である。
BarBri WS-18 Federal Rule 106 provides that, when a statement or part of statement is introduced,
the adverse party may introduce any other statement or part of statement which ought, in fairness, to
be considered at the same time.

2. THE RULE:
Absent an exception or exclusion, hearsay is inadmissible.
例外又は除外事由ない限り証拠能力なし。
3. THE RATIONALE:
The credibility (perception, memory, sincerity) of the declarant (out-of-court speaker or
author) at the time the statement was made cannot be tested through cross-examination in
the presence of the current fact-finder. 供述者の信用性が反対尋問でテストされない。
B. NON-HEARSAY STATEMENTS 非伝聞証拠
1. PURPOSE is key:
Whether a statement is or is not hearsay will depend upon the purpose for which it is
offered. Some out-of-court statements may look like hearsay at first glance, but are not
hearsay if they are not offered to prove the truth of the matter asserted in the statement .
供述が伝聞証拠かどうかは、提出される目的による。法廷外の供述が一見伝聞証拠に見えても、 供
述中で主張される事項の真実性を証明するために提出されるのでなければ、伝聞証拠ではない。
2. Hearsay Tip:
Ask yourself, “Do we care whether or not the declarant is telling the truth?” If the answer
is “no,” then the statement is not hearsay (nor offered for the truth). 供述者が真実を言ったか
どうかが問題となる場合は伝聞証拠。そうでなければ伝聞証拠ではない。
HYPO 53. Action by the estate of Percy against Damien seeking damages for the pain and suffering
Percy experienced in an auto accident caused by Damien. Damien denies liability and also asserts that
Percy died instantly in the accident. Witness on the stand proposes to testify for Percy’s estate that
shortly after the accident, Percy said, “Damien’s car ran the red light.” 交通事故に基づく P の D に対する
請求。D は責任を否定し、P は即死したと供述。証人は P の遺産の管理人のために、P が事故の直後に「D
の車が赤信号なのに渡ってきた。」といっていたことを供述。
(a). Is it hearsay if offered to prove who ran the red light?
- Yes. Declarant is Percy. This is an out of court statement by a person and it is to prove the truth of the
statement.「赤で渡った」事を証明しようとしている。
(b). Is it hearsay if offered to prove that Percy was alive following the accident?
- No. Whether Percy was alive after the accident is related whether he suffered pain. This is not offered
to prove the truth of the statement. Whether he was lying or not does not matter. What matters is
whether he could speak or not. 「Percy が事故直後に生きていた」事の証明目的であれば、伝聞
証拠ではない。

C. THE 3 PRINCIPAL CATEGORIES OF NON-HEARSAY PURPOSES


非伝聞証拠となる立証目的 3 つの類型

54
Evidence 2016 Summer BarBri NY Bar

3. Verbal Acts (Legally Operative Words 法的効力発生文言)


a) THE RULE: If certain words have “independent legal significance” – in other
words, if the law attaches rights and obligations to certain words simply because they
are said – then they will not be hearsay.
言葉に「独立した法的重要性」が含まれている場合=その言葉が発せられたことのみをもって
法律上権利又は義務がその言葉に付与される場合は伝聞証拠ではない。
b) Examples:
(1) words of offer, repudiation or cancellation of contract;
契約の申込、履行拒絶、解約の文言
(2) words that have the effect of making a gift or a bribe; 贈与もしくは贈賄
(3) words that are themselves an act of perjury, a criminal misrepresentation or a
defamation. 偽証、犯罪上の不実表示又は名誉毀損
HYPO 54. Gates sued Trump for breach of an oral contract. Witness called by Gates proposes to
testify as follows: “I heard Trump say to Gates: ‘I accept your offer to sell Microsoft.’” Hearsay?
- No. Not hearsay because it is called verbal act, of independent legal significance. “I accept” are
legally operative words. 「了承した」は法的効力発生文言であるので、伝聞証拠ではない。

4. To Show Effect on Person Who Heard or Read the Statement


供述を聞き、又は読んだ者に対する効果を証明するためのもの
a) THE RULE: A statement that is relevant simply because someone heard it (or read
it) is not hearsay.
b) Examples: Hearing something can put someone on notice or can give someone a
motive or can make someone’s belief reasonable.
HYPO 55. Plaintiff v. Supermarket. Plaintiff alleges she slipped and fell on broken jar of salsa in aisle
3 and that Supermarket had prior notice of the dangerous condition. Plaintiff’s witness takes stand and
proposes to testify: “Several minutes before Plaintiff entered aisle 3, I heard another shopper tell
Supermarket manager, ‘There’s a broken jar of salsa on the floor in aisle 3.’” Inadmissible hearsay?
- It depends on the purpose for which it was offered. If this is offered to prove the truth of the assertion,
then it’s hearsay and inadmissible. However, if this is to prove the notice of the supermarket, then it’s
not hearsay. Whether the assertion is true or false does not matter but what matters is whether the
manager heard it or not. 目的による。供述内容の真実性立証のためであれば、伝聞証拠となり証拠能力な
し。しかし、注意がなされていたこと(したがってスーパーは修理すべきであった)の立証である場合に
は伝聞証拠ではなく、証拠能力あり。供述内容が真実かどうかは関係ないが、マネージャーがそれを聞い
たかどうかが問題となる

HYPO 56. Sybil is charged with the murder of her husband Basil. The prosecutor seeks to introduce an
anonymous note to Sybil that was found in her possession at the time of her arrest. The note stated,
“Basil is having an affair with Polly.” S は夫 B の殺人で起訴。検察官は、S が逮捕のときに保有していた
匿名のノートを証拠として提出。ノートには「B は P と関係があった」との記載。
(a). If the prosecution offers the note to prove motive, is it hearsay?
- No. If the purpose is to prove a motive, it’s not hearsay. We do not care about who wrote it. 動機
を立証するために検察官が提出する場合には、Hearsay とはならない。誰が書いたかは関係がない。
記載自体が真実かどうかについて立証するわけではない。
(b). If the prosecution offers the note to prove that Basil and Polly were having an affair, it is hearsay?
- Yes. Now it matters whether the statement was true or not. 記載自体が正しいことを立証する場合
には Hearsay に該当する。実際に正しいかどうかは関係がない。

5. Circumstantial Evidence of Speaker’s State of Mind 供述者の心理的状態の状況証拠


下記 F.7 の Statement of then existing mental との違いに留意。

a) THE RULE: A statement that unintentionally reveals something about the speaker’s
state of mind is not hearsay. 供述者の心理的状態が意図せずして明らかになる供述は伝聞証
拠ではない。精神異常者と思われるような発言。
b) Examples: Statements demonstrating insanity, lies that demonstrate a consciousness
of guilt, questions that demonstrate a lack of knowledge.
心神喪失を証明する供述、有罪の認識を証明する虚偽、知識の欠如を証明する質問
HYPO 56. Homer is prosecuted for murder. Defense: Insanity. Witness for Homer proposes to testify:
“Two days before the killing, Homer said, ‘I am Elvis Presley. It’s good to be back.’”

55
Evidence 2016 Summer BarBri NY Bar

- No. Obviously it is not offered to prove the truth but it is offered to prove Homer’s state of mind in his
insanity defense.
e.g., 有罪認識:犯行後に被告人が虚偽のアリバイを述べたという事実は、良心の呵責を示す
情況証拠であるから、これも発言の存在を発言者の心理状態の立証ための情況証拠として用
いる場合として許容される(False alibi! - Circumstantial evidence of guilty consciousness.)。
e.g., 知識の欠如:(ある人の司法試験についての知識が問題となっている場合に、その人
が)「司法試験の勉強はとても楽しいものなのか」と尋ねたということについての証拠は、
そのような質問をすること自体が主題(この場合は司法試験)についての知識の欠如を示し
ており、発言の存在を発言者の心理状態の立証ための情況証拠として用いる場合として許容
される(Question showing lack of knowledge-“Is studying for bar a lot of fun?”)。

B. PRIOR STATEMENTS OF TRIAL WITNESS 公判証人の以前の供述 (EXCLUSION FROM HEARSAY)

1. The Rule:
A witness’s own prior statement, if offered to prove the truth of the matter asserted in the
statement, is hearsay and is inadmissible unless an exception or exclusion applies.
供述内容の真実性を証明するために提出される場合、証人の以前の供述は伝聞証拠となり、証拠能
力なし。(Prior Inconsistent Statement と Prior Consistent Statement の双方を含むことに注意)

2. The Three Exceptions (exclusions: from definition of hearsay):


Prior statements of witnesses that are “excluded” from the definition of hearsay:
法律上伝聞証拠の定義から除外される 3 つの場合

a) Prior statement of identification 同一性に関する以前の供述


同一性に関する以前の供述は、実質的な証拠として証拠能力を有する。Cf. IV.G.2
但し、証人が出廷し、反対尋問を受けることが要件として必要である。
BSM-121 同一性に関する以前の供述が実質的な証拠能力を有するとしても、同一性について
の供述をした本人が供述不能で法廷に出廷せずに、その供述を聞いた警察官を証人として申
請して、供述者の供述を証言する場合は、Hearsay の例外に該当せず、証拠能力を有しない。

b) Prior inconsistent statement, Cf. IV.H.1. 以前の不一致の供述


(i) For impeachment: (inconsistency を証明する目的であれば制限なく利用可)
(ii) For substantive evidence if:
(1) made under oath, and
(2) during a formal trial, hearing, proceeding or deposition
宣誓のもと、正式な手続きの下で作成された場合には実質証拠としても利用可
BIM-1 Grand Jury 起訴陪審での供述も含む(former testimony との違いに留意)。

c) Prior consistent statement, if 以前の一致する供述 cf. IV.I.


(1) used to rebut an accusation of motive to lie.
(2) and made before the motive to lie arose.
嘘の動機の追及に反論するため、嘘の動機が発生する前になされたものを利用
PB-50,51 これら 3 つの例外に該当するには、供述を行った者が、実際に公判において証言を行う
必要がある(出廷のみでは不十分)。公判において証言を行わずに、以前の供述だけを提出するのは
認められない。以前の供述だけを提出すると Hearsay として証拠能力は認められない。
BIM-2 以前の Grand Jury での供述は、供述者が、当該供述が提出される公判において供述を行う
場合に例外として Non-Hearsay として証拠能力が認められることがある。証言も行わずに、単に以
前の Grand Jury での供述(記録を書面で提出)だけを提出するのは認められない。以前の供述は
現在の供述と比較するところに意味がある。
HYPO 58. Prosecution of D for robbery. D takes the stand in his own defense and testifies: (a) “I
didn’t do it. (b) And I told the cops when they arrested me that I didn’t do it.” Should (a) and (b) be
excluded as hearsay?
(a): No. It’s not an out-of-court statement.
(b): Yes. It fits the definition. It does not matter the declarant is the same person as the witness.
供述者が証人と同一人物かどうかは問題とならない。

C. PARTY ADMISSIONS (STATEMENT OF AN OPPOSING PARTY) 当 事 者 の 承 認 (FRE


801(D))

56
Evidence 2016 Summer BarBri NY Bar

1. THE RULE:
Any statement made by a party is admissible if it is offered against the party. (i.e., offered
by the other side). 当事者によってなされた如何なる供述も、当該当事者に対する証拠とする場合
には、証拠能力あり。
* 明確に against interest な内容の発言である必要はない( statement against interest)。相手方は訴訟当
事者のいかなる発言も攻撃証拠として提出できる。
* personal knowledge がなくて発言した当事者の発言でも、party admission として認められる。
PR-124 ある事件における当事者の訴答書面での供述は、最終のものか、後に撤回される
か、修正されるか、覆されるかを問わず、他のいかなる訴訟においても当事者の不利益とな
る供述として証拠能力が認められ、供述者に対して不利に利用される。
電話の相手が名乗らなかった場合でも、声から発話者が誰か特定できるのであれば party
admission になる。
下記例外 10 の Business Record での事業上の報告でも、会社にとって不利になるような情報が記載さ
れることもあり、この場合 Party Admission があるとして証拠能力を認められる。Business Record とし
て認められる 5 要件の充足を主張するよりは当事者の承認の方が立証は容易。

2. Terminology:
Under the FRE, party admissions are called “exclusions” or “not hearsay” because they are
statutorily “excluded” from the hearsay definition.
連邦法上は「伝聞証拠の定義外」そもそも Hearsay ではない。
BarBri Set 4-9 など MBE 上は Non Hearsay として扱う(admissible hearsay exception ではない)。

3. Rationale:
“You say it, you’re stuck with it” (estoppel, not reliability 信頼性ではなく、禁反言).
BIM-3 Actual Plea of Guilty も、反対当事者の承認に基づき、証拠能力が認められる。ただし、
被告はこれに対して説明を行うことができる。
BIM-52 Silence によっても、合理的な人間であれば追及に対して反論したであろうという場合
には、当事者による承認とみなされる。
PS-39 Threats by a defendant against testifying witness either before or during a trial are relevant
evidence to show consciousness and are admissible as an admission. 被告による証人に対する脅迫が
公判前又は公判中に行われた場合、罪の認識を示すための証拠として関連性があり、当事者
による証人として伝聞証拠の例外となって証拠能力を有する。

4. Adoptive Admission
If a party expressly or impliedly adopts a statement made by another person, it is as though
the party herself made the statement. 自己に不利な他人の発言を明示又は黙示に認めた場合
Adoption by silence occurs when a party who hears another person’s statement remains
silent under circumstances in which a reasonable person would protest if the statement
were false.
Emanuel-22 警察での事情聴取で、共犯者が自らの犯行を認めて供述していたとしても、同席
していた被告人がかかる供述を adopt したとみなすことはできない(警察の前では黙秘したい
と考えることも自然)。
Emanuel-24 覚せい剤のディーラーが、被告人のことを「覚せい剤の販売のパートナーだ」と
顧客役のおとり捜査官に紹介し、被告人がおとり捜査官と黙って握手した場合は、 adoption が
認められる。

Emanuel 故人の妻が原告、故人の同僚が被告となる、故人を殺したことについての損害賠償
請求で、原告は別の同僚を証人として呼び、故人と被告の仲が悪かったことを証言させたが
これに被告は異議を唱えなかった。しかし、証人が「故人は被告がやったと言った。ちょう
どそのときに被告が店の中にいることに私は気付いた」と証言。これに対して。被告は異議
を唱えた。認められるか?→No. Admission by silence.

5. Vicarious Admission 代位承認


(1) THE RULE:
A statement
(a) By an agent or employee of a party,
(b) Is admissible against the party (principal or employer)
(c) If it concerns a matter within the scope of the agency or employment

57
Evidence 2016 Summer BarBri NY Bar

(d) And was made during the agency or employment


(a) 代理人・従業員による (b) 当事者(本人・雇用者)に不利益となる (c)代理・業務の範囲
内の事項に関する供述であって、(d) 代理・業務の期間中において行われたもの

(2) Vicarious Admissions by Co-Conspirators 共謀者による代位承認 : A statement


of one co-conspirator is admissible against other co-conspirators if the statement
was made during and in furtherance of the conspiracy. 他の共謀者に不利な共謀者
の供述は共謀中に、かつ共謀を進めるために行われた場合には証拠能力あり。
HYPO 59. X is charged with income tax evasion for the year 2000. Prosecutor wants to prove X’s
income during 2000, and offers into evidence a loan application X submitted to a bank in that year. X
objects on the ground that the loan application, which is filled with inflated numbers, was self-serving
and unreliable.
- Even if it was false, the loan application is admissible against X as a party admission. It does not
matter it’s reliable not or it’s true or false. It is statement of opponent party rather than admission. It’s
not always admission against its interests. Anything which can be used against the opponent can be
party admissions.
HYPO 60. Ma v. Life Insurance Co. for non-payment of policy proceeds on the life of Pa. Defense:
Suicide. Defendant offers a letter by Ma to her friend in which she wrote, “When I came home from
shopping I found Pa dead on the floor with his revolver nearby. I didn’t see what happened, but this was
no accident. Pa did himself in.” Admissible despite Ma’s lack of personal knowledge?
- Yes. Even though she has no personal knowledge thus otherwise inadmissible, it is admissible as a
party admission. Reliability does not matter. Personal Knowledge は Party’s Admission には不要。
HYPO 61. Charlie the truck driver smashed into Pam’s house while on a run for Acme Trucking, his
employer. Charlie descended from the cab and calmly told Pam, “Sorry about wrecking your home. I
had my head down looking for a joint and a beer and wasn’t paying attention to where I was going.” In
Pam v. Acme, is Charlie’s statement admissible against Acme?
- Yes. Charlie was an employee and he was speaking about his driving which is within the scope of the
employment.
HYPO 62. Thelma and Louise v. Acme Trucking for sex discrimination in failing to hire them. They
offer the statement of Charlie, an Acme truck driver, who told them over drinks one night, “I know the
Acme personnel office has a policy against hiring women no matter how qualified they are.” Charlie’s
statement is inadmissible because:
(A) Charlie was not on the job when he was speaking to Thelma and Louise.
(B) Charlie’s statement did not concern a matter within the scope of his employment.
(B) is correct. (A) is incorrect because there is no requirement of “on the job” ( 勤務関係がある時期であ
ることは必要だが、勤務中である必要はない)

58
Evidence 2016 Summer BarBri NY Bar

D. HEARSAY EXCEPTIONS TO KNOW FOR THE BAR EXAM【伝聞例外】


1 Forfeiture by wrongdoing 違法行為による喪失
2 Former testimony 以前の証言
declarant unavailable
3 Statement against declarant’s interest 供述者が供述不能な場合
供述者の利益に反する供述
4 Dying declaration 臨終供述
5 Excited utterance 興奮による発声
6 Present sense impression
当該時点での認識による印象
7 Statement of then-existing mental, emotional, anyone,
intent, or physical condition spontaneous statements
当時の精神的、感情的、意図、身体的状況の供述 誰でも、自然発生的供述
8 Statement for purpose of medical treatment
or diagnosis 医学的治療、診療目的の供述
9 Business and public records 事業及び公的記録

OTHER HEARSAY EXCEPTIONS


(a) Past Recollection Recorded (IV.D.2 参照)
(b) Learned Treatises (IV.E.2 参照)
(c) Judgment (Felony convictions)
(i) In BOTH criminal and civil cases, judgments of felony convictions are admissible
for proving any fact essential to the judgment.
(ii) Judgments of acquittal are inadmissible.

PS-86 “Catch all” or Residual exception to the hearsay rule 伝聞証拠ルールに対するキャッチオールの例外


In order to qualify for admission, a hearsay statement not falling within one of the recognized exceptions must
satisfy the following condition. どの例外にも該当しなかった伝聞証拠が、証拠能力を認められるためには、
以下の条件を満たすことが必要。
(1) It must be “equivalent circumstantial guarantees of trustworthiness” 信用性の状況的担保
Circumstantial Guarantees of Trustworthiness: Court will consider 裁判所は以下の事項を考慮
(i) whether the statement was under oath, 供述が宣誓の下になされているか。
(ii) the duration of the time lapse between the event and the statement 事実発生から供述迄に経過した時間
(iii) motivation to speak truthfully, and 真実を供述する動機
(iv) whether the declarant had firsthand knowledge. 供述者が直接得た知識かどうか。
(2) It must be offered as evidence of a material fact 重要事実の証拠として提出されなければならない。
(3) Court must determine that the statement “is more probative on the point for which it is offered than any other
evidence which the proponent can procure through reasonable efforts”.裁判所が、主張者が合理的な努力を通
じて提出しようとするその他証拠よりも、当該証拠の証拠力が高いと判断していなければならない。
PR-140 Catch-all による例外の example
大学の Transcript も信用性を保証する状況証拠が十分であれば、Hearsay の例外として Admissible とな
る。

59
Evidence 2016 Summer BarBri NY Bar

F-#1 DECLARANT UNAVAILABLE 供述不能


以下の 1.〜4.の類型はいずれも供述者の供述不能が要件となる類型。

Note: Unavailability 供述不能:


The hearsay exceptions for forfeiture by wrongdoing (1.), former testimony (2.),
declarations against interest (3.), and dying declarations (4.) require that the declarant be
“unavailable”.
Grounds of unavailability: “PAILS”
(a) P - Privilege 特権
(b) A - Absence from the jurisdiction 管轄権内に不在
(i) cannot be found with due diligence OR
(ii) beyond court’s subpoena power
(c) I - Illness or death 病気・死亡
(d) L - Lack of memory 記憶の欠如
(e) S - Stubborn refusal to testify 頑固な証言拒絶
(Simple refusal is not included. Witness is not allowed to testify.)
BarBri (Civil Procedure) Set 2-1 imprisonment の場合も unavailability の要件を満たす。

1. Forfeiture by Wrongdoing 違法行為による喪失(FRE 804)


THE RULE:
Any type of hearsay statement is admissible against a defendant whose wrongdoing
made the witness unavailable if the court finds
(1) by a preponderance of evidence 証拠の優越
(2) that defendant’s wrongdoing was designed to prevent witness from testifying.
被告人が意図的に証人の証言を阻害したこと
(By making the witness unavailable through his own wrongdoing, defendant forfeits
both the hearsay and Sixth Amendment objection.)
Example: Tony Soprano is prosecuted for loan-sharking. Paulie made incriminating
statements about Tony to the police and during a grand jury proceeding. Tony learned
that Paulie was going to be the key government witness at trial; and the next day
Paulie’s dead body was found in the river. The court determines by a preponderance
of the evidence that Tony arranged for Paulie’s disappearance in order to prevent
Paulie from testifying. Paulie’s grand jury testimony and interview statements to the
police are admissible against Tony even though the statements are testimonial, Paulie
is unavailable, and Tony had no opportunity for cross-examination.
2. Former Testimony in former proceedings 以前の証言(FRE 804)
Prior Statements of Trial Witness との違いに注意すること
a) ELEMENTS of the former testimony exception:
(1) The declarant is unavailable 供述不能
(2) The prior statement was given in a proceeding or deposition
以前の供述が公判手続または証言録取手続でなされた
(3) And is offered against a party who, in the prior occasion, had an opportunity and
a motive to cross-examine or to otherwise develop the testimony.
当時の状況下で、反対尋問または証言を行う機会及び類似の動機があった場合にのみ認め
られる。(trial testimony/deposition は認められるが、grand jury testimony は反対尋問の機会が
ないので NG)
反対尋問の機会とは、被告人の代理人の同席で足りる。被告人の同席は不要。

BarBri Set 5-15 当事者が異なる場合

60
Evidence 2016 Summer BarBri NY Bar

・新しく事件の当事者となった者には Former Statement の作成時に反対尋問の機会がな


く、Former Statement による Hearsay Exception は認められないのが原則。
(例)殺人事件に関し、先行する刑事事件で被告人側の証人が被告人のアリバイを証
言し無罪判決が出た場合に、これに続く被害者遺族による被告に対する民事事件で上
記証人の証言を former testimony として用いることはできない(∵刑事と民事は別事件
であり、民事の原告である被害者遺族は刑事事件の際に反対尋問を行使する機会を得
ていない)。
例外:民事事件の場合、第 1 事件の当事者と、第 2 事件の当事者に privity relationship
がある場合、第 2 事件の当事者自身が第 1 事件において反対尋問権を行使する機会が
なかったとしても反対尋問権の機会があったものとみなされる。
例:不動産の譲渡人と譲受人、life tenant と remainderman、joint tenant

(4) Issue in both proceedings must be essentially the same.

b) Rationale: Reliability is assured by the cross-examination on the prior occasion (so


long as the opposing party had a similar motive); however, we prefer live testimony,
so witness must now be unavailable.
以前の状況での反対尋問により信頼性が確認される。しかし、実際の証言のほうが好ましいの
で、現時点での供述不能を要する。
HYPO 63. Bus accident. Passengers A and B were seriously injured. A sued Bus Co., alleging
negligence by bus driver. At trial, Witness testified for A that bus driver was intoxicated at time of
accident. Thereafter, Witness died. B now sues Bus Co. and seeks to admit a transcript of Witness’s
former testimony.
- Yes. W is unavailable because he is dead. The prior statement was offered in a proceeding. Bus had
an opportunity to cross-exam W and had the same motive because the issues and Bus’s liabilities are
same in two proceedings. 当事者であるバス会社は反対尋問の機会があった(同じ被告で違う原告の事
例)。

HYPO 64. Same bus accident. At grand jury, Witness testified that bus driver was intoxicated at time
of accident. Thereafter, Witness died. Bus driver is prosecuted for DWI. Prosecutor seeks to admit a
transcript of Witness’s grand jury testimony. バスの事故で運転手が起訴された。公判手続で検察官が証
人の起訴陪審での証言を記載した書面を証拠として提出。証人は起訴陪審証言の中で運転手が事故の時点
で酩酊中であったことを証明することを意図している。しかし、証人が死亡。
- No. Even though W is unavailable and the issue is the same, D had no opportunity to cross-exam W in
a grand jury proceeding. A grand jury proceeding is secret and the defendant has no opportunity to
cross-exam the witness (only grand jury, witness and prosecutors are there). 証人が出廷不能で同じ問題
であっても、被告は証人の起訴陪審での証言には(在廷していないので)反対尋問をすることができない 。
起訴陪審手続は秘密で、被告は証人の反対尋問をする機会を全く与えられない。証人と検察官が大陪審に
いることを要する。→証拠能力なし。

c) Review and Summary of Former Testimony v. Prior Inconsistent Statement


Former Testimony Prior Inconsistent
Statement
(as substantial evidence)
When former Formal Proceeding Formal Proceeding
statement made? * Excluding grand jury * Including grand jury
Declarant’s Required NOT Required
Unavailability
Opportunity and Required NOT Required
motive of *criminal の 場 合 は 別 途
cross-examination confrontation clause の問題がある
Inconsistency NOT required Required
Same issue? Yes Yes
(∵ inconsistency requirement)
3. Statement Against Declarant’s Interest 供述者の利益に反する供述(FRE 804)
a) ELEMENTS:
(1) Declarant is unavailable. 供述不能
(2) Statement was against Declarant’s (i) pecuniary (money), (ii) proprietary

61
Evidence 2016 Summer BarBri NY Bar

(property), or (iii) penal (criminal liability) interest when made.


供述の時点で、供述者の金銭、財産、刑事責任についての利益に相反する供述
b) Rationale: Someone is not likely to lie when making a personally damaging
statement. 「自分の利益に反することを供述しようとする者は嘘をつかないはず」
*しかし、真意は異なるかもしれない。そのため、本人を連れてきた方が better であり、
unavailability が要件になっている。

c) Qualification in criminal cases 刑事事件の場合:


Statement against penal interest, when offered to inculpate a defendant, must be
supported by circumstances showing trustworthiness of the statement (i.e., need a
corroborating evidence). 被告人に刑事責任を課すために提出された供述は、補強証拠によっ
て補強されなければならない。
HYPO 65. Plaintiff v. Acme Trucking, based on Charlie the truck driver’s negligent driving. Charlie
was fired immediately after the accident. Two weeks later, Charlie told Plaintiff’s insurance adjuster
that he had been drunk while driving. At trial, Charlie refused to testify on the ground of self-
incrimination. Plaintiff offers the insurance adjuster’s testimony about Charlie’s statement as evidence
against Acme.
(a). Admissible as a vicarious party admission?
- No. When he made the statement, he was already fired (no longer employee), thus no vicarious
admission.
(b). Admission as a statement against interest?
- Yes. Charlie’s statement was against his pecuniary and penal interests. Charlie is unavailable because
of his privileges.
HYPO 66. State v. Doppler for arson of Town Hall. Doppler calls Waldo to testify that while sitting in
a bar one night recently, Waldo heard Stranger say, “I’m the guy who torched Town Hall, but I’m sure
glad they think it’s Doppler. Just to be safe, I’m leaving town tomorrow.” Doppler’s attorney
demonstrates that Stranger has not been located despite diligent efforts to find him.
- Not unless there is some evidence that corroborates the S’s statement.

d) REVIEW AND SUMMARY: Party Admissions vs. Statements Against Interest


Must Be Personal
Unavailabl Declarant Against? When? Knowledge
e? Required?
Party or an For agent, must be
Party
No agent of a Party an agent when No.
Admission
party made
Statement
Against Anyone Against interests
Yes Anyone Yes.
Declarant’s (Declarant) when made.
Interest
相手方当事者の発言の場合、party admission と statement against interest が両方成り立つことがあ
る。

62
Evidence 2016 Summer BarBri NY Bar

4. Dying Declaration 臨終供述(FRE 804)


a) ELEMENTS:
(1) Declarant is unavailable 供述不能
(2) Statement was made under a belief of certain and impending death.
特定の差し迫った死を信じた状態でなされていること
(3) Statement concerns the cause or circumstances of the declarant’s death.
供述者の死の原因または状況に関連する供述。
BIM-59 他人の犯罪についての供述は、死ぬ間際であっても証拠能力なし。
PR-40 供述者が死亡していなくても、供述不能であれば(e.g. 昏睡)、証拠能力あり。
BSM-189, BarBri Set 6-5 Firsthand Knowledge に基づかない単なる Opinion は死亡に関連す
るとしても「事実」や「状況」とはいえないので証拠能力なし。
被害者が実際に見たわけではないのに「被害者の車のブレーキを事前に壊しておいたの
は A に違いない」などといった発言。
BRQ-4 Dying Declaration をした者が死亡せずに後になって死亡の原因は違う者による刺
傷 で ある と供 述し た場 合、 それ 自 体 は Dying Declaration で はな いが 、最 初の Dying
Declaration は Impeachment として利用することができる。

b) Rationale: No one wants to die with a lie on his or her lips. 嘘をついて死にたくない。
c) Type of case:
(1) Civil: any case
(2) Criminal: Only homicide case
PR-54 Criminal Homicide Case には Attempt Murder 殺人未遂は含まれない。殺人が既遂に
なっていることが必要であり、被害者が危篤/昏睡でも生きていれば homicide case になら
ない。
HYPO 67. Prosecution of Dagger Dan for the murder of Victor Victim. A passerby found Victor lying
in the gutter in a pool of blood with a knife in his stomach. Victor told the passerby, “It’s not looking too
good for me. Dagger Dan did it, and I’m going to get him for this.” Victor died an hour later. May the
passerby testify to Victor’s statement as a dying declaration?
- No. There is no showing here that Victor believed his certain and impending death. Rather he showed
his hope for living. 生きようとしている場合は×
HYPO 68. Prosecution of Derringer for bank robbery. At the bank, Agent Malone spoke with
wounded Teller Tim, who gasped, “I’m a dead man. Get me a priest. Derringer shot me as he made his
getaway.” Tim then lapsed into a coma (昏睡) from which he has not emerged. May Malone testify to
Tim’s statement as a dying declaration?
- No. We have a belief of certain and impending death. “Get me a priest” is o.k. but “get me a doctor’
shows the declarant’s minds hoping to live. Declarant is not available because he is in coma. Whether
he died or not does not matter. But this is not a homicide case. 本件は殺人事件でないので ×。なお、
「牧師を呼んでくれ」は OK だが、「医者を呼んでくれ」は生きる望みがあるので ×。実際に死んだかど
うかは関係ない。

HYPO 69. Same bank robbery as Hypo 68, except now the lawsuit is a civil action against Derringer
for Tim’s personal injury damages. Tim is still in a coma. May Malone testify to Tim’s statement as a
dying declaration—
Yes. It’s a dying declaration in a civil case.民事事件であれば OK

63
Evidence 2016 Summer BarBri NY Bar

F-#2 SPONTANEOUS STATEMENTS 自然発生的供述


The next five exceptions rely on spontaneity to assure reliability. Unavailability is not required.
以降の 5 つの例外は、自然発生性を根拠としており、供述不能は不要

5. Excited Utterance 興奮による発声(803(2))


a) ELEMENTS:
(1) The statement concerns a startling event
驚くような出来事(下記 6 の Present Sense Impression との違い)
(2) And was made while the declarant was still under the stress caused by the event.
供述者がその出来事により生じた緊迫下にある状態での供述
PR-60 事件の直後でもよい。たとえば、交通事故の直後に現場で見ていた目撃者が加害者に
駆け寄り、加害者の不注意であることを叫んだような場合にもこの例外に該当する。
「 あ の 人 酔 っ 払 っ て 立 ち 上 が れ な い ね 」 と い う 発 言 は startling event が な い の で excited
utterance にはならない。Statement then existing mental condition を検討。

b) Rationale: Excitement suspends one’s capacity to fabricate.


興奮状態であれば捏造の意図はなくなるはず。
c) Factors that may make a statement “excited”:
(1) Nature of the event (dramatic/traumatic accidents or robbery are startling; forgery
or breaching of contract not startling) 出来事の性質
(2) Passage of time 時間の経過
・イベントのインパクトにもよるが 1 時間程度までは OK
OPE4-17 自身に関係しない事件で 2 時間経過したものは excited utterance 認められな
い。
BarBri WS-15 意識を取り戻した直後も認められ得る(WS Q15)

(3) Visual clues


(a) exclamatory phrase (e.g., good heaven, get zooks, oh my god)
(b) excitement- oriented verbs (e.g., shout, scream)
(c) exclamation points (!)
HYPO 70. Ernie observes a horrific head-on auto collision and excitedly tells a cop, who arrives 10
minutes later, “Oh my God, Officer! Both of those cars were going 80 miles an hour!” May the cop
properly testify to Ernie’s statement in subsequent litigation arising out of the accident?
- Yes. This is an exited utterance. Exclamation points matter.

6. Present Sense Impression 当該時点での認識による印象(FRE 803(1))


a) ELEMENTS:
(1) The statement describes an event; and 事件についての描写;
(2) Is made while the event is occurring, or immediately thereafter. 事件最中/直後
PR-28 交通事故のあと Emergency Room での被害者の発言は、“occurring, immediately
thereafter”とはいえない。
夫が盗人の自動車ナンバーを読み取りその場で妻に伝え、妻がその場で警察に電話で
伝えた場合、その電話記録は present sense impression になる。
・実況中継のようなもの。
・日本では認められていない類型なので注意!
b) Rationale: Contemporaneousness -- declarant has no time to fabricate. Does not need
startling events but must be immediate (in seconds, not in minutes)
事件の最中又は直後に行われる供述であるため、でっち上げる時間がない。人を驚かせるよう
な出来事である必要はないが(上記 6 の Excited Utterance との相違)、即時のものでなければ
ならない。数分経ってしまったら×(<=>Excited Utterance の場合には 10 分経過していても OK)
Emanuel-44 10 分前に交通事故を見た、という証人の供述は Presented Sense Impression の例
外に該当しない。
HYPO 71. Mom telephones her son Victor Victim at his apartment. “Mom,” he says, “Can you wait a
minute? Someone’s at the door.” Thirty seconds later, Victor gets back on the phone and says, “Mom,

64
Evidence 2016 Summer BarBri NY Bar

can’t talk now. My new friend, Hannibal Lecter, is here for dinner. Call you later.” The next day, the
remains of Victor’s dead body are found in his apartment. Hannibal is on trial for the murder. May Mom
testify that Victor identified Hannibal as his dinner guest that night?
- Yes. It’s admissible as present sense impression because it describes what’s happening while it
happens. If this is a criminal case, the constitutional confrontation clause issue is still unsettled in this
area.証拠能力あり。供述はそのとき何が起こっているかを描写しており、Present Sense Impression 当該時
点での認識による印象である。刑事事件であれば、対面条項が問題となるがまだ解決を見ていない。

7. Statement of Then-Existing Mental, Emotional, Intent, or Physical Condition


当該時点で存在した精神的、感情的、身体的状況の供述 (FRE 803(3))
この場合要証事実は、当該時点において供述者がそのような認識を有していたことであり、そのため
真実性の証明のために提出される証拠に該当し、non-hearsay にはならない。
a) THE RULE:
(1) A contemporaneous statement 同時的な供述
(2) Concerning the declarant’s then existing: 供述者に当該時点で存在した以下の事情
(a) Physical condition, or 身体的状況
BIM-53 痛みの存在を含む経験した兆候についての本人の供述は、医者などの医
療専門家に対してなされていなくても、証拠能力を有する。

(b) State of mind 精 神 的 状 況 (includes emotions, mental feelings, intent or


future plans, sensations, and bodily health 感情、精神的感覚、意図、将来の計
画、興奮、身体的健康)

(c) Declaration of intent to do something with a third person.


(3) But Not a statement of memory or belief about a past condition.
過去の状態についての記憶または信念の供述は× 将来の意図なら OK

b) Rationale: Matters about which the declarant has unique knowledge.


HYPO 72. Probate of Wanda’s Will, in which she left all her money to the local pet cemetery.
Wanda’s family challenges the will on the ground that Wanda was insane at the time of the will’s
execution. Pet cemetery offers testimony that a few days before execution of the will, Wanda said to her
Neighbor, “I do not love my family anymore.” Admissible over hearsay objection? 遺言の検認をしたら、
家族でなくペット協会に全ての資産を遺贈するとの記載。家族は精神異常の抗弁をしたが、ペット協会側
は遺言作成の 1 週間前に Wanda が隣人に「家族を愛することができない」といっていたと供述。
- Yes. This statement is about her present state of mind, present feelings. This exception overlaps with
the non-hearsay use of a statement. Doesn’t matter in Bar Exam. 当時の精神状態に関する供述。
HYPO 73. Susan’s Family v. Life Insurance Co. for nonpayment of proceeds upon Susan’s death.
Defense: Suicide. Life Insurance Co. seeks to introduce a note found in Susan’s apartment (in Susan’s
handwriting) in which she said “I’m going to end it all next week.”
- Admissible. This is a statement of a future intent. 将来の意図に関するもの。自殺を示唆するものとし
て関連性あり。

HYPO 74. State v. Raymond for murder of Victim. Before going out Monday night, Victim told wife,
“I’m meeting Raymond tonight at the bowling alley.” Victim’s dead body was found Tuesday morning
outside the bowling alley. Is Victim’s statement to his wife admissible? 殺人の被害者が、外出前に妻に
対して、「今夜 Raymond とボウリング場で会ってくる」といっていた。被害者の死体がボウリング上の
外で発見された。妻の供述は証拠能力があるか。
- Yes. This is a statement of future intent. This can be used to prove that both Victim and Raymond
were at the bowling alley. 将来の意図の供述として証拠能力あり。供述の時点から考えて将来かどうかが
問題。
BSM-58 被告人が Robbery の罪で起訴されている時間帯(夕方)に、他の場所にいたことを立証するため
に、証人を呼び「犯罪の行われた日の午前中に被告は『今日の午後は義理の母親の誕生日パーティーに参
加する』といっていた」と供述。この場合、被告の State of Mind は直接的には争点となっていないが、将
来の意図としてその時点での State of Mind を供述しているのであって、状況証拠として証拠能力あり。
HYPO 75. Plaintiff, whose arm was broken in accident with Defendant in January, sues for damages
for pain and suffering. At trial in December, Plaintiff testifies about the pain she experienced. Plaintiff
also calls Neighbor to testify (a) “I was with Plaintiff last July when she said, ‘I’m feeling a lot of pain
in my arm’ and again in December when she said (b) ‘I sure did feel a lot of pain in my arm last July.’”
Admissible over hearsay objections?

65
Evidence 2016 Summer BarBri NY Bar

- (a) is admissible but (b) is not. (a) is a statement of present physical condition. This is not a party
admission because this is not a statement of the opposite party. But (b) is a statement of past physical
condition, not a statement of present condition.
(b)は(供述当時から見て)過去の身体的状況についての供述なので証拠能力なし。

8. Statement for Purpose of Medical Treatment or Diagnosis


医学的治療、診療目的の供述(FRE 803(4))
a) ELEMENTS: A statement:
(1) Concerning: 以下のいずれかに関連する必要がある。
(a) Present symptoms 現在の兆候(上記 7 でも同じ)
(b) Past symptoms, 過去の兆候(上記 7 では過去の兆候は含まれない。医者に対する発言で
あるという要件を満たせばこちらに含まれる。)or
(c) General cause of a medical condition 医学的状態についての一般的な理由
(2) For the purpose of treatment or diagnosis. 治療・診療を受ける目的
(3) But not: Statements describing the details of liability or the identity of the
wrongdoer, unless it is the identity of the abuser in a domestic abuse or child
abuse case.
過失または違法行為者の同一性については原則として含まない→治療とは無関係のため
患者と医者の間の通信の特権に含まれる場合でも、患者側が主張しようとする場合、伝聞証
拠の問題となり、例外として証拠能力が認められることになる。特権を行使して秘密として
保持することも可能。
BRQ-14 医師に対する供述が対象となるが、医師による診察の結果、医師が得た所見・感想は
例外に含まれない。
事故の直前、重労働をしていたこと、も medical history になる。

b) Rationale: Patients have an incentive to be honest and accurate to get good medical
care.「患者はよい治療を受けるために正直かつ正確に話をしようとする incentive が働く」
HYPO 76. Paula v. Defendant for pain-and-suffering damages based on alleged accident at
Defendant’s store. Defendant disputes both the happening of the accident and damages. At trial, Paula
calls her treating physician to testify, “When Paula came to see me for treatment a year after the
accident, she said—
(a). ‘The pain in my arm is killing me.’
- Admissible as statement of present symptoms.
(b). ‘I’ve been losing sleep at night for the past six months because of the pain in my arm.’
- Admissible as statement of past symptoms made to a medical professional for the purpose of
treatment.
(c). ‘This all started when I fell down the stairway—’
- Admissible as statement of general cause of a medical condition.
(d). ‘—the poorly maintained stairway at Defendant’s store.’”
- Not admissible. This is about fault and the identification of the wrongdoer.
(c)はあくまで general cause of the condition であるのに対し、(d)は details of liability or the identity
of the wrongdoer になってしまっている。

66
Evidence 2016 Summer BarBri NY Bar

9. Business and Public Records 事業及び公的記録 (FRE 803(6)(8))


a) Business Records
(i) ELEMENTS
(1) Records of a business 事業上の記録
any type of association and organization, including public agencies such as police
(2) Made in the regular course of business 通常業務において作成
(i.e., germane to the business)
(3) The business routinely keeps such records 記録を常時保管
(4) Made contemporaneously 同時になされた
(at or about the time of the event recorded), and
(5) The contents consist of:
(a) Information observed by employees of the business, 従業員が看取した情報 or
(b) A statement that falls within some independent other hearsay exception.
その他の伝聞証拠の例外の範囲内の供述
従業員が看取した情報であっても、その従業員が事故を起こしたかどうかが争点と
なっている場合、その従業員はうその供述をする動機を有しているため、証拠能力は
認められない。
PR-12 Business Report に記載される内容について Firsthand Knowledge を有する者がい
る 場 合 に は 、 そ の 者 の 直 接 の 証 言 に 証 拠 能 力 が 認 め ら れ る 。 Business Report が
Hearsay の例外として Admissible と評価される場合であっても、直接の証言自体は制
限されない。
BRQ-16 訴訟のために作成されたレポートは含まれない。(Self-Serving)
Business Record に記録されていないことを、当該事実の不存在の証明として用いるこ
とも可能。(Business Record の使用の可否が問題になっているに過ぎないから)
BarBri Set 6-15 作成者にとって文書の作成が business activity 上の義務であることが必
要。職場で起きたセクハラに関して従業員が書いたクレームは business record にはな
らない。(人事部のファイルだったら business record になりうる)

(ii) Rationale:
Reliable because businesses depend on accurate, up-to-date record-keeping, and
practical because such records are often the most efficient way to prove relevant
facts.

b) Public Records 公的記録:


(1) Elements:
(a) The record is made by a public employee;
(b) The record is made within the scope of duty of the public employee;
(c) The record is made contemporaneously; AND
(d) The record is trustworthy.
(2) What can be Admitted:
Records of a public office or agency setting forth any of following:
(a) the internal activities of the office or agency (e.g., payroll records); or
(b) matters observed pursuant to a duty imposed by law (e.g., Weather Bureau
records of temperature); or
(c) findings of fact or opinion resulting from an investigation authorized by law
(e.g., OSHA inspection report on safety conditions of factory).

(3) Exclusion:
Police reports prepared for prosecutorial purposes may not be offered against

67
Evidence 2016 Summer BarBri NY Bar

the defendant in a criminal case. Nor is the prosecution in such cases allowed to
introduce such reports against the defendant under the alternative theory of
business records.
起訴目的で作成された警官の報告は刑事事件の被告人に対して用いることはできない。
Police reports are admissible ONLY in civil cases 民事事件でのみ OK.
HYPO 77. Pedestrian sues Hot Rod Kid for damages for recklessly running him down. At trial,
Pedestrian seeks to introduce the report of Officer Muldoon, who arrived at the scene ten minutes after
the accident. The report, which was prepared by Muldoon at the scene, states:
(a). “Upon arrival, I measured skid marks 50 feet in length.”
- Admissible as a business record. Police is a business and it’s germane to the business, within
regular practice of business and made contemporaneously by a police officer.
(b). “Officer Mendez, who witnessed the accident, told me that Hot Rod Kid was driving nearly 60
miles per hour.”
- Admissible as a business record. Because Mendez is also an employee of the business. If all
involved in the record are employees of the business, the record can be a business record. ビジネス
上の報告は正確に行われるはずである(business incentive/ duty があるから)
(c). “Bill Bystander told me, ‘I saw the accident and Hot Rod Kid ran through the stop sign.’”
- Inadmissible. Because Bill is not an employee of the business, thus cannot be included in the
business record. If it falls within one of other exceptions (e.g. exited utterance, etc.), then may be
admissible. 事故報告書の記載であっても、供述者が警官でなく Lay Person であるので、Business
Record にはならない。ただし、他の例外(Excited Utterance)に該当して、証拠能力がある場合もあ
る。

68
Evidence 2016 Summer BarBri NY Bar

F-#3 OTHER EXEPTION


10. Other Exception
(1) Records of Felony Conviction (FRE803(22))重罪有罪判決の記録
Felony Conviction の記録(刑事有罪判決)の写しは、伝聞証拠である。
しかし、伝聞証拠の例外に該当するものである。したがって、弾劾目的の利用のみならず、刑
事・民事を問わず判決の本質的内容(有罪判決で認定された犯罪事実)について証拠能力が認め
られる。
刑事事件の有罪判決で認定された犯罪事実に基づいて、有罪判決の記録を出せば、民事事件で
主張することができる。Misdemeanor(軽罪)の場合には伝聞証拠の例外に該当しないが、真実
性に関するものは弾劾目的で利用可能。
(2) Past Recollection Recorded 記録された過去の記憶(IV.D.2 参照)
記録自体を提出する場合には、以下の 5 つの要件を満たすことにより、伝聞証拠の例外となる。
1) The witness once had personal knowledge, 個人的な知識があった。
2) The witness now forgets, and showing the writing to witness fails to jog the
witness’s memory, 現在忘れている。
3) The writing was either made by the witness or adopted by the witness,
証人によって作成され、又は証人によって採用された書面
4) The writing was made when the event was fresh in the witness’s memory.
証人の記憶が新鮮なときに書面が作成された。
5) And, the witness can attest that, when made, the writing was accurate.
作成時において正確であったと被告が証言できる。
*証人による Unavailability 証言の不能の要件は必要とされず、証拠能力が認められる。

(3) Learned Treatise in Aid of Expert Testimony (Hearsay Exception)


専門家証人の補助目的の専門的学術書・論文(伝聞証拠の例外)(IV.E.2.d)参照)

(4) Pedigree Exception 家系図についての例外


家族、血族、離婚、死亡、養子、出生などについての評判などは伝聞証拠の例外となる。
eg. 執事がその家の子供に対して、家の主と妻の間の子供であると伝えていた場合。
Unavailability is required. 証言不能であることが必要。

(5) Family History Record 家族の歴史の記録 BarBri Set 5-11


家族のメンバーの出生日、出生地、死亡日、婚姻などの関係についての事実を含む。
Statement of fact concerning personal or family history contained in family Bibles, genealogies ( 家 系 図 ), charts,
engravings on rings, inscriptions on family portraits, engraving on urn, crypts, or tombsome

(6) Ancient Documents 古文書

69
Evidence 2016 Summer BarBri NY Bar

Hearsay Exceptions
Non-Hearsay
Unavailability Required Unavailability Not required
Impeachment Former Testimony Excited Utterance
(i) unavailable (i) startling event
(ii)formal proceeding/ deposition (ii) made under the stress (immediately
(iii)opportunity to cross-examine with after the event)
same motivation
Verbal Acts Forfeiture for wrongdoing Present Sense Impression
(i) description of event
(ii) while event if occurring or
immediately thereafter
Show Effect on Person Who Statement Against Interest Then Existing State of Mind
Heard or Read the Statement (i) unavailable (i) present physical condition or state of
(ii) statement against declarant’s mind, including future intention
interest when made

Circumstantial Evidence of Dying Declaration Medical Treatment


Speaker’s State of Mind (i) civil or criminal homicide (i) made to medical professional
(ii) unavailable (ii) present or past symptom
(iii) made under a belief of impending or cause of medical condition
death (iii) for the purpose of treatment or
(iv) concerning the cause of diagnosis
circumstance of death
Prior Identification statement Business Record/Public Record
by Present Trial Witness (i) record of business
(ii) made in regular course of business
(iii) as regular basis record
(iv) information* observed by employee
*criminal case では police report は
against defendant の証拠として提出できな

*public record の場合は observation の
conclusion も含む
Prior Inconsistent Statement Records of Felony Conviction
made under Oath by a Present
Trial Witness Past Recollection Recorded
Prior Consistent Statement of Learned Treaties
Present Trial Witness Offered
to Rebut motive to lie Pedigree Exception
Party Admission (including Family History Record
vicarious liability*)
(i)offered against declaring
party Ancient Documents

70
Evidence 2016 Summer BarBri NY Bar

E. THREE FINAL MISCELLANEOUS HEARSAY ISSUES


1. Procedural Issue: Laying the Foundation for Hearsay Exceptions
伝聞証拠の例外のための前提事実の立証(FRE 104(a))
THE RULE:
Whether a party has established the required elements of a hearsay exception is to be
decided by the judge [by preponderance of evidence]. And in making that determination,
the judge may rely on anything (including inadmissible evidence).
当事者が伝聞証拠例外の要件を立証したかどうかは証拠の優越により裁判官が判断する。裁判官は
決定に当たり、証拠能力のない証拠も含めて何に依拠してもよい。
BIM-32
証言が Hearsay の例外に該当するかの判断は裁判官が行う。裁判官が判断をするに当たっては、一
般的に証拠ルールに拘束されない。証拠の適格性は preliminary fact の決定によるからである。その
場合には、特権で保護されていない関連する証拠、証拠ルール上は証拠能力がないと判断されるよ
うな証拠も考慮に入れて判断することができる。
PR-53 Preliminary Question
以下のような Preliminary Question についての判断は Judge が行う。
(1) if a dying declarant had a sense of impending death. 臨終供述者が死に瀕しているかどうか。
(2) if an entry was made promptly in the regular course of business 通常の業務に従って記載がされている
か。
(3) if there was a necessary state of excitement to qualify a declaration as an excited utterance.
供述を興奮による発生と判断するために必要となる興奮状態があったか。
(4) if a witness is unavailable. 証人が供述不能か。
(5) if an original document is unavailable so as to justify the admission of a copy under the best evidence rule.
最良証拠ルールにおいて写しの証拠能力を認めるのに原本が利用不可能か。
(6) 専門家証人の専門家としての適格性

2. Note on the Confrontation Clause 対面条項(刑事事件のみ)


a) THE RULE:
In criminal cases, the Sixth Amendment requires that the defendant be “confronted”
with the witnesses against him. Confrontation is the opportunity for cross-examination
by the defendant.
刑事事件では修正第 6 条により被告人は証人に対する反対尋問の機会を与えられなければなら
ない。
b) Application of the rule to trial hearsay 公判での伝聞証拠への適用.
In the context of hearsay, the prosecution may NOT use a hearsay statement against
the criminal defendant (even if it falls within a hearsay exception) if:
(1) the statement is Testimonial,
(2) the declarant is unavailable, and
(3) the defendant has had no opportunity for cross-examination (cross- examination
requirement may be satisfied either before or at trial).
注:対面条項は証人の証言が Testimonial 証言証拠である場合にのみ適用される。
対面条項が適用されると、伝聞証拠において被告人は証人と対面していないため、伝聞証拠の
証拠能力が否定される。
OPE4-31 刑事事件で共同被告人 A と B がいる場合、A の供述を伝聞証拠として提出すること
はできない。A にとっては party admission により伝聞例外になるが、B には反対尋問の機会
がないため。
この場合、A との関係でのみ証拠とする旨の limitation があっても憲法違反と解されている。

71
Evidence 2016 Summer BarBri NY Bar

c) What is a “testimonial” statement?


The meaning of “testimonial” statements is still being developed, but the following
applications have been established:
(1) Grand jury testimony is testimonial. 大陪審での証言
(2) Statements in response to police interrogation: 警察の取調べ
(a) Testimonial if the primary purpose of the questioning is to establish or prove
past events that are potentially relevant to later criminal prosecution.
Example: Police arrive at domestic abuse crime scene, arrest the abuser, and
sit down with the victim to ask “what happened?” At defendant’s trial, the
victim refuses to testify. The victim’s statements to the police, even if they fall
within a hearsay exception (e.g., excited utterances), cannot be used against
the defendant because they are testimonial, the witness is now unavailable,
and there has been no cross-examination.
(b) Non-Testimonial if the primary purpose of the questioning is to enable
police assistance to meet an ongoing emergency.
Example: Domestic abuse victim calls 911 to seek help while abuser is in
other room with plans to resume the abuse.
事件を目撃した証人の警察への 911Call の録音テープは、Present Sense Impression とし
て、伝聞証拠の例外となる。しかし、 Confrontation Clause との関係では、Testimonial
かどうかが問題となるが、警察への 911Call は Testimonial には含まれないため、証拠
能力が認められる。
“Ongoing emergency” includes a situation in which the crime has recently
ended, the perpetrator is armed, and he still poses a threat to the victim, the
police, or the public at large.
BarBri Essay-7 Testimonial かどうかの判断基準:primary purpose で判断
(1) 現在、現に生じている危機の解決のための供述は testimonial に該当しない
(2) 今後の(潜在的な)起訴のための、過去の出来事を確認・立証するための供述は
testimonial に該当する。

(3) Documents
(a) Business records are Non-testimonial.
(e.g., bank or phone company records).
(b)Sworn Affidavits are testimonial.
(c) A forensic laboratory report is testimonial,
if its primary purpose is to accuse a targeted individual of criminal conduct.
Examples:
(1) Analysis of drugs seized from a particular suspect to ascertain if the drugs
are cocaine, etc. 特定被疑者から押収した薬物がコカインなのかを確認する場合
等は証言としての性質を持つ
(2) Analysis of the blood of a suspected DWI driver to ascertain his blood
alcohol content. 飲酒運転をしたとされる運転手についての血中アルコール量の
測定も証言としての性質を持つ
But a DNA report is Non-testimonial if it analyzes a sample of bodily fluid
collected from a crime scene for the purpose of developing a DNA profile if no
particular person is suspected at the time of the analysis. (blind DNA analysis
test) 現場から得られた DNA サンプルは、特定の人を対象としていなければ証言としての
性質を持たない

Even if a forensic report is testimonial, no confrontation violation occurs if the

72
Evidence 2016 Summer BarBri NY Bar

prosecutor calls a testifying expert who performed an independent analysis of


the data, and the testifying expert only generally refers to the report to show a
partial basis for her opinion without reading the report to the jury or introducing
it as an exhibit. Then the report is not being used for a hearsay purpose.
当該分析を行った専門家が証人となった場合に、鑑定書を参照したとしても、一般的参照
に止まり、陪審員に対する読み上げ・展示がされなければ、証人対面権違反ではない
3. Hearsay Declarants and Impeachment 伝聞証人及び弾劾(FRE 806)
THE RULE:
If hearsay is admitted, the opposing party may use any of the impeachment methods to
attack the credibility of a hearsay declarant.
伝聞証拠に証拠能力が認められた場合、反対当事者は伝聞証人の信用性を攻撃するためにいかなる
弾劾方法もとることが可能。
HYPO 78. Shooter is on trial for murder of Victim. In hospital bed, Victim told the nurse, “I’m feeling
pretty good considering Billy Ray tried to kill me.” The next day, Victim told a cop, “I know I’m about
to die. Shooter’s the one who shot me.” Victim died.
(a) May the prosecution introduce Victim’s statement to the cop over a hearsay objection?
- Yes. It’s dying declaration.
(b) If the prosecution introduces Victim’s statement to the cop, may Shooter introduce Victim’s
statement to the nurse over a hearsay objection?
- Yes. It’s not a dying declaration thus is not admissible but can be used for impeachment.
Impeachment purpose is non-hearsay use.

73
Evidence 2016 Summer BarBri NY Bar

VII. Judicial Notice 裁判所による確知


1. Definition: Judicial Notice is the recognition of a fact as true without formal presentation
of evidence 公式な証拠の提示なく事実が真実であることを確認する手続
裁判所による確知:裁判所が、訴訟手続上、証拠によらずに、一定の事実の存在を認める行為。
その事実については、当事者による立証は不要となり、反証も許されなくなる。これが許される範
囲は、公知の事実や、確実な根拠により正確かつ容易に認知することのできる事実に限られている.
to take judicial notice =「裁判所に顕著なものと認める」こと

2. Rule
Court may take judicial notice of indisputable facts, in the following forms:
裁判所は争いのない事実につき以下の方法で確知することができる。
a) Notorious Fact (well known fact)
Matters of common knowledge within the courts territorial jurisdiction 一般常識
BarBri Set 2-7
Notorious Fact については情報源への参照不要(manifest fact は情報源への参照を前提にして
いる点で異なる)。
community knowledge としての常識であれば良い。
例えば当該市内の city hall の場所は当該 community においては常識に該当する。

b) Manifest Fact
Matters capable of easy verification by resort to unquestionable sources
確実な情報源によって容易に認証可能な事項
PR-102 裁判所は、Ultimate Fact 主要事実について、Judicial Notice による認定を行うことに
は消極的。
PR-10 裁判所はレーダー装置(道路のスピード違反検査)についての有効性について裁判所
に顕著なものとして認めている。
PR-108 裁判所は、血液検査について、父性の確認をのぞき、検査者の適格性が確認され、
検査方法に問題がないと判断される場合には、検査の正当性についての裁判所による確知を
認めている。
PR-144 裁判所が適用すべき法律
最高裁判所は、Diversity Case においては Substantive Law が適用されるべきであるとし、連邦
裁判所では、連邦裁判所が所在する州の裁判所によって適用される法律と同じ法律を適用す
べきと判断した。 Erie R.R. v. Tompkins
BarBri Set 1-17 外国法についての judicial Notice は認められない。Legislative fact はカバーし
ない。

3. Timing
Judicial notice may be taken at any time, including on appeal
上訴の時を含めていつでも確知できる。
PS-126 A court must take judicial notice of an adjudicative fact if (i) requested by a party and (ii) the court is
supplied with the necessary information. (i)当事者による請求が行われ、かつ(ii)必要な情報が裁判所に
提出された場合には、裁判所は司法事実(adjudicative fact)については裁判所による確知を行わな
くてはならない(確知義務)。
司法事実:法律上の紛争で争われる当該事件に固有の事実(<=>legislative fact (立法事実))

4. Effect
(a) Civil Case
Judicially noticed facts are considered conclusive in civil cases, but not criminal cases.
確知によって認定された事実は民事事件では最終的判断とみなされる。これに対して刑事事件
では最終的判断とはならない。
(b) Criminal Case
The jury may, but is not required to, accept the fact noticed.
刑事事件では陪審員は確知された事実を認めることが出来るが、認める必要はない。
The effect is only to relieve the prosecutor of her burden of producing evidence on
that fact. 確知の効果は検察官のその事実に対する証拠提出責任を緩和するのみである。
*Not establishing the fact beyond a reasonable doubt.

74
Evidence 2016 Summer BarBri NY Bar

確知によって合理的な疑いを超える事実が認定されるわけではなく、検察官が証拠を提出しな
くてよい、というだけ。
BarBri-WS-3 Shift of burden of proof, shift of burden of persuasion も生じない。

75
Evidence 2016 Summer BarBri NY Bar

VIII. Real Evidence 物的証拠


A. DEFINITION:
Real Evidence: actual physical evidence that is displayed to the trier of fact
Ex. drugs, guns
B. AUTHENTICATION RULE
1. Rule: Party seeking to introduce real evidence must introduce sufficient evidence that the
item is what it claims to be. 物的証拠の提出を求める当事者は、主張しようとしている物であるこ
との十分な証拠を提出しなければならない。
2. Process of proving called laying the foundation
PR-33 科学的な証拠は、実験装置などが合理的なものであり、問題なく作動していることの
Foundation が必要である。

3. Method of Proof: 証明方法


a) Personal Knowledge – call witness to testify 個人的知識
b) Chain of Custody 物証保管の継続性
NOTE: chain must be substantially unbroken (must be based on reliable procedures for
identification and custody) 完璧に継続している必要はないが、実質的に継続性が破壊されていな
いことが必要。
BIM-18 Chain of Custody 物証保管の継続性と証拠の真正性の問題。警察官が犯行現場で血液を採
取したあと、行方をくらました。その後科学捜査研究所に当該犯行現場で採取された血液である
ことが記載されたガラス瓶が置かれており、それを鑑定したところ被告人のものであった。この
場合、血液採取から血液鑑定までの Chain of Custody 物証保管の継続性がないことになり、血液
検査の結果の真正性が認められない。検察側は継続性があったことを証明しなければならない。
BarBri Set 3-6
Chain of custody が問題になるのは他の証拠と混同・取違いが生じる証拠に限定される。
例えば、銃の場合、シリアルナンバーで区別できるため、chain of custody の問題は生じない。

4. Condition of Real Evidence 物的証拠の状態


If the condition of the item before trial is relevant, it must be shown at trial to be in
substantially the same condition.
BSM-162 Real Evidence の Authentification が行われた場合には、証拠を Jury の前に提示してもよい。
例えば、クリーニングでひどい匂いのついたコートについて損害賠償請求を行う場合に、 Jury に対
してにおいのついてしまったコートの匂いをかがせることができる。
PR-110 Demonstrative Evidence
裁判所は父性認知訴訟において裁量で、子供の顔が父親と似ているということを証明させるために
子供を展示証拠として在廷させることができる。

76
Evidence 2016 Summer BarBri NY Bar

IX. PROCEDURAL CONSIDERATIONS: Dividing responsibility between


the judge and the jury.
A. BURDENS OF PROOF
1. The usual standard in civil cases: Preponderance of evidence
2. The standard in criminal cases: Beyond reasonable doubt
After all the evidence is in, it is up to the jury to decide whether the burden of proof has
been met for each element of the charge, claim, or defense.
B. PRELIMINARY FACTS
3. Preliminary Facts for the jury
a) The jury decides question of conditional relevance, which come in three forms:
(1) Whether a witness has personal knowledge
(2) Whether an exhibit is authentic
(3) Whether the defendant is in fact the person who committed a bad act offered as
MIMIC evidence
b) The judge’s role for such questions is simply to ensure that there is sufficient
evidence for a reasonable jury to conclude that the conditional fact is true (FRE
104(b))
4. The judge decides question of admissibility, for example:
a) Whether the testimony is hearsay
b) Whether communication is privileged
c) Whether the expert is qualified
NOTE: For these questions, the burden of proof is preponderance of evidence, and the
judge may consider anything (i.e., the judge is not limited to admissible evidence.)
* Whether the jury should be excused during the judge's preliminary fact finding is
generally within the judge's discretion.
5. Instructions on Limited Admissibility of Evidence (FRE 105)
If evidence is admissible on a limited basis, the judge must, upon timely request, restrict
evidence to its proper scope and instruct the jury accordingly.
e.g.) Dual purpose evidence: When evidence is admissible for one purpose, but
inadmissible for another purpose, the judge should give the jury a limiting instruction.
ある目的について証拠能力があるが他の目的については証拠能力がない場合、裁判官は陪審員に説
明しなくてはならない。

BarBri Set 2-3 法律上の推定(presumption)


前提事実が立証された場合、推定される事実に関し推定が生じる。
この場合、他に推定を覆す証拠が提出されていない場合、 Jury は当該推定に反する事実認定を行うことがで
きない。

BarBri Set 2-15 Burden of Persuasion と Burden of going forward with evidence
Burden of Persuasion は、証拠が双方から提出され、jury の判断がどっちつかず(issue is equally balanced in the
mind)の場合にいずれの当事者が敗訴するかという問題であり、訴訟進行に伴って shift しない。
Burden of going forward with evidence は、いずれの当事者が証拠提出責任を負うかという問題であり、訴訟進
行に伴って shift する。

THE END

77
Evidence 2016 Summer BarBri NY Bar

その他
Emanuel -25 民事事件において、被告が同一事件の刑事事件で有罪判決を受けた旨を立証する方法として、
原告は、原告自身が刑の宣告を聞いた旨を証言することができる( testimony of the plaintiff, who was present at
the time of the sentence)。

BarBri-WS-18 書面の一部について証拠採用した場合、他方当事者は同一の書面の別の部分についても証拠
として採用することを求めることができる。

OPE4-72 郵便物について適切な宛先に、stamp を貼って送付した場合、receive されることが推定される。た


だし、上記推定は rebuttable であり、例えば、受信側の担当者が受診していない旨証言することが反証する
ことが可能。

78

You might also like